CME4PAC

Online CME for Physician Assistants

Menu

Skip to content
  • Home
  • Membership Benefits
  • My Account
  • About Us
  • Log In
  • Contact Us
  • Ortho on the Go

2022 ortho questions

Time limit: 0

Quiz-summary

0 of 64 questions completed

Questions:

  1. 1
  2. 2
  3. 3
  4. 4
  5. 5
  6. 6
  7. 7
  8. 8
  9. 9
  10. 10
  11. 11
  12. 12
  13. 13
  14. 14
  15. 15
  16. 16
  17. 17
  18. 18
  19. 19
  20. 20
  21. 21
  22. 22
  23. 23
  24. 24
  25. 25
  26. 26
  27. 27
  28. 28
  29. 29
  30. 30
  31. 31
  32. 32
  33. 33
  34. 34
  35. 35
  36. 36
  37. 37
  38. 38
  39. 39
  40. 40
  41. 41
  42. 42
  43. 43
  44. 44
  45. 45
  46. 46
  47. 47
  48. 48
  49. 49
  50. 50
  51. 51
  52. 52
  53. 53
  54. 54
  55. 55
  56. 56
  57. 57
  58. 58
  59. 59
  60. 60
  61. 61
  62. 62
  63. 63
  64. 64

Information

2022 ortho questions

You have already completed the quiz before. Hence you can not start it again.

Quiz is loading...

You must sign in or sign up to start the quiz.

You have to finish following quiz, to start this quiz:

Results

0 of 64 questions answered correctly

Your time:

Time has elapsed

You have reached 0 of 0 points, (0)

Categories

  1. Not categorized 0%
  1. 1
  2. 2
  3. 3
  4. 4
  5. 5
  6. 6
  7. 7
  8. 8
  9. 9
  10. 10
  11. 11
  12. 12
  13. 13
  14. 14
  15. 15
  16. 16
  17. 17
  18. 18
  19. 19
  20. 20
  21. 21
  22. 22
  23. 23
  24. 24
  25. 25
  26. 26
  27. 27
  28. 28
  29. 29
  30. 30
  31. 31
  32. 32
  33. 33
  34. 34
  35. 35
  36. 36
  37. 37
  38. 38
  39. 39
  40. 40
  41. 41
  42. 42
  43. 43
  44. 44
  45. 45
  46. 46
  47. 47
  48. 48
  49. 49
  50. 50
  51. 51
  52. 52
  53. 53
  54. 54
  55. 55
  56. 56
  57. 57
  58. 58
  59. 59
  60. 60
  61. 61
  62. 62
  63. 63
  64. 64
  1. Answered
  2. Review
  1. Question 1 of 64
    1. Question

     

    A 65-year-old male presents to the emergency room with fingertip amputations to the thumb, index, and middle fingers after a table saw injury two hours earlier. AP and lateral radiographs of the hand (figures 1 and 2) show bone loss of the distal phalanx of the thumb, index, and middle fingers. Figure 3 and 4 are pictures of the patient’s dorsal hand showing the fingertip amputations. The index and middle finger have more dorsal skin loss with a dorsal oblique fingertip amputation pattern. The thumb has more volar involvement with a volar oblique type amputation pattern. Which statement is true regarding management of fingertip amputations?

    Correct

    Fingertip amputations can be extremely painful and complicated injuries that are best treated by a hand specialist. The goal of treatment is to save as much of the finger as possible while maintaining a functional digit. Factors that determine treatment include how much soft tissue is left on the finger, orientation of amputation (guillotine vs oblique), bone exposure, and whether there is more dorsal or volar skin remaining. Partial fingertip amputations (<1.5 cm) without bone exposure and adequate skin volarly can often heal with primary closure or healing with secondary intention. Healing by secondary intention usually occurs by 4 weeks and is associated with less time out of work and few complications than flap procedures. Injuries with exposed bone and limited soft tissue coverage volarly often require removal of some or all of the distal phalanx to allow for skin closure. Partial remaining nail bed should be visualized and removed to prevent a hook nail deformity and digital nerves must be transected as far proximally as possible to prevent a painful neuroma. 1,2
    Answer C.
    References
    1. DaCruz DJ, Slade RJ, Malone W. Fractures of the distal phalanges. The Journal of Hand Surgery: British & European Volume. 1988 Aug 1;13(3):350-2.
    2. Neustein TM, Payne SH, Seiler JG. Treatment of Fingertip Injuries. JBJS Reviews - Review Articles: 21 April 2020 - Volume 8 - Issue 4 - p. e018

    Incorrect

    Fingertip amputations can be extremely painful and complicated injuries that are best treated by a hand specialist. The goal of treatment is to save as much of the finger as possible while maintaining a functional digit. Factors that determine treatment include how much soft tissue is left on the finger, orientation of amputation (guillotine vs oblique), bone exposure, and whether there is more dorsal or volar skin remaining. Partial fingertip amputations (<1.5 cm) without bone exposure and adequate skin volarly can often heal with primary closure or healing with secondary intention. Healing by secondary intention usually occurs by 4 weeks and is associated with less time out of work and few complications than flap procedures. Injuries with exposed bone and limited soft tissue coverage volarly often require removal of some or all of the distal phalanx to allow for skin closure. Partial remaining nail bed should be visualized and removed to prevent a hook nail deformity and digital nerves must be transected as far proximally as possible to prevent a painful neuroma. 1,2
    Answer C.
    References
    1. DaCruz DJ, Slade RJ, Malone W. Fractures of the distal phalanges. The Journal of Hand Surgery: British & European Volume. 1988 Aug 1;13(3):350-2.
    2. Neustein TM, Payne SH, Seiler JG. Treatment of Fingertip Injuries. JBJS Reviews - Review Articles: 21 April 2020 - Volume 8 - Issue 4 - p. e018

  2. Question 2 of 64
    2. Question


    A 44-year-old male presents to your office with right knee pain and weakness after a skiing injury 2 days prior. He was following his son off a ski jump when he landed and his knee was forced in deep flexion. He felt a pop in the right knee just after he landed. He was unable to ski down the mountain due to the pain and weakness. AP and lateral x-rays of the right knee taken at an urgent care clinic the day of injury are shown in figures 1 and 2. On physical exam he has severe anterior knee pain and is unable to extend his knee. What is the most likely diagnosis?

    Correct

    The lateral radiograph demonstrates an obvious patella alta or high riding patella. This finding is consistent with a patella tendon rupture. The patella tendon is able to withstand significant forces during knee motion considering 3.2 times body weight is generated through the knee while climbing stairs. Patella tendon ruptures most commonly occur in patients over 40 years old during a sudden quadriceps contraction. Patients on chronic oral corticosteroids and with a history of patella tendinitis are the most prone to tendon rupture.  The tendon typically tears off the inferior pole of the patella. Most patella tendon ruptures can be diagnosed with a careful physical exam and x-ray finding as above, and therefore MRI is only used if the diagnosis is in question. Patella tendon ruptures require timely surgical repair within two weeks of injury. Rehab after primary repair includes touch down weight bearing for 6 weeks with a gradual increase in weight bearing when quadriceps strength returns. The prognosis after a timely repair is excellent with most patients having a complete return of knee motion and strength. 1,2

    Answer A.

    References

    1. Matava, Matthew J. MD Patellar Tendon Ruptures, Journal of the American Academy of Orthopaedic Surgeons: November 1996 – Volume 4 – Issue 6 – p 287-296
    2. Patella tendon rupture. http://www.orthobullets.com. Accessed on 3/5/21.
    Incorrect

    The lateral radiograph demonstrates an obvious patella alta or high riding patella. This finding is consistent with a patella tendon rupture. The patella tendon is able to withstand significant forces during knee motion considering 3.2 times body weight is generated through the knee while climbing stairs. Patella tendon ruptures most commonly occur in patients over 40 years old during a sudden quadriceps contraction. Patients on chronic oral corticosteroids and with a history of patella tendinitis are the most prone to tendon rupture.  The tendon typically tears off the inferior pole of the patella. Most patella tendon ruptures can be diagnosed with a careful physical exam and x-ray finding as above, and therefore MRI is only used if the diagnosis is in question. Patella tendon ruptures require timely surgical repair within two weeks of injury. Rehab after primary repair includes touch down weight bearing for 6 weeks with a gradual increase in weight bearing when quadriceps strength returns. The prognosis after a timely repair is excellent with most patients having a complete return of knee motion and strength. 1,2

    Answer A.

    References

    1. Matava, Matthew J. MD Patellar Tendon Ruptures, Journal of the American Academy of Orthopaedic Surgeons: November 1996 – Volume 4 – Issue 6 – p 287-296
    2. Patella tendon rupture. http://www.orthobullets.com. Accessed on 3/5/21.
  3. Question 3 of 64
    3. Question


    A 15-year-old male presents to an urgent care facility with left knee pain from an injury earlier in the day. He was playing football when he tripped and the knee struck a cement wall with a direct impact. He had difficulty bearing weight after the injury and had severe pain over the anterior knee. On physical exam the patient has a small abrasion over the anterior patella and he is able to perform a straight leg raise. AP and lateral x-rays are shown in figures 1 and 2, respectively. What is the next best step in treatment?

    Correct

    The patella is the largest sesamoid bone in the body which has the quadriceps tendon attached proximally and the patella ligament attached distally.  The quadriceps, patella, and patella ligament form the extensor mechanism of the knee. Disruption in any of these three structures may cause an inability to straight leg raise or bear weight on the knee. Patella fractures may be treated non-operatively if the extensor mechanism is intact, there is less than 2-3 mm of articular step off, and less than 4mm of fracture displacement. Non-operative acute treatment includes weight bearing as tolerated in a knee immobilizer or hinged knee brace locked in extension.  Weight bearing as tolerated is permitted in a knee immobilizer as the extensor mechanism is not activated with the knee in extension. Any type of flexion, especially with weight bearing, activates the extensor mechanism which creates forces through the patella that can displace the patella fracture. MRI is rarely indicated in displaced fractures unless the patient is unable to straight leg raise. CT is often ordered if the amount of fracture displacement is unclear.

    Answer B.

    References

    1. Schuett DJ, Hake ME, Mauffrey C, Hammerberg EM, Stahel PF, Hak DJ. Current treatment strategies for patella fractures. Orthopedics. 2015 Jun 1;38(6):377-84.
    2. Hargett, Damayea I. MD; Sanderson, Brent R. DO; Little, Milton T.M. MD Patella Fractures: Approach to Treatment, Journal of the American Academy of Orthopaedic Surgeons: March 15, 2021 – Volume 29 – Issue 6 – p 244-253
    Incorrect

    The patella is the largest sesamoid bone in the body which has the quadriceps tendon attached proximally and the patella ligament attached distally.  The quadriceps, patella, and patella ligament form the extensor mechanism of the knee. Disruption in any of these three structures may cause an inability to straight leg raise or bear weight on the knee. Patella fractures may be treated non-operatively if the extensor mechanism is intact, there is less than 2-3 mm of articular step off, and less than 4mm of fracture displacement. Non-operative acute treatment includes weight bearing as tolerated in a knee immobilizer or hinged knee brace locked in extension.  Weight bearing as tolerated is permitted in a knee immobilizer as the extensor mechanism is not activated with the knee in extension. Any type of flexion, especially with weight bearing, activates the extensor mechanism which creates forces through the patella that can displace the patella fracture. MRI is rarely indicated in displaced fractures unless the patient is unable to straight leg raise. CT is often ordered if the amount of fracture displacement is unclear.

    Answer B.

    References

    1. Schuett DJ, Hake ME, Mauffrey C, Hammerberg EM, Stahel PF, Hak DJ. Current treatment strategies for patella fractures. Orthopedics. 2015 Jun 1;38(6):377-84.
    2. Hargett, Damayea I. MD; Sanderson, Brent R. DO; Little, Milton T.M. MD Patella Fractures: Approach to Treatment, Journal of the American Academy of Orthopaedic Surgeons: March 15, 2021 – Volume 29 – Issue 6 – p 244-253
  4. Question 4 of 64
    4. Question


    A 14-year-old female presents to the office with a 6-month history of left knee pain. She is very active with sports and feels a catching and popping sensation when she runs. She is able to play through the pain. She denies a history of trauma and known precipitating event. She has failed 4 months of conservative treatment including activity modification and physical therapy. On physical exam she has no knee effusion. She has a popping sensation with range of motion with patellofemoral compression. MRI is unremarkable for any soft tissue injury. An intra-operative arthroscopic image is shown in figure 1. A thickened band of tissue is located between the patella and the medial femoral condyle. What is the most likely diagnosis?

    Correct

    Synovial plica is a thin cord-like band of tissue that is part of the synovial lining. The most common type is the suprapatellar plica which arises from the posterior quadriceps tendon and passes to the medial side of the knee joint. Synovial plica is normally found in the knee as a thin and flexible structure. A thick enlarged plica can impinge or become trapped at the patellofemoral joint during knee motion. Medial plica syndrome refers to pain caused from plica impingement during knee motion. Symptoms can also include a sense of catching, locking, or giving way with knee motion. The diagnosis is difficult to make as MRI is often inconclusive. The gold standard for diagnosis and management is knee arthroscopy. The initial treatment of choice should be conservative as medial plica syndrome can often resolve over time. Arthroscopic removal of the symptomatic plica confirms the diagnosis and is successful at resolving pain. 1,2

    Answer A.

     

    References

    1. Al-Hadithy N, Gikas P, Mahapatra AM, Dowd G. Plica syndrome of the knee. Journal of Orthopaedic Surgery. 2011 Dec;19(3):354-8.
    2. Bellary SS, Lynch G, Housman B, Esmaeili E, Gielecki J, Tubbs RS, Loukas M. Medial plica syndrome: a review of the literature. Clinical Anatomy. 2012 May;25(4):423-8.
    Incorrect

    Synovial plica is a thin cord-like band of tissue that is part of the synovial lining. The most common type is the suprapatellar plica which arises from the posterior quadriceps tendon and passes to the medial side of the knee joint. Synovial plica is normally found in the knee as a thin and flexible structure. A thick enlarged plica can impinge or become trapped at the patellofemoral joint during knee motion. Medial plica syndrome refers to pain caused from plica impingement during knee motion. Symptoms can also include a sense of catching, locking, or giving way with knee motion. The diagnosis is difficult to make as MRI is often inconclusive. The gold standard for diagnosis and management is knee arthroscopy. The initial treatment of choice should be conservative as medial plica syndrome can often resolve over time. Arthroscopic removal of the symptomatic plica confirms the diagnosis and is successful at resolving pain. 1,2

    Answer A.

     

    References

    1. Al-Hadithy N, Gikas P, Mahapatra AM, Dowd G. Plica syndrome of the knee. Journal of Orthopaedic Surgery. 2011 Dec;19(3):354-8.
    2. Bellary SS, Lynch G, Housman B, Esmaeili E, Gielecki J, Tubbs RS, Loukas M. Medial plica syndrome: a review of the literature. Clinical Anatomy. 2012 May;25(4):423-8.
  5. Question 5 of 64
    5. Question


    A 14-year-old female presents to your office with complaints of a painful prominence over her right medial midfoot for the past 2 years. She is a hockey player and the pain is made worse while wearing tight skates. She has tried NSAIDs and orthotics which have failed to provide relief. Figure 1 shows an AP x-ray of her right foot with an arrow identifying the painful prominence. Figure 2 is a picture of the bump on the medial side of her foot. Physical exam reveals a firm, non-mobile bump that is tender to palpation. Which of the following is the best treatment option?

    Correct

    The patient is presenting with a symptomatic accessory navicular. An accessory navicular is a normal variant found in up to 14% of the adolescent and pediatric population. The accessory navicular can vary in size ranging from a small round bone within the substance of the tibialis posterior tendon (type 1), a large bony structure connected to the navicular body by a synchondrosis (type 2), or a large ossicle completely fused to the navicular body (type 3). Most patients will remain asymptomatic but some patients, particularly athletes, may complain of pain after an injury or with tight shoe wear. The most common complaint is pain to palpation over the medial prominence of the foot. Most patients can be treated successfully with a period of rest, NSAIDs, pads to cushion the bony prominence, and immobilization if necessary. Indications for surgical excision include persistent pain with shoe wear and pain with athletic activities. Excision generally involves complete resection of the accessory ossicle with re-attachment of the tibialis posterior tendon if necessary. 1,2

    Answer D.

    References

    1. Murphy, Robert F. MD; Van Nortwick, Sara S. MD; Jones, Richard MD; Mooney, James F. III MD Evaluation and Management of Common Accessory Ossicles of the Foot and Ankle in Children and Adolescents, Journal of the American Academy of Orthopaedic Surgeons: January 20, 2021
    2. Grogan DP, Gasser SI, Ogden JA. The painful accessory navicular: a clinical and histopathological study. Foot Ankle. 1989 Dec;10(3):164-9.
    Incorrect

    The patient is presenting with a symptomatic accessory navicular. An accessory navicular is a normal variant found in up to 14% of the adolescent and pediatric population. The accessory navicular can vary in size ranging from a small round bone within the substance of the tibialis posterior tendon (type 1), a large bony structure connected to the navicular body by a synchondrosis (type 2), or a large ossicle completely fused to the navicular body (type 3). Most patients will remain asymptomatic but some patients, particularly athletes, may complain of pain after an injury or with tight shoe wear. The most common complaint is pain to palpation over the medial prominence of the foot. Most patients can be treated successfully with a period of rest, NSAIDs, pads to cushion the bony prominence, and immobilization if necessary. Indications for surgical excision include persistent pain with shoe wear and pain with athletic activities. Excision generally involves complete resection of the accessory ossicle with re-attachment of the tibialis posterior tendon if necessary. 1,2

    Answer D.

    References

    1. Murphy, Robert F. MD; Van Nortwick, Sara S. MD; Jones, Richard MD; Mooney, James F. III MD Evaluation and Management of Common Accessory Ossicles of the Foot and Ankle in Children and Adolescents, Journal of the American Academy of Orthopaedic Surgeons: January 20, 2021
    2. Grogan DP, Gasser SI, Ogden JA. The painful accessory navicular: a clinical and histopathological study. Foot Ankle. 1989 Dec;10(3):164-9.
  6. Question 6 of 64
    6. Question


    A 16 year-old male presents to your office with right knee pain after a trampoline injury 3 days ago. He went to jump and felt a sharp pain in the right knee. He was able to walk after the injury but bending the knee causes severe pain. Over 5 years ago he injured his right knee in a bicycle accident but he had since fully recovered from that injury. On physical exam he has a trace effusion with tenderness to palpation over the tibial tubercle. AP and lateral images taken in the ED (figures 1 and 2) are reported to be positive for calcification in the patella ligament of unknown significance. What is the best treatment option for this patient?

    Correct

    The patient has a minimally displaced tibial tubercle avulsion fracture. Interestingly the calcification in the patella ligament was likely related to his old injury and proved to be a distraction on the radiology report from the acute tibial tubercle avulsion injury.

    Avulsion fractures of the lower extremity in the pediatric population can be easily missed without a careful radiographic evaluation and high clinical suspicion. To the untrained or inexperienced eye, growth plates at varying degrees of skeletally maturity can be confused with fractures and vice versa. The tibial tubercle is an attachment site for the patella ligament and is critical for knee extension. The tibial tubercle apophysis generally fuses to the tibial epiphysis between the ages of 14 and 18 years. Tibial tubercle avulsion fractures typically occur in males as they approach skeletal maturity (ages 12-15). Tibial tubercle avulsion fractures can present with vary degrees of fracture displacement and fracture extension. Fractures can range from a minimally displaced fracture such as this case to a fracture that extends through the entire proximal tibia physis. Fractures with minimal displacement can be treated in a long leg cast or brace with the goal of keeping the knee in extension at all times. A quadriceps contraction could cause displacement of the avulsed fragment. Displaced fractures more than 2mm require surgical fixation. 1,2

    Answer C.

    References

    1. Schiller, Jonathan MD; DeFroda, Steven MD, ME; Blood, Travis MD Lower Extremity Avulsion Fractures in the Pediatric and Adolescent Athlete, Journal of the American Academy of Orthopaedic Surgeons: April 2017 – Volume 25 – Issue 4 – p 251-259
    2. Tibial tubercle fracture. http://www.orthobullets.com. Accessed on 3/1/2021
    Incorrect

    The patient has a minimally displaced tibial tubercle avulsion fracture. Interestingly the calcification in the patella ligament was likely related to his old injury and proved to be a distraction on the radiology report from the acute tibial tubercle avulsion injury.

    Avulsion fractures of the lower extremity in the pediatric population can be easily missed without a careful radiographic evaluation and high clinical suspicion. To the untrained or inexperienced eye, growth plates at varying degrees of skeletally maturity can be confused with fractures and vice versa. The tibial tubercle is an attachment site for the patella ligament and is critical for knee extension. The tibial tubercle apophysis generally fuses to the tibial epiphysis between the ages of 14 and 18 years. Tibial tubercle avulsion fractures typically occur in males as they approach skeletal maturity (ages 12-15). Tibial tubercle avulsion fractures can present with vary degrees of fracture displacement and fracture extension. Fractures can range from a minimally displaced fracture such as this case to a fracture that extends through the entire proximal tibia physis. Fractures with minimal displacement can be treated in a long leg cast or brace with the goal of keeping the knee in extension at all times. A quadriceps contraction could cause displacement of the avulsed fragment. Displaced fractures more than 2mm require surgical fixation. 1,2

    Answer C.

    References

    1. Schiller, Jonathan MD; DeFroda, Steven MD, ME; Blood, Travis MD Lower Extremity Avulsion Fractures in the Pediatric and Adolescent Athlete, Journal of the American Academy of Orthopaedic Surgeons: April 2017 – Volume 25 – Issue 4 – p 251-259
    2. Tibial tubercle fracture. http://www.orthobullets.com. Accessed on 3/1/2021
  7. Question 7 of 64
    7. Question


    A 32-year-old male presents to your office with right elbow pain after a fall 2 days ago. He slipped on ice and fell with an outstretched hand on his right side. He is having difficulty moving the elbow since the injury. On physical exam the patient is able to move the elbow to -20 degrees of extension and 90 degrees of flexion. He has pain to palpation over the radiocapitellar joint. AP and lateral x-rays of the right elbow are shown in figures 1 and 2, respectively. What is the next best treatment option?

    Correct

    Radial head fractures are the most common type of fracture of the elbow. The mechanism of injury is typically a fall on an outstretched hand where the radial head is compressed into the capitellum. Injury severity can range from a simple nondisplaced radial head fracture to a completely displaced, comminuted fracture with associated dislocation or collateral ligament injury. Minimally displaced fractures with less than 2mm of displacement can be treated non-operatively with excellent results. Early motion of the elbow should be initiated as soon as pain tolerates. The elbow can become permanently stiff with prolonged immobilization so early motion is crucial. A sling can be used for the first 1-2 weeks to allow for elbow rest but the patient should remove the sling for gentle range of motion 2-3 times a day. Patients should be instructed to avoid any lifting with the injured arm as any valgus stress placed on the elbow could cause fracture displacement. Follow-up radiographs are generally performed at one week, three weeks, and six weeks after injury to be certain the fracture doesn’t displace. Full range of motion of the elbow without pain is expected at 6 weeks. Patients may resume lifting with the injured arm when full painless range of motion is achieved and radiographs show full healing. 1,2

    Answer B.

     

    References

    1. Tejwani, Nirmal C. MD; Mehta, Hemang MD Fractures of the Radial Head and Neck: Current Concepts in Management, Journal of the American Academy of Orthopaedic Surgeons: July 2007 – Volume 15 – Issue 7 – p 380-387
    2. Rosenblatt Y, Athwal GS, Faber KJ. Current recommendations for the treatment of radial head fractures. Orthopedic Clinics of North America. 2008 Apr 1;39(2):173-85.
    Incorrect

    Radial head fractures are the most common type of fracture of the elbow. The mechanism of injury is typically a fall on an outstretched hand where the radial head is compressed into the capitellum. Injury severity can range from a simple nondisplaced radial head fracture to a completely displaced, comminuted fracture with associated dislocation or collateral ligament injury. Minimally displaced fractures with less than 2mm of displacement can be treated non-operatively with excellent results. Early motion of the elbow should be initiated as soon as pain tolerates. The elbow can become permanently stiff with prolonged immobilization so early motion is crucial. A sling can be used for the first 1-2 weeks to allow for elbow rest but the patient should remove the sling for gentle range of motion 2-3 times a day. Patients should be instructed to avoid any lifting with the injured arm as any valgus stress placed on the elbow could cause fracture displacement. Follow-up radiographs are generally performed at one week, three weeks, and six weeks after injury to be certain the fracture doesn’t displace. Full range of motion of the elbow without pain is expected at 6 weeks. Patients may resume lifting with the injured arm when full painless range of motion is achieved and radiographs show full healing. 1,2

    Answer B.

     

    References

    1. Tejwani, Nirmal C. MD; Mehta, Hemang MD Fractures of the Radial Head and Neck: Current Concepts in Management, Journal of the American Academy of Orthopaedic Surgeons: July 2007 – Volume 15 – Issue 7 – p 380-387
    2. Rosenblatt Y, Athwal GS, Faber KJ. Current recommendations for the treatment of radial head fractures. Orthopedic Clinics of North America. 2008 Apr 1;39(2):173-85.
  8. Question 8 of 64
    8. Question


    A 26-year-old female presents to your office with right hand pain after punching a wall 2 days ago. She had immediate pain and the hand has started to swell over the last few days. She is having difficulty using the hand. On physical exam she has loss of knuckle prominence of the 5th metacarpal head and her skin is intact. AP and lateral x-rays of the hand (figures 1 and 2) show a displaced 5th metacarpal neck fracture with 60 degrees of angulation. Which choice below is a potential complication from this significant fracture angulation?

    Correct

    Fractures of the 5th metacarpal are common injuries seen in the emergency room and orthopedic settings so knowing what fractures need surgery and why is critical. These fractures are often called “boxers fractures” due to the most common mechanism of injury. These fractures often result in shortening and angulation of the 5th metacarpal. Significant shortening of the metacarpal head can create a tendon imbalance resulting in an extension lag of the small finger. Shortening and angulation of the metacarpal neck fracture can also result in loss of knuckle prominence and grip strength weakness. The small finger is able to tolerate more displacement that other metacarpals without functional impairment due to increased metacarpophalangeal joint flexibility of the small finger compared to the others. Non-operative treatment is recommended with up to 40-50 degrees of angulation for the 5th metacarpal whereas 30, 20, and 15 degrees are acceptable for the ring, middle, and index, respectively. 1,2

    Answer D.

    References

    1. Kollitz KM, Hammert WC, Vedder NB, Huang JI. Metacarpal fractures: treatment and complications. Hand (N Y). 2014;9(1):16-23. doi:10.1007/s11552-013-9562-1
    2. Malik S, Herron T, Rosenberg N: Fifth Metacarpal Fractures (Boxer’s Fracture). StatPearls [Internet], Treasure Island (FL); 2020.
    Incorrect

    Fractures of the 5th metacarpal are common injuries seen in the emergency room and orthopedic settings so knowing what fractures need surgery and why is critical. These fractures are often called “boxers fractures” due to the most common mechanism of injury. These fractures often result in shortening and angulation of the 5th metacarpal. Significant shortening of the metacarpal head can create a tendon imbalance resulting in an extension lag of the small finger. Shortening and angulation of the metacarpal neck fracture can also result in loss of knuckle prominence and grip strength weakness. The small finger is able to tolerate more displacement that other metacarpals without functional impairment due to increased metacarpophalangeal joint flexibility of the small finger compared to the others. Non-operative treatment is recommended with up to 40-50 degrees of angulation for the 5th metacarpal whereas 30, 20, and 15 degrees are acceptable for the ring, middle, and index, respectively. 1,2

    Answer D.

    References

    1. Kollitz KM, Hammert WC, Vedder NB, Huang JI. Metacarpal fractures: treatment and complications. Hand (N Y). 2014;9(1):16-23. doi:10.1007/s11552-013-9562-1
    2. Malik S, Herron T, Rosenberg N: Fifth Metacarpal Fractures (Boxer’s Fracture). StatPearls [Internet], Treasure Island (FL); 2020.
  9. Question 9 of 64
    9. Question


    A 78-year-old female presents to the office with 6 months of left shoulder pain. She is two years out from a left proximal humerus fracture which she recovered well from. She denies having a known injury when the pain started 6 months ago and the pain has continually progressed since. AP x-ray of the left shoulder in figure 1 shows advanced avascular necrosis of the proximal humerus. The glenoid appears to be intact without arthritic changes. What is the best treatment option for this patient?

    Correct

    The anterior and posterior circumflex arteries provide most of the blood supply to the humeral head. Fractures of the proximal humerus can disrupt these vessels causing avascular necrosis (AVN) of the humeral head. A displaced proximal humerus fracture may go on to heal and become asymptomatic for a period of time before AVN starts to develop. Symptoms associated with AVN include increasing pain and decreasing shoulder motion. Early radiographic findings may be normal or show signs of humeral head sclerosis and osteopenia. As AVN progresses, the subchondral bone of the humeral head begins to break down and collapse and the glenoid cartilage may degenerate late in the disease.  The average length of time from proximal humerus fracture to the development of symptomatic AVN is usually around 2 years. The risk of AVN may be greater after operative fixation with a proximal humerus plate compared to nonoperative treatment. Conservative treatment of proximal humerus AVN includes rest, NSAIDs, and intra-articular steroid injections. Patients will often do well with conservative treatment and may be asymptomatic. If conservative treatments fail, surgical fixation generally involves a hemiarthroplasty (replace just the humeral surface) or a total shoulder (replace the humeral side and the glenoid).  A hemiarthroplasty is generally chosen when the articular surface of the glenoid is intact and a total shoulder when the glenoid shows signs of degeneration. 1,2

     

    References

    1. Large, Thomas M. MD; Adams, Mark R. MD; Loeffler, Bryan J. MD; Gardner, Michael J. MD Posttraumatic Avascular Necrosis After Proximal Femur, Proximal Humerus, Talar Neck, and Scaphoid Fractures, Journal of the American Academy of Orthopaedic Surgeons: November 1, 2019 – Volume 27 – Issue 21 – p 794-805 doi: 10.5435/JAAOS-D-18-00225
    2. Xu J, Zhang C, Wang T. Avascular necrosis in proximal humeral fractures in patients treated with operative fixation: a meta-analysis. J Orthop Surg Res. 2014;9:31. Published 2014 Apr 27. doi:10.1186/1749-799X-9-31
    Incorrect

    The anterior and posterior circumflex arteries provide most of the blood supply to the humeral head. Fractures of the proximal humerus can disrupt these vessels causing avascular necrosis (AVN) of the humeral head. A displaced proximal humerus fracture may go on to heal and become asymptomatic for a period of time before AVN starts to develop. Symptoms associated with AVN include increasing pain and decreasing shoulder motion. Early radiographic findings may be normal or show signs of humeral head sclerosis and osteopenia. As AVN progresses, the subchondral bone of the humeral head begins to break down and collapse and the glenoid cartilage may degenerate late in the disease.  The average length of time from proximal humerus fracture to the development of symptomatic AVN is usually around 2 years. The risk of AVN may be greater after operative fixation with a proximal humerus plate compared to nonoperative treatment. Conservative treatment of proximal humerus AVN includes rest, NSAIDs, and intra-articular steroid injections. Patients will often do well with conservative treatment and may be asymptomatic. If conservative treatments fail, surgical fixation generally involves a hemiarthroplasty (replace just the humeral surface) or a total shoulder (replace the humeral side and the glenoid).  A hemiarthroplasty is generally chosen when the articular surface of the glenoid is intact and a total shoulder when the glenoid shows signs of degeneration. 1,2

     

    References

    1. Large, Thomas M. MD; Adams, Mark R. MD; Loeffler, Bryan J. MD; Gardner, Michael J. MD Posttraumatic Avascular Necrosis After Proximal Femur, Proximal Humerus, Talar Neck, and Scaphoid Fractures, Journal of the American Academy of Orthopaedic Surgeons: November 1, 2019 – Volume 27 – Issue 21 – p 794-805 doi: 10.5435/JAAOS-D-18-00225
    2. Xu J, Zhang C, Wang T. Avascular necrosis in proximal humeral fractures in patients treated with operative fixation: a meta-analysis. J Orthop Surg Res. 2014;9:31. Published 2014 Apr 27. doi:10.1186/1749-799X-9-31
  10. Question 10 of 64
    10. Question


    A 16-year-old male presents to your office with left knee pain and instability after a ski injury two days ago. He fell while skiing and the knee buckled awkwardly. He was unable to ski down the mountain after the injury. He was seen in the emergency room a few hours later where x-rays showed a tibial spine avulsion fracture (figures 1 and 2). MRI was done the next day showing an avulsion fracture of the tibial spine at the distal attachment site of the anterior cruciate ligament (ACL) with 3mm of displacement (figure 3). What is the best treatment option?

    Correct

    The tibial spine is the distal attachment site of the anterior cruciate ligament (ACL). A tibial spine fracture is uncommon but can occur before skeletal maturity as the ligament can be stronger than the attachment site leading to fracture rather than ligament rupture. Non-displaced fractures can be treated in a knee immobilizer with slight flexion (less tension on ACL) until the fracture heals. If there is any question of fracture displacement, CT can provide a precise measurement. MRI helps delineate the fracture and identify other soft tissue injury. Minimally displaced fractures may reduce with the knee placed in extension. Displaced and non-displaced fractures that can’t be reduced require surgical fixation, most commonly with an arthroscopic approach. The most common arthroscopic technique involves using suture to pull the fracture back down for an anatomic reduction. Post-operative weight bearing and range of motion is surgeon dependent, although early motion helps reduce the risk of arthrofibrosis. The expected return to sports is 3-4 months. 1,2

    Answer C.

    References

    1. Herman, Martin J. MD; Martinek, Melissa A. DO, PhD; Abzug, Joshua M. MD Complications of Tibial Eminence and Diaphyseal Fractures in Children, Journal of the American Academy of Orthopaedic Surgeons: November 2014 – Volume 22 – Issue 11 – p 730-741
    2. Osti L, Buda M, Soldati F, Del Buono A, Osti R, Maffulli N. Arthroscopic treatment of tibial eminence fracture: a systematic review of different fixation methods. Br Med Bull. 2016;118(1):73-90.
    Incorrect

    The tibial spine is the distal attachment site of the anterior cruciate ligament (ACL). A tibial spine fracture is uncommon but can occur before skeletal maturity as the ligament can be stronger than the attachment site leading to fracture rather than ligament rupture. Non-displaced fractures can be treated in a knee immobilizer with slight flexion (less tension on ACL) until the fracture heals. If there is any question of fracture displacement, CT can provide a precise measurement. MRI helps delineate the fracture and identify other soft tissue injury. Minimally displaced fractures may reduce with the knee placed in extension. Displaced and non-displaced fractures that can’t be reduced require surgical fixation, most commonly with an arthroscopic approach. The most common arthroscopic technique involves using suture to pull the fracture back down for an anatomic reduction. Post-operative weight bearing and range of motion is surgeon dependent, although early motion helps reduce the risk of arthrofibrosis. The expected return to sports is 3-4 months. 1,2

    Answer C.

    References

    1. Herman, Martin J. MD; Martinek, Melissa A. DO, PhD; Abzug, Joshua M. MD Complications of Tibial Eminence and Diaphyseal Fractures in Children, Journal of the American Academy of Orthopaedic Surgeons: November 2014 – Volume 22 – Issue 11 – p 730-741
    2. Osti L, Buda M, Soldati F, Del Buono A, Osti R, Maffulli N. Arthroscopic treatment of tibial eminence fracture: a systematic review of different fixation methods. Br Med Bull. 2016;118(1):73-90.
  11. Question 11 of 64
    11. Question


    An 82-year-old female presents with 4 months of left knee pain. She denies a known injury or precipitating event. She has tried intra-articular steroid and hyaluronic acid injections over the last few months with only minimal relief. On physical exam she has pain over the medial joint line that is made worse with weight bearing and varus stress to the knee. Figure 1 shows an AP x-ray of the left knee. Coronal MRI image (figure 2) shows bone marrow edema over the medial femoral condyle and tibial plateau. Which statement is true regarding bone marrow edema associated with osteoarthritis?

    Correct

    Bone marrow edema is a common MRI finding in patients with advanced osteoarthritis. Bone marrow edema is more common in patients with knee pain compared to asymptomatic arthritic knees. Edema is also associated with an acceleration of cartilage loss over time. Patients who present with arthritis associated bone marrow edema are 8 times more likely to go on to knee replacement compared to those with no edema. The loss of articular cartilage increases subchondral bone stress leading to arthritic progression.  As the arthritic cascade progresses the overloading of subchondral bone causes microdamage that outpaces repair of the bone. The differential diagnosis for bone marrow edema includes osteonecrosis, bone marrow edema syndrome, osteochondritis dissecans, bone contusion, stress fracture, and tumors. Bone marrow edema caused by arthritis is often confused with a stress fracture on MRI. Both have similar histopathology with findings of bone marrow necrosis, fibrosis, and trabeculae abnormalities. Bone marrow edema associated with arthritis is generally found at the knee compartment with the most arthritis, which is usually the medial compartment. Treatment of bone marrow edema involves resting the joint so bone repair can catch up to subchondral damage of weight bearing. An unloader brace has not been shown to improve symptoms or reduce bone marrow edema. Intravenous zoledronic acid has been shown to help reduce bone marrow edema and improve symptoms by reducing resorption of the subchondral bone. A subchondroplasty procedure, which includes an injection of a polymerizable calcium phosphate bone substitute in the subchondral bone at the location of the bone marrow edema, has recently been used to help restore healing and strengthen subchondral bone in these patients. 1,2

    Answer A.

    References

    1. Hofmann, Siegfried, et al. “Painful bone marrow edema of the knee: differential diagnosis and therapeutic concepts.” Orthopedic Clinics 35.3 (2004): 321-333.
    2. Sharkey PF, Cohen SB, Leinberry CF, Parvizi J. Subchondral bone marrow lesions associated with knee osteoarthritis. Am J Orthop. 2012 Sep 1;41(9):413-7.

     

    Incorrect

    Bone marrow edema is a common MRI finding in patients with advanced osteoarthritis. Bone marrow edema is more common in patients with knee pain compared to asymptomatic arthritic knees. Edema is also associated with an acceleration of cartilage loss over time. Patients who present with arthritis associated bone marrow edema are 8 times more likely to go on to knee replacement compared to those with no edema. The loss of articular cartilage increases subchondral bone stress leading to arthritic progression.  As the arthritic cascade progresses the overloading of subchondral bone causes microdamage that outpaces repair of the bone. The differential diagnosis for bone marrow edema includes osteonecrosis, bone marrow edema syndrome, osteochondritis dissecans, bone contusion, stress fracture, and tumors. Bone marrow edema caused by arthritis is often confused with a stress fracture on MRI. Both have similar histopathology with findings of bone marrow necrosis, fibrosis, and trabeculae abnormalities. Bone marrow edema associated with arthritis is generally found at the knee compartment with the most arthritis, which is usually the medial compartment. Treatment of bone marrow edema involves resting the joint so bone repair can catch up to subchondral damage of weight bearing. An unloader brace has not been shown to improve symptoms or reduce bone marrow edema. Intravenous zoledronic acid has been shown to help reduce bone marrow edema and improve symptoms by reducing resorption of the subchondral bone. A subchondroplasty procedure, which includes an injection of a polymerizable calcium phosphate bone substitute in the subchondral bone at the location of the bone marrow edema, has recently been used to help restore healing and strengthen subchondral bone in these patients. 1,2

    Answer A.

    References

    1. Hofmann, Siegfried, et al. “Painful bone marrow edema of the knee: differential diagnosis and therapeutic concepts.” Orthopedic Clinics 35.3 (2004): 321-333.
    2. Sharkey PF, Cohen SB, Leinberry CF, Parvizi J. Subchondral bone marrow lesions associated with knee osteoarthritis. Am J Orthop. 2012 Sep 1;41(9):413-7.

     

  12. Question 12 of 64
    12. Question


    A 78-year-old male presents to the emergency room with severe right knee pain after a fall from a ladder. He fell from the top of a 6-foot ladder and landed on his right leg. He is unable to bear weight since in the injury. AP and lateral x-rays of the right tibia and fibula are shown in figures 1 and 2, respectively. X-rays show a complex comminuted proximal third tibia fracture with mild displacement. On physical exam his skin is intact with mild swelling of the knee. What is the best treatment option for this patient?

    Correct

    Extra-articular proximal one third tibia fractures are uncommon injuries that present unique challenges to treat. The patella tendon tends to pull the proximal fragment in extension while the gastrocnemius pulls the proximal fragment in flexion. Minimally displaced fractures can be treated non-operatively with long leg immobilization. Fractures that are considered for non-operative treatment include those with <5 degrees of varus-valgus angulation, < 10 degrees of anterior/posterior angulation, > 50% cortical opposition, < 1 cm shortening, and < 10 degrees of rotational alignment. Patients with poor healing capacity such as elderly osteoporotic patients, diabetics, and smokers may not be candidates for surgery as well. Non-operative treatment usually includes 4 weeks of a long leg cast with the knee in 10-20 degrees of flexion. Mild anterior/posterior angulation can be correct in a long leg cast with proper molding. Surgical techniques may include plate osteosynthesis, intra-medullary nailing, and external fixation. Of the three, intra-medullary nailing is the preferred surgical technique. However, nailing of proximal third fractures has a higher risk of malalignment then distal fractures treated with nailing. Patients that present with significant soft tissue swelling and fracture displacement are usually put in an external fixator until swelling subsides. 1,2

    Answer B

    References

    1. Wu D, Reng G, Shrivastava A, Yu Y, Zhang Y, Peng C. A useful surgical strategy for proximal tibial fractures (AO/OTA type 41-C) with diaphyseal involvement. Int J Clin Exp Med. 2015;8(8):13455-13463. Published 2015 Aug 15.
    2. Hiesterman TG, Shafiq BX, Cole PA. Intramedullary Nailing of Extraarticular Proximal Tibia Fractures. J Am Acad Orthop Surg 2011;19: 690-700
    Incorrect

    Extra-articular proximal one third tibia fractures are uncommon injuries that present unique challenges to treat. The patella tendon tends to pull the proximal fragment in extension while the gastrocnemius pulls the proximal fragment in flexion. Minimally displaced fractures can be treated non-operatively with long leg immobilization. Fractures that are considered for non-operative treatment include those with <5 degrees of varus-valgus angulation, < 10 degrees of anterior/posterior angulation, > 50% cortical opposition, < 1 cm shortening, and < 10 degrees of rotational alignment. Patients with poor healing capacity such as elderly osteoporotic patients, diabetics, and smokers may not be candidates for surgery as well. Non-operative treatment usually includes 4 weeks of a long leg cast with the knee in 10-20 degrees of flexion. Mild anterior/posterior angulation can be correct in a long leg cast with proper molding. Surgical techniques may include plate osteosynthesis, intra-medullary nailing, and external fixation. Of the three, intra-medullary nailing is the preferred surgical technique. However, nailing of proximal third fractures has a higher risk of malalignment then distal fractures treated with nailing. Patients that present with significant soft tissue swelling and fracture displacement are usually put in an external fixator until swelling subsides. 1,2

    Answer B

    References

    1. Wu D, Reng G, Shrivastava A, Yu Y, Zhang Y, Peng C. A useful surgical strategy for proximal tibial fractures (AO/OTA type 41-C) with diaphyseal involvement. Int J Clin Exp Med. 2015;8(8):13455-13463. Published 2015 Aug 15.
    2. Hiesterman TG, Shafiq BX, Cole PA. Intramedullary Nailing of Extraarticular Proximal Tibia Fractures. J Am Acad Orthop Surg 2011;19: 690-700
  13. Question 13 of 64
    13. Question


    A 21-year-old male hockey player presents to your clinic with complaints of left groin pain for 6 months. He has been able to play through the groin discomfort until he recently aggravated the injury during a game. He was hit from behind and his leg did a split when he felt a “pop” in his left groin. He is unable to push off with skating since the injury. Walking produces only mild pain but running and skating causes the pain to significantly increase. Coronal MRI image of the pelvis (figure 1) done one week ago shows tears of the left pectineus, adductor longus and adductor brevis off the pubic symphysis with moderate fluid around the tears. What is the best treatment option?

    Correct

    The hip and groin region is a problematic area for injury in high level hockey players. The adductor muscles, including the adductor longus, adductor magnus, and adductor brevis muscles, as well as the gracilis, obturator externus, and pectineus muscles, are particular susceptible to injury when skating at high speeds. Adductor strains/tears occur during eccentric contraction (forces applied to the muscle in a lengthened position). Injury usually occurs at the myotendinous junction of the adductors at the ischium. Physical exam features of an adductor injury include groin pain with resisted hip adduction and passive stretching of the adductors. Treatment of adductor injuries is nonoperative with a period of rest and physical therapy. As the adductor strain heals physical therapy is important to maintain flexibility and hip motion. Adductor strains have a high incidence of reoccurrence with emphasizes the importance of continued rehabilitation with return to play. Athletes with weak adductors (abductor strength > adductor strength) are more prone to adductor injuries. Steroid and PRP injections may be used in patients with chronic adductor pain without structural damage on MRI. Selective adductor surgical release has also been used as a successful treatment in patients with chronic symptoms. 1,2

    Answer B.

    References

    1. Lynch, T. Sean MD; Bedi, Asheesh MD; Larson, Christopher M. MD Athletic Hip Injuries, Journal of the American Academy of Orthopaedic Surgeons: April 2017 – Volume 25 – Issue 4 – p 269-279
    2. Tyler TF, Nicholas SJ, Campbell RJ, McHugh MP. The association of hip strength and flexibility with the incidence of adductor muscle strains in professional ice hockey players. The American journal of sports medicine. 2001 Mar;29(2):124-8.
    Incorrect

    The hip and groin region is a problematic area for injury in high level hockey players. The adductor muscles, including the adductor longus, adductor magnus, and adductor brevis muscles, as well as the gracilis, obturator externus, and pectineus muscles, are particular susceptible to injury when skating at high speeds. Adductor strains/tears occur during eccentric contraction (forces applied to the muscle in a lengthened position). Injury usually occurs at the myotendinous junction of the adductors at the ischium. Physical exam features of an adductor injury include groin pain with resisted hip adduction and passive stretching of the adductors. Treatment of adductor injuries is nonoperative with a period of rest and physical therapy. As the adductor strain heals physical therapy is important to maintain flexibility and hip motion. Adductor strains have a high incidence of reoccurrence with emphasizes the importance of continued rehabilitation with return to play. Athletes with weak adductors (abductor strength > adductor strength) are more prone to adductor injuries. Steroid and PRP injections may be used in patients with chronic adductor pain without structural damage on MRI. Selective adductor surgical release has also been used as a successful treatment in patients with chronic symptoms. 1,2

    Answer B.

    References

    1. Lynch, T. Sean MD; Bedi, Asheesh MD; Larson, Christopher M. MD Athletic Hip Injuries, Journal of the American Academy of Orthopaedic Surgeons: April 2017 – Volume 25 – Issue 4 – p 269-279
    2. Tyler TF, Nicholas SJ, Campbell RJ, McHugh MP. The association of hip strength and flexibility with the incidence of adductor muscle strains in professional ice hockey players. The American journal of sports medicine. 2001 Mar;29(2):124-8.
  14. Question 14 of 64
    14. Question


    A 19-year-old girl presents to the office with right calf pain after a lacrosse injury two days ago. She was making a sharp cutting maneuver in practice and felt and pop in the back of her heal. She was unable to continue playing and her ankle quickly became swollen over the next day. On physical exam she has tenderness to palpation over the myotendinous junction of the Achilles and a positive Thompson’s test. Figure 1 is sagittal MRI imaging showing a proximal Achilles tendon rupture. What is the best treatment option?

    Correct

    Achilles ruptures are common injuries that typically occur in middle aged men during sporting activities. Achilles ruptures typically (75%) occur in the mid-substance of the tendon or within 6 cm of where the tendon attaches to the calcaneus. Ruptures in this location are typically treated surgically in the active population as the re-rupture rate is higher with non-operative treatment. Ruptures of the Achilles tendon at the myotendinous junction are less common (<15% of all Achilles ruptures). Ahmad et al treated 30 patients with acute myotendinous junction injuries over a 6-year period with conservative treatment consisting of 3 weeks of non-weight bearing followed by 3 weeks of progressive weight bearing in a boot. At 6 weeks, physical therapy was initiated for range of motion exercises. The study concluded that nonsurgical treatment of myotendinous Achilles tendon ruptures resulted in a high rate of healing and return to function. Of the 30 patients followed in the study, not one had a re-rupture after 3 years. 1,2

    Answer B.

    References

    1. Ahmad J, Repka M, Raikin SM. Treatment of myotendinous Achilles ruptures. Foot & ankle international. 2013 Aug;34(8):1074-8.
    2. Kadakia, Anish R. MD; Dekker, Robert G. II MD; Ho, Bryant S. MD Acute Achilles Tendon Ruptures: An Update on Treatment, Journal of the American Academy of Orthopaedic Surgeons: January 2017 – Volume 25 – Issue 1 – p 23-31

     

     

    Incorrect

    Achilles ruptures are common injuries that typically occur in middle aged men during sporting activities. Achilles ruptures typically (75%) occur in the mid-substance of the tendon or within 6 cm of where the tendon attaches to the calcaneus. Ruptures in this location are typically treated surgically in the active population as the re-rupture rate is higher with non-operative treatment. Ruptures of the Achilles tendon at the myotendinous junction are less common (<15% of all Achilles ruptures). Ahmad et al treated 30 patients with acute myotendinous junction injuries over a 6-year period with conservative treatment consisting of 3 weeks of non-weight bearing followed by 3 weeks of progressive weight bearing in a boot. At 6 weeks, physical therapy was initiated for range of motion exercises. The study concluded that nonsurgical treatment of myotendinous Achilles tendon ruptures resulted in a high rate of healing and return to function. Of the 30 patients followed in the study, not one had a re-rupture after 3 years. 1,2

    Answer B.

    References

    1. Ahmad J, Repka M, Raikin SM. Treatment of myotendinous Achilles ruptures. Foot & ankle international. 2013 Aug;34(8):1074-8.
    2. Kadakia, Anish R. MD; Dekker, Robert G. II MD; Ho, Bryant S. MD Acute Achilles Tendon Ruptures: An Update on Treatment, Journal of the American Academy of Orthopaedic Surgeons: January 2017 – Volume 25 – Issue 1 – p 23-31

     

     

  15. Question 15 of 64
    15. Question


    A 12-year-old male presents to your clinic with right heal pain for 6 weeks. He noticed the pain when he started soccer practice and the pain has become progressively worse since. He is now having trouble running due to the right heal pain. On physical exam he has no obvious deformity to the foot or ankle. He has tenderness to palpation over the posterior calcaneus and mild pain with resisted plantar flexion. Lateral x-ray of the right foot is shown in figure 1. What is the most likely diagnosis?

    Correct

    Sever’s disease, or calcaneal apophysitis, is a common condition seen in pediatric and orthopedic clinics. The typical presentation includes children between the ages of 10-12 who present with heel pain after starting an athletic activity. These patients often start the sport activity with minimal to no pain but soon after participation will limp off the field or court. The etiology is unknown although the widely accepted theory is that the condition occurs during a growth spurt when the long bones grow faster in length than the tendons. Subsequently the Achilles tendon is tight creating a traction apophysitis to the calcaneal apophysis. As the foot grows in length the plantar fascia can increase in tension at its insertion site on the calcaneus, which can contribute to the pain. The diagnosis of Sever’s disease is made clinically by pain with palpation over the posterior heel (calcaneal apophysis) and open growth plates seen on radiographs. Radiographs are typically normal but may show fragmentation at the apophysis. Treatment involves activity modification, NSAIDS, and if symptoms warrant, a period of immobilization. Padded heal cups, orthotics with arch support, and heal lifts have been used without definitive evidence showing significant efficacy. 1,2

    Answer D

    References

    1. Scharfbillig RW, Jones S, Scutter SD. Sever’s disease: what does the literature really tell us?. Journal of the American Podiatric Medical Association. 2008 May;98(3):212-23.
    2. James AM, Williams CM, Haines TP. Effectiveness of interventions in reducing pain and maintaining physical activity in children and adolescents with calcaneal apophysitis (Sever’s disease): a systematic review. Journal of foot and ankle research. 2013 Dec;6(1):1-1.
    Incorrect

    Sever’s disease, or calcaneal apophysitis, is a common condition seen in pediatric and orthopedic clinics. The typical presentation includes children between the ages of 10-12 who present with heel pain after starting an athletic activity. These patients often start the sport activity with minimal to no pain but soon after participation will limp off the field or court. The etiology is unknown although the widely accepted theory is that the condition occurs during a growth spurt when the long bones grow faster in length than the tendons. Subsequently the Achilles tendon is tight creating a traction apophysitis to the calcaneal apophysis. As the foot grows in length the plantar fascia can increase in tension at its insertion site on the calcaneus, which can contribute to the pain. The diagnosis of Sever’s disease is made clinically by pain with palpation over the posterior heel (calcaneal apophysis) and open growth plates seen on radiographs. Radiographs are typically normal but may show fragmentation at the apophysis. Treatment involves activity modification, NSAIDS, and if symptoms warrant, a period of immobilization. Padded heal cups, orthotics with arch support, and heal lifts have been used without definitive evidence showing significant efficacy. 1,2

    Answer D

    References

    1. Scharfbillig RW, Jones S, Scutter SD. Sever’s disease: what does the literature really tell us?. Journal of the American Podiatric Medical Association. 2008 May;98(3):212-23.
    2. James AM, Williams CM, Haines TP. Effectiveness of interventions in reducing pain and maintaining physical activity in children and adolescents with calcaneal apophysitis (Sever’s disease): a systematic review. Journal of foot and ankle research. 2013 Dec;6(1):1-1.
  16. Question 16 of 64
    16. Question


    A 38-year-old female presents to the office with a 2-month history of left knee pain with bending and squatting. She denies a known injury or precipitating event. She describes occasional catching and giving way while walking. On physical exam she has pain to palpation over the anterior medial knee and no effusion. Her knee pain is made worse with deep knee flexion. AP radiograph (figures 1) show a cyst on the medial femoral condyle and no arthritis changes. MRI of the left knee (figure 2) shows a osteochondral lesion with cyst formation at the medial femoral notch measuring 2 by 2.5 cm. This is a full thickness osteochondral defect with concavity of the articular surface. What is the best treatment option for this patient?

    Correct

    The most common location for an osteochondral (OCD) lesion is in the medial femoral condyle, which accounts for 75% of all lesions. The severity of an OCD lesion ranges from early softening of the articular cartilage, early cartilage separation, and detachment and loose body formation of the osteochondral lesion. MRI is the diagnostic modality of choice for identifying OCD lesions and determining the most appropriate treatment. Most adult forms of OCD stem from asymptomatic or untreated juvenile OCD. Adult OCD lesions often require surgical intervention as the articular cartilage has a poor ability to heal itself. For small non-repairable lesions, an arthroscopic debridement with loose body removal and/or microfracture is generally recommended. For larger unstable lesions, treatment generally includes fragment removal and ostechondral allograft, autograft, or autologous chondrocyte implantation. For smaller lesions <2 cm2 , an osteochondral autograft can be used, but for larger lesions with bone loss, an osteochondral allograft is recommended. Autologous chondrocyte implementation is not recommended when extensive bone loss that requires restoration. 1,2

    Answer C.

    References

    1. Friel NA, Bajaj S, Cole BJ. Articular cartilage injury and adult OCD: treatment options and decision making. Scott WN. Insall & Scott Surgery of the Knee. 2012;5:153-62.
    2. Sherman, Seth L. MD; Garrity, Joseph MS; Bauer, Kathryn; Cook, James DVM, PhD; Stannard, James MD; Bugbee, William MD Fresh Osteochondral Allograft Transplantation for the Knee: Current Concepts, Journal of the American Academy of Orthopaedic Surgeons: February 2014 – Volume 22 – Issue 2 – p 121-133
    Incorrect

    The most common location for an osteochondral (OCD) lesion is in the medial femoral condyle, which accounts for 75% of all lesions. The severity of an OCD lesion ranges from early softening of the articular cartilage, early cartilage separation, and detachment and loose body formation of the osteochondral lesion. MRI is the diagnostic modality of choice for identifying OCD lesions and determining the most appropriate treatment. Most adult forms of OCD stem from asymptomatic or untreated juvenile OCD. Adult OCD lesions often require surgical intervention as the articular cartilage has a poor ability to heal itself. For small non-repairable lesions, an arthroscopic debridement with loose body removal and/or microfracture is generally recommended. For larger unstable lesions, treatment generally includes fragment removal and ostechondral allograft, autograft, or autologous chondrocyte implantation. For smaller lesions <2 cm2 , an osteochondral autograft can be used, but for larger lesions with bone loss, an osteochondral allograft is recommended. Autologous chondrocyte implementation is not recommended when extensive bone loss that requires restoration. 1,2

    Answer C.

    References

    1. Friel NA, Bajaj S, Cole BJ. Articular cartilage injury and adult OCD: treatment options and decision making. Scott WN. Insall & Scott Surgery of the Knee. 2012;5:153-62.
    2. Sherman, Seth L. MD; Garrity, Joseph MS; Bauer, Kathryn; Cook, James DVM, PhD; Stannard, James MD; Bugbee, William MD Fresh Osteochondral Allograft Transplantation for the Knee: Current Concepts, Journal of the American Academy of Orthopaedic Surgeons: February 2014 – Volume 22 – Issue 2 – p 121-133
  17. Question 17 of 64
    17. Question


    A 44-year-old female presents to the office with right hand and thumb pain after falling on a dock while getting off a boat 4 days ago. She jammed her thumb against the wooden dock and had immediate pain and swelling. On physical exam, there is no discernable endpoint with radial deviation at the metacarpophalangeal (MCP) joint. AP and lateral view x-rays of the right hand are shown in figures 1 and 2. What is the next best step in treatment?

    Correct

    The patient has a complete tear of the ulnar collateral ligament (UCL) of the thumb as noted by no discernable endpoint with radial stress to the MCP joint. An intact UCL is critical for hand function including grip strength. Tenderness to palpation over the MCP joint and instability with valgus stress (radial deviation) are key findings with a UCL injury. Radiographs are routinely ordered to assess for fractures and joint instability. MRI may be used if the diagnosis is in question or the degree of injury is unclear. MRI is not routinely ordered when patients have an obvious injury to the UCL.  Complete tears of the UCL should be treated surgically with repair of the UCL ligament. Almost 90% of complete UCL ruptures have a Stener lesion which will not allow healing without surgical repair. Small bony avulsion fractures of the UCL can be treated with a small screw or pin. If the avulsed fragment is too small for fixation the piece of bone can be removed and then ligament repaired down to bone. 1,2

     

    Answer D.

    References

    1. Tang, Peter MD, MPH Collateral Ligament Injuries of the Thumb Metacarpophalangeal Joint, American Academy of Orthopaedic Surgeon: May 2011 – Volume 19 – Issue 5 – p 287-296
    2. Pulos N, Alexander Y. Treatment of Ulnar Collateral Ligament Injuries of the Thumb. A Critical Analysis Review. JBJS Reviews – Review Article with Critical Analysis Component: 21 February 2017 – Volume 5 – Issue 2 – p. e3
    Incorrect

    The patient has a complete tear of the ulnar collateral ligament (UCL) of the thumb as noted by no discernable endpoint with radial stress to the MCP joint. An intact UCL is critical for hand function including grip strength. Tenderness to palpation over the MCP joint and instability with valgus stress (radial deviation) are key findings with a UCL injury. Radiographs are routinely ordered to assess for fractures and joint instability. MRI may be used if the diagnosis is in question or the degree of injury is unclear. MRI is not routinely ordered when patients have an obvious injury to the UCL.  Complete tears of the UCL should be treated surgically with repair of the UCL ligament. Almost 90% of complete UCL ruptures have a Stener lesion which will not allow healing without surgical repair. Small bony avulsion fractures of the UCL can be treated with a small screw or pin. If the avulsed fragment is too small for fixation the piece of bone can be removed and then ligament repaired down to bone. 1,2

     

    Answer D.

    References

    1. Tang, Peter MD, MPH Collateral Ligament Injuries of the Thumb Metacarpophalangeal Joint, American Academy of Orthopaedic Surgeon: May 2011 – Volume 19 – Issue 5 – p 287-296
    2. Pulos N, Alexander Y. Treatment of Ulnar Collateral Ligament Injuries of the Thumb. A Critical Analysis Review. JBJS Reviews – Review Article with Critical Analysis Component: 21 February 2017 – Volume 5 – Issue 2 – p. e3
  18. Question 18 of 64
    18. Question


    An 18-year-old male presents to your clinic with chronic lower back pain for over a year with new onset of pain and weakness down the right leg for the last few months. He is currently playing lacrosse for his high school and is right knee seems to be giving out on him. He denies any left sided leg symptoms, any numbness or tingling, and any bowel or bladder issues. On physical exam he has 4 out of 5 weakness with quadriceps and anterior tibialis testing on right, 5/5 on left. AP and lateral radiographs (figures 1 and 2) of the lumbar spine show an anterior listhesis at L4-5, hemi sacralized left L5. MRI shows a grade 1 chronic spondylolytic spondylolisthesis at L4-L5 with no bone marrow edema. He has mild foraminal narrowing on the left at L4-L5. What is the best treatment option for this patient?

    Correct

    Spondylolysis with low grade spondylolisthesis generally responds well to conservative treatment including rest, activity modification, corset brace to limit extremes of spine motion, and then physical therapy for core strengthening. Patients/athletes can return work and sports when the back is pain free, motion is full, and hamstring spasm has subsided. Return to sports generally occurs after 6-12 weeks depending on the severity of the injury. Surgical treatment is indicated in patients who have failed conservative treatment for 6-12 months and have persistent pain or neurological deficits. Treatment involves a spinal fusion at the effected level. In situ spinal fusion indirectly decompresses the nerve room and eliminates spinal motion contributing to nerve root irritation. Nerve root decompression with a laminectomy is not generally performed with fusions for spondylolysis. The patient was educated on his condition and advised to avoid any sports that involve contact, bending, twisting, or lifting. Due to his lower extremity weakness and nerve impingement on MRI, an in situ fusion was recommended. He was placed in a corset brace and physical therapy was prescribed until surgery could be performed in a non-urgent matter. 1,2

    Answer D.

    References

    1. Cavalier, Ralph MD; Herman, Martin J. MD; Cheung, Emilie V. MD; Pizzutillo, Peter D. MD Spondylolysis and Spondylolisthesis in Children and Adolescents: I. Diagnosis, Natural History, and Nonsurgical Management, Journal of the American Academy of Orthopaedic Surgeons: July 2006 – Volume 14 – Issue 7 – p 417-424
    2. Cheung, Emilie V. MD; Herman, Martin J. MD; Cavalier, Ralph MD; Pizzutillo, Peter D. MD Spondylolysis and Spondylolisthesis in Children and Adolescents: II. Surgical Management, Journal of the American Academy of Orthopaedic Surgeons: August 2006 – Volume 14 – Issue 8 – p 488-498

     

    Incorrect

    Spondylolysis with low grade spondylolisthesis generally responds well to conservative treatment including rest, activity modification, corset brace to limit extremes of spine motion, and then physical therapy for core strengthening. Patients/athletes can return work and sports when the back is pain free, motion is full, and hamstring spasm has subsided. Return to sports generally occurs after 6-12 weeks depending on the severity of the injury. Surgical treatment is indicated in patients who have failed conservative treatment for 6-12 months and have persistent pain or neurological deficits. Treatment involves a spinal fusion at the effected level. In situ spinal fusion indirectly decompresses the nerve room and eliminates spinal motion contributing to nerve root irritation. Nerve root decompression with a laminectomy is not generally performed with fusions for spondylolysis. The patient was educated on his condition and advised to avoid any sports that involve contact, bending, twisting, or lifting. Due to his lower extremity weakness and nerve impingement on MRI, an in situ fusion was recommended. He was placed in a corset brace and physical therapy was prescribed until surgery could be performed in a non-urgent matter. 1,2

    Answer D.

    References

    1. Cavalier, Ralph MD; Herman, Martin J. MD; Cheung, Emilie V. MD; Pizzutillo, Peter D. MD Spondylolysis and Spondylolisthesis in Children and Adolescents: I. Diagnosis, Natural History, and Nonsurgical Management, Journal of the American Academy of Orthopaedic Surgeons: July 2006 – Volume 14 – Issue 7 – p 417-424
    2. Cheung, Emilie V. MD; Herman, Martin J. MD; Cavalier, Ralph MD; Pizzutillo, Peter D. MD Spondylolysis and Spondylolisthesis in Children and Adolescents: II. Surgical Management, Journal of the American Academy of Orthopaedic Surgeons: August 2006 – Volume 14 – Issue 8 – p 488-498

     

  19. Question 19 of 64
    19. Question


    A 12-year-old female presents to the emergency room with severe left ankle pain after twisting her ankle and falling while running earlier in the day. On physical exam of the left ankle her skin is intact with mild swelling over the medial and lateral aspects of the ankle. Ankle range of motion is limited due to pain and swelling. AP and lateral radiographs are shown in figures 1 and 2. What is the next best step in treatment?

    Correct

    A triplane fracture is a unique pattern that occurs at the distal tibial epiphysis during the period of physeal closure or around 12-15 years of age. Triplane refers to the pattern of fracture in the sagittal, transverse, and coronal planes. Specifically, the pattern includes a vertical fracture through the epiphysis, a horizontal fracture through the physis, and an oblique fracture through the metaphysis. The presence of a metaphyseal fractures distinguishes a triplane fracture from a Tillaux fracture. Triplane fractures most commonly occur from a twisting injury during athletic participation. The fracture can be in multiple fragments which is difficult to visualize on plain radiographs. Like most displaced intra-articular distal tibia fractures, CT is necessary to better visualize the fracture pattern and determine fracture displacement. Indications for surgery for triplane fractures include >2mm of intra-articular displacement. Without an anatomical reduction patients are at risk for premature physeal closure, ankle deformity, and osteoarthritis. 1,2

    Answer A.

    References

    1. Schnetzler, Kent A. MD, MS; Hoernschemeyer, Daniel MD The Pediatric Triplane Ankle Fracture, Journal of the American Academy of Orthopaedic Surgeons: December 2007 – Volume 15 – Issue 12 – p 738-747
    2. Liporace FA, Yoon RS, Kubiak EN, Parisi DM, Koval KJ, Feldman DS, Egol KA. Does adding computed tomography change the diagnosis and treatment of Tillaux and triplane pediatric ankle fractures?. Orthopedics. 2012 Feb 17;35(2):e208-12.
    Incorrect

    A triplane fracture is a unique pattern that occurs at the distal tibial epiphysis during the period of physeal closure or around 12-15 years of age. Triplane refers to the pattern of fracture in the sagittal, transverse, and coronal planes. Specifically, the pattern includes a vertical fracture through the epiphysis, a horizontal fracture through the physis, and an oblique fracture through the metaphysis. The presence of a metaphyseal fractures distinguishes a triplane fracture from a Tillaux fracture. Triplane fractures most commonly occur from a twisting injury during athletic participation. The fracture can be in multiple fragments which is difficult to visualize on plain radiographs. Like most displaced intra-articular distal tibia fractures, CT is necessary to better visualize the fracture pattern and determine fracture displacement. Indications for surgery for triplane fractures include >2mm of intra-articular displacement. Without an anatomical reduction patients are at risk for premature physeal closure, ankle deformity, and osteoarthritis. 1,2

    Answer A.

    References

    1. Schnetzler, Kent A. MD, MS; Hoernschemeyer, Daniel MD The Pediatric Triplane Ankle Fracture, Journal of the American Academy of Orthopaedic Surgeons: December 2007 – Volume 15 – Issue 12 – p 738-747
    2. Liporace FA, Yoon RS, Kubiak EN, Parisi DM, Koval KJ, Feldman DS, Egol KA. Does adding computed tomography change the diagnosis and treatment of Tillaux and triplane pediatric ankle fractures?. Orthopedics. 2012 Feb 17;35(2):e208-12.
  20. Question 20 of 64
    20. Question


    A 76-year-old male presents to your office with progressive hip “tightness” over the last 4 months. He denies significant pain in the hip and his gait remains unchanged. He notices the hip is a little tighter when getting out of the car and shifting in bed. He is one year out from a right total hip replacement. On physical exam of the right hip he has internal rotation measuring 15 degrees, abduction 20 degrees, and 95 degrees of hip flexion. Figure 1 shows an AP pelvis x-ray with heterotopic ossification noted in the right hip. What is the best treatment option for this patient?

    Correct

    Heterotopic ossification (HO) is the formation of bone in soft tissue, which is most commonly seen after total hip arthroplasty. HO can start to grow within 16 hours after surgery and progressively enlarge up to 1-2 years postoperatively. HO generally becomes visible 3-4 weeks post-operatively on x-rays. Hypertrophic osteoarthritis, Paget’s disease, ankylosing spondylitis, and diffuse idiopathic skeletal hyperostosis (DISH) are risk factors for HO formation. The amount of HO formation varies from small islands of bone in soft tissue, progressing to bone spurs from the femur and pelvis, to complete ankylosing of the hip. The amount of pain and loss of hip motion caused by HO can vary by the amount of HO formed, but in general a significant loss of hip function is rare and many patients never notice. Patients at a high risk must be identified preoperatively and treated with NSAIDS (most commonly indomethacin) and/or external beam radiation to prevent formation. Aspirin can also decrease the risk of HO formation and is commonly used for DVT prophylaxis postoperatively. Once HO forms and is visible on x-ray the only way to remove it is surgical excision. This patient is asymptomatic and his motion loss has not caused hip impairment so he should be treated with observation. 1,2,3

     

    Answer A.

    References

    1. Iorio, Richard MD; Healy, William L. MD Heterotopic Ossification After Hip and Knee Arthroplasty: Risk Factors, Prevention, and Treatment, Journal of the American Academy of Orthopaedic Surgeons: November 2002 – Volume 10 – Issue 6 – p 409-416
    2. Cohn RM, Schwarzkopf R, Jaffe F. Heterotopic ossification after total hip arthroplasty. Am J Orthop. 2011 Nov 1;40(11):E232-5.
    3. Kocic M, Lazovic M, Mitkovic M, Djokic B. Clinical significance of the heterotopic ossification after total hip arthroplasty. Orthopedics (Online). 2010;33(1):16.

     

    Incorrect

    Heterotopic ossification (HO) is the formation of bone in soft tissue, which is most commonly seen after total hip arthroplasty. HO can start to grow within 16 hours after surgery and progressively enlarge up to 1-2 years postoperatively. HO generally becomes visible 3-4 weeks post-operatively on x-rays. Hypertrophic osteoarthritis, Paget’s disease, ankylosing spondylitis, and diffuse idiopathic skeletal hyperostosis (DISH) are risk factors for HO formation. The amount of HO formation varies from small islands of bone in soft tissue, progressing to bone spurs from the femur and pelvis, to complete ankylosing of the hip. The amount of pain and loss of hip motion caused by HO can vary by the amount of HO formed, but in general a significant loss of hip function is rare and many patients never notice. Patients at a high risk must be identified preoperatively and treated with NSAIDS (most commonly indomethacin) and/or external beam radiation to prevent formation. Aspirin can also decrease the risk of HO formation and is commonly used for DVT prophylaxis postoperatively. Once HO forms and is visible on x-ray the only way to remove it is surgical excision. This patient is asymptomatic and his motion loss has not caused hip impairment so he should be treated with observation. 1,2,3

     

    Answer A.

    References

    1. Iorio, Richard MD; Healy, William L. MD Heterotopic Ossification After Hip and Knee Arthroplasty: Risk Factors, Prevention, and Treatment, Journal of the American Academy of Orthopaedic Surgeons: November 2002 – Volume 10 – Issue 6 – p 409-416
    2. Cohn RM, Schwarzkopf R, Jaffe F. Heterotopic ossification after total hip arthroplasty. Am J Orthop. 2011 Nov 1;40(11):E232-5.
    3. Kocic M, Lazovic M, Mitkovic M, Djokic B. Clinical significance of the heterotopic ossification after total hip arthroplasty. Orthopedics (Online). 2010;33(1):16.

     

  21. Question 21 of 64
    21. Question


    A 56-year-old male presents to the office with right shoulder pain after a fall two weeks ago. He has significant pain after the fall and is having difficulty lifting his arm. He has also noticed a slight bump on top of his shoulder since the fall. However, his pain and range of motion are improving over the last week. AP x-ray is shown in figure 1. What benefit would surgical management provide this patient?

    Correct

    The patient has sustained a grade III AC joint separation with 100% displacement of the distal clavicle on AP radiograph. Grade III injuries disrupt both the coracoclavicular and acromioclavicular ligaments causing horizontal and vertical instability. These injuries are almost always treated non-operatively. Surgical management may be considered for grade III injuries in overhead athletes who fail conservative treatment and continue to have symptoms with overhead activities. At one year after injury patients can expect no difference in strength and range of motion between the injured and non-injured shoulder within non-operative treatment. Patients have an earlier return to sports and work with non-surgical treatment compared to surgical treatment. Surgery also brings an increased risk of complications such as wound infection and failure of fixation. Non-operative treatment involves the use of a sling for comfort for 4-6 weeks. Light range of motion can be initiated with physical therapy during this time. Active reaching overhead usually starts at 6 weeks and patients often regain their overhead strength at 3 months. 1,2

    Answer D.

    References

    1. Frank RM, Cotter, Eric J, Leroux, Timothy S, Romeo AA. Acromioclavicular Joint Injuries: Evidence-based Treatment, Journal of the American Academy of Orthopaedic Surgeons: September 1, 2019 – Volume 27 – Issue 17 – p e775-e788
    2. Cook JB, Krul KP. Challenges in Treating Acromioclavicular Separations: Current Concepts, Journal of the American Academy of Orthopaedic Surgeons: October 1, 2018 – Volume 26 – Issue 19 – p 669-677
    Incorrect

    The patient has sustained a grade III AC joint separation with 100% displacement of the distal clavicle on AP radiograph. Grade III injuries disrupt both the coracoclavicular and acromioclavicular ligaments causing horizontal and vertical instability. These injuries are almost always treated non-operatively. Surgical management may be considered for grade III injuries in overhead athletes who fail conservative treatment and continue to have symptoms with overhead activities. At one year after injury patients can expect no difference in strength and range of motion between the injured and non-injured shoulder within non-operative treatment. Patients have an earlier return to sports and work with non-surgical treatment compared to surgical treatment. Surgery also brings an increased risk of complications such as wound infection and failure of fixation. Non-operative treatment involves the use of a sling for comfort for 4-6 weeks. Light range of motion can be initiated with physical therapy during this time. Active reaching overhead usually starts at 6 weeks and patients often regain their overhead strength at 3 months. 1,2

    Answer D.

    References

    1. Frank RM, Cotter, Eric J, Leroux, Timothy S, Romeo AA. Acromioclavicular Joint Injuries: Evidence-based Treatment, Journal of the American Academy of Orthopaedic Surgeons: September 1, 2019 – Volume 27 – Issue 17 – p e775-e788
    2. Cook JB, Krul KP. Challenges in Treating Acromioclavicular Separations: Current Concepts, Journal of the American Academy of Orthopaedic Surgeons: October 1, 2018 – Volume 26 – Issue 19 – p 669-677
  22. Question 22 of 64
    22. Question


    A 50-year-old male presents to your office with a left foot injury he sustained 2 days ago after falling off a ladder. The foot landed flat on hard ground from about 6 feet off the ground. He has had pain and swelling in the foot and is unable to bear weight. On physical exam of the left foot he has some swelling about the midfoot and tenderness to palpation over the dorsum of the navicula. Distal neurovascular status is intact. AP, oblique, and lateral x-rays are shown in figures 1,2, and 3 respectively. What is the best treatment option for this patient?

    Correct

    The patient sustained a navicular body fracture with 4 to 5mm’s of displacement. The navicular is an important supporting structure in the midfoot for walking as it allows the foot to be flexible at heal strike and more rigid at toe off. Navicular fractures are described as avulsion fractures, tuberosity fractures, and body fractures. Avulsion fractures represent a disruption of the dorsal talonavicular ligaments. Body fractures are further divided into three subtypes based on fracture orientation and comminution. CT is routinely ordered for displaced fractures to better characterize the fracture pattern. Nonoperative treatment is recommended for avulsion fractures, nondisplaced body fractures, and tuberosity fractures with less than 2mm of displacement. Simple body fractures are usually treated with two interfragmentary screws where more comminuted displaced fractures may require plate and screw constructs. 1,2

    Answer D.

    References

    1. Rosenbaum AJ, Depreta JA, Tartaglione J, Patel N, Richard L. Acute Fractures of the Tarsal Navicular A Critical Analysis ReviewJBJS Reviews – Review Article with Critical Analysis Component: 31 March 2015 – Volume 3 – Issue 3 – p. e
    2. Tarsalnavicular fractures. http://www.orthobullets.com. Accessed on 3/7/21.
    Incorrect

    The patient sustained a navicular body fracture with 4 to 5mm’s of displacement. The navicular is an important supporting structure in the midfoot for walking as it allows the foot to be flexible at heal strike and more rigid at toe off. Navicular fractures are described as avulsion fractures, tuberosity fractures, and body fractures. Avulsion fractures represent a disruption of the dorsal talonavicular ligaments. Body fractures are further divided into three subtypes based on fracture orientation and comminution. CT is routinely ordered for displaced fractures to better characterize the fracture pattern. Nonoperative treatment is recommended for avulsion fractures, nondisplaced body fractures, and tuberosity fractures with less than 2mm of displacement. Simple body fractures are usually treated with two interfragmentary screws where more comminuted displaced fractures may require plate and screw constructs. 1,2

    Answer D.

    References

    1. Rosenbaum AJ, Depreta JA, Tartaglione J, Patel N, Richard L. Acute Fractures of the Tarsal Navicular A Critical Analysis ReviewJBJS Reviews – Review Article with Critical Analysis Component: 31 March 2015 – Volume 3 – Issue 3 – p. e
    2. Tarsalnavicular fractures. http://www.orthobullets.com. Accessed on 3/7/21.
  23. Question 23 of 64
    23. Question


    A 48-year-old male presents to the office with left shoulder pain for the past 3 months. He denies having an injury and the pain is made worse when reaching over his head. On physical exam he has significant tenderness to palpation over the acromion and mildly positive impingement signs. Grashey, outlet, and axillary view radiographs show an apparent os acromiale. Which diagnostic study would be the best option for this patient?

    Correct

    Os acromiale is a failure of fusion of the anterior acromial apophysis and is found in an estimated 8% of the population worldwide. There are three ossification centers in the acromion including the pre-acromion, meso-acromion, and meta-acromion.  The three ossification centers are expected to be completely fused by the age of 25. Most os acromiale are asymptomatic and found incidentally on radiographs. The os acromiale can be a symptomatic non-union or cause dynamic impingement as the unfused fragment moves with shoulder motion. Physical exam findings include should pain with overhead motion with pain directly over the superior acromion. Three view radiographs should be taken including an AP, outlet and axillary views of the shoulder. Os acromiale is often missed on AP radiograph and is most often seen on axillary view. MRI is most commonly used image modality to confirm the diagnosis and help determine if other pathology is present. CT scan is a good option to help seen the unfused acromion. Conservative treatment is recommended for at least a 6-month period before surgery is considered. Surgical options include excision for symptomatic small fragments, arthroscopic decompression (and rotator cuff repair if present), and/or open fusion (open reduction and internal fixation). 1,2

    Answer B.

     

    References

    1. You T, Frostick S, Zhang WT, Yin Q. Os acromiale: reviews and current perspectives. Orthopaedic surgery. 2019 Oct;11(5):738-44.
    2. Hurst SA, Gregory TM, Reilly P. Os acromiale: a review of its incidence, pathophysiology, and clinical management. EFORT open reviews. 2019 Aug;4(8):525-32.
    Incorrect

    Os acromiale is a failure of fusion of the anterior acromial apophysis and is found in an estimated 8% of the population worldwide. There are three ossification centers in the acromion including the pre-acromion, meso-acromion, and meta-acromion.  The three ossification centers are expected to be completely fused by the age of 25. Most os acromiale are asymptomatic and found incidentally on radiographs. The os acromiale can be a symptomatic non-union or cause dynamic impingement as the unfused fragment moves with shoulder motion. Physical exam findings include should pain with overhead motion with pain directly over the superior acromion. Three view radiographs should be taken including an AP, outlet and axillary views of the shoulder. Os acromiale is often missed on AP radiograph and is most often seen on axillary view. MRI is most commonly used image modality to confirm the diagnosis and help determine if other pathology is present. CT scan is a good option to help seen the unfused acromion. Conservative treatment is recommended for at least a 6-month period before surgery is considered. Surgical options include excision for symptomatic small fragments, arthroscopic decompression (and rotator cuff repair if present), and/or open fusion (open reduction and internal fixation). 1,2

    Answer B.

     

    References

    1. You T, Frostick S, Zhang WT, Yin Q. Os acromiale: reviews and current perspectives. Orthopaedic surgery. 2019 Oct;11(5):738-44.
    2. Hurst SA, Gregory TM, Reilly P. Os acromiale: a review of its incidence, pathophysiology, and clinical management. EFORT open reviews. 2019 Aug;4(8):525-32.
  24. Question 24 of 64
    24. Question


    A 22-year-old female presents to the office with severe right knee pain after her sprinting dog struck her knee while she was standing.  She is having severe pain bearing weight on the right leg since the injury. On physical exam she has a large right knee effusion and significant laxity of the medial collateral ligament (MCL)  at 20 degrees and at 0 degrees. She has a negative Lachman’s and posterior drawer. She has severe tenderness to palpation along the medial tibial plateau at the insertion area of the MCL. Initial x-rays taken in the office are normal. A coronal MRI image (figure 1) shows a 2 mm depressed lateral tibial plateau fracture with a complete rupture of the distal MCL off the tibia. What is the best treatment option?

    Correct

    The medial collateral ligament (MCL) is the most commonly injured ligament in the knee. The mechanism of injury is generally an impact to the lateral knee that causes a valgus stress to the knee. Isolated grade I and II MCL injuries are the most common types which can be treated non-operatively. Grade III injuries (the most severe type) occur with some other concomitant knee injury in 80% of cases; most commonly ACL injuries, followed by meniscus tears and plateau fractures. Generally isolated grade III MCL injuries can be treated non-operatively however the location of the tear plays a crucial role in predicting healing. MRI is crucial in determining the location of the tear and should be performed for all Grade III MCL tears. A careful physical exam can locate the site of the MCL tear with point of maximal tenderness to palpation being proximal on the femur, midsubstance, or distal on the tibia. Complete distal MCL tears off the tibial side have a tendency not to heal with non-operative treatment, particularly in the athletic population. Distal MCL injuries can cause entrapment of the ligament in the medial joint space or displacement over the pes anserinus tendons (Stener like lesion).  For this reason, acute MCL injuries in athletes with complete disruption at the tibial side should be treated with surgical repair within 7-10 days of injury. 1,2

    Answer C.

    References

    1. Miyamoto, Ryan G. MD; Bosco, Joseph A. MD; Sherman, Orrin H. MD Treatment of Medial Collateral Ligament Injuries, Journal of the American Academy of Orthopaedic Surgeons: March 2009 – Volume 17 – Issue 3 – p 152-161
    2. Wilson TC, Satterfield WH, Johnson DL. Medial collateral ligament” tibial” injuries: indication for acute repair. Orthopedics. 2004 Apr 1;27(4):389-93.
    Incorrect

    The medial collateral ligament (MCL) is the most commonly injured ligament in the knee. The mechanism of injury is generally an impact to the lateral knee that causes a valgus stress to the knee. Isolated grade I and II MCL injuries are the most common types which can be treated non-operatively. Grade III injuries (the most severe type) occur with some other concomitant knee injury in 80% of cases; most commonly ACL injuries, followed by meniscus tears and plateau fractures. Generally isolated grade III MCL injuries can be treated non-operatively however the location of the tear plays a crucial role in predicting healing. MRI is crucial in determining the location of the tear and should be performed for all Grade III MCL tears. A careful physical exam can locate the site of the MCL tear with point of maximal tenderness to palpation being proximal on the femur, midsubstance, or distal on the tibia. Complete distal MCL tears off the tibial side have a tendency not to heal with non-operative treatment, particularly in the athletic population. Distal MCL injuries can cause entrapment of the ligament in the medial joint space or displacement over the pes anserinus tendons (Stener like lesion).  For this reason, acute MCL injuries in athletes with complete disruption at the tibial side should be treated with surgical repair within 7-10 days of injury. 1,2

    Answer C.

    References

    1. Miyamoto, Ryan G. MD; Bosco, Joseph A. MD; Sherman, Orrin H. MD Treatment of Medial Collateral Ligament Injuries, Journal of the American Academy of Orthopaedic Surgeons: March 2009 – Volume 17 – Issue 3 – p 152-161
    2. Wilson TC, Satterfield WH, Johnson DL. Medial collateral ligament” tibial” injuries: indication for acute repair. Orthopedics. 2004 Apr 1;27(4):389-93.
  25. Question 25 of 64
    25. Question


    An 11-year-old female presents to your office with her parents complaining of severe right hip pain and difficulty walking. She denies having a known injury and the pain has progressively increased over the last 4 weeks. She is very healthy and active with no underlying medical conditions. On physical exam she has pain with hip motion but no obvious skin changes or deformity. AP pelvis and lateral x-rays are shown in figures 1 and 2, respectively. What is the next best step in treatment?

    Correct

    The patient is presenting with a chronic slipped capital femoral epiphysis (SCFE). Chronic SCFE is defined as symptoms lasting >3 weeks and is by far the most common form, accounting for 85% of all patients who present with the condition. The stability of SCFE is based on the patient’s ability to bear weight. If patients can’t put any weight on the leg the SCFE is unstable whereas the ability to weight bear represents a stable pattern. Given the patient was able to walk in, although with pain, her SCFE would be classified as stable. The patient’s x-rays show a grade I mild slip which appears chronic. Given the duration of symptoms and the patients difficulty with bearing weight, the SCFE is still not healed. MRI has a role in diagnosing early pre-slip SCFE in the presence of normal radiographs, however this patient has growth plate changes noted. The treatment of choice for stable and unstable SCFE is single or double screw fixation to prevent further slippage of the growth plate. Unstable and severe slips may require open reduction of the growth plate with fixation. 1,2

    Answer D.

    References

    1. Aronsson, David D. MD; Loder, Randall T. MD; Breur, Gert J. DVM, PhD; Weinstein, Stuart L. MD Slipped Capital Femoral Epiphysis: Current Concepts, Journal of the American Academy of Orthopaedic Surgeons: November 2006 – Volume 14 – Issue 12 – p 666-679
    2. Wylie JD, Novais EN. Evolving Understanding of and Treatment Approaches to Slipped Capital Femoral Epiphysis. Curr Rev Musculoskelet Med. 2019;12(2):213-219. doi:10.1007/s12178-019-09547-5
    Incorrect

    The patient is presenting with a chronic slipped capital femoral epiphysis (SCFE). Chronic SCFE is defined as symptoms lasting >3 weeks and is by far the most common form, accounting for 85% of all patients who present with the condition. The stability of SCFE is based on the patient’s ability to bear weight. If patients can’t put any weight on the leg the SCFE is unstable whereas the ability to weight bear represents a stable pattern. Given the patient was able to walk in, although with pain, her SCFE would be classified as stable. The patient’s x-rays show a grade I mild slip which appears chronic. Given the duration of symptoms and the patients difficulty with bearing weight, the SCFE is still not healed. MRI has a role in diagnosing early pre-slip SCFE in the presence of normal radiographs, however this patient has growth plate changes noted. The treatment of choice for stable and unstable SCFE is single or double screw fixation to prevent further slippage of the growth plate. Unstable and severe slips may require open reduction of the growth plate with fixation. 1,2

    Answer D.

    References

    1. Aronsson, David D. MD; Loder, Randall T. MD; Breur, Gert J. DVM, PhD; Weinstein, Stuart L. MD Slipped Capital Femoral Epiphysis: Current Concepts, Journal of the American Academy of Orthopaedic Surgeons: November 2006 – Volume 14 – Issue 12 – p 666-679
    2. Wylie JD, Novais EN. Evolving Understanding of and Treatment Approaches to Slipped Capital Femoral Epiphysis. Curr Rev Musculoskelet Med. 2019;12(2):213-219. doi:10.1007/s12178-019-09547-5
  26. Question 26 of 64
    26. Question


    A 55-year-old male presents to your office with a one-year history of right knee pain. He has a known history of medial compartment arthritis. He has tried conservative treatments including oral NSAIDS, physical therapy, and cortisone injections with minimal relief. He had an arthroscopy with medial meniscectomy eight months ago which only made the knee pain worse. AP and sunrise view x-rays are shown in figures 1 and 2, respectively. MRI (figure 3) shows significant edema and cystic changes within the femoral condyle and proximal tibia. The articular cartilage of the lateral and patellofemoral components are preserved. He would like to discuss medial unicompartmental knee replacement. What is the difference between unicompartmental knee replacement (UKA) and total knee replacement (TKA)?

    Correct

    A unicompartmental knee replacement (UKA) refers a replacement of either the medial, lateral, or patellofemoral compartment. UKA’s represent only 5% of knee arthroplasty procedures. The most common compartment to develop arthritis is the medial compartment, which is also the most common location of a UKA. A medial UKA preserves the lateral and patellofemoral compartment, as well as the anterior and posterior cruciate ligaments, as compared to a total knee arthroplasty (TKA). This results in a more natural feeling knee and better range of motion as compared to a TKA. The most common cause of failure of a medial UKA is knee pain from arthritis that develops in the lateral or patellofemoral compartment. Robotic assisted UKA has helped with component positioning and ligament balancing with hopes that long term studies will show increased implant survivorship. Contraindications for a medial UKA include rheumatoid arthritis (which effects all compartments), ACL insufficiency, history of lateral meniscectomy, and flexion contraction >10 degrees. 1,2

    Answer D.

    References

    1. Jennings, Jason M. MD, DPT; Kleeman-Forsthuber, Lindsay T. MD; Bolognesi, Michael P. MD Medial Unicompartmental Arthroplasty of the Knee, Journal of the American Academy of Orthopaedic Surgeons: March 1, 2019 – Volume 27 – Issue 5 – p 166-176
    2. Lonner, Jess H. MD; Klement, Mitchell R. MD Robotic-assisted Medial Unicompartmental Knee Arthroplasty: Options and Outcomes, Journal of the American Academy of Orthopaedic Surgeons: March 1, 2019 – Volume 27 – Issue 5 – p e207-e214

     

    Incorrect

    A unicompartmental knee replacement (UKA) refers a replacement of either the medial, lateral, or patellofemoral compartment. UKA’s represent only 5% of knee arthroplasty procedures. The most common compartment to develop arthritis is the medial compartment, which is also the most common location of a UKA. A medial UKA preserves the lateral and patellofemoral compartment, as well as the anterior and posterior cruciate ligaments, as compared to a total knee arthroplasty (TKA). This results in a more natural feeling knee and better range of motion as compared to a TKA. The most common cause of failure of a medial UKA is knee pain from arthritis that develops in the lateral or patellofemoral compartment. Robotic assisted UKA has helped with component positioning and ligament balancing with hopes that long term studies will show increased implant survivorship. Contraindications for a medial UKA include rheumatoid arthritis (which effects all compartments), ACL insufficiency, history of lateral meniscectomy, and flexion contraction >10 degrees. 1,2

    Answer D.

    References

    1. Jennings, Jason M. MD, DPT; Kleeman-Forsthuber, Lindsay T. MD; Bolognesi, Michael P. MD Medial Unicompartmental Arthroplasty of the Knee, Journal of the American Academy of Orthopaedic Surgeons: March 1, 2019 – Volume 27 – Issue 5 – p 166-176
    2. Lonner, Jess H. MD; Klement, Mitchell R. MD Robotic-assisted Medial Unicompartmental Knee Arthroplasty: Options and Outcomes, Journal of the American Academy of Orthopaedic Surgeons: March 1, 2019 – Volume 27 – Issue 5 – p e207-e214

     

  27. Question 27 of 64
    27. Question


    You are asked to consult on a 58-year-old diabetic male who was admitted to the hospital two days ago with frost bite injuries to bilateral feet. One week ago he helped a neighbor shovel her driveway while wearing tennis shoes. He noticed when he came inside that his toes were cold, swollen, and red. Two days later he noticed his toes start to turn black and purple. Over the next few days he started to get pain in the toes and trouble walking. He was admitted to the hospital and a vascular surgery consult determined he had good blood flow to bilateral feet. Images of the left foot taken on hospital admission are shown in figures 1 and 2. The right foot looks nearly identical to the left. What is the next best step in treatment for this patient?

    Correct

    This patient sustained frost bite injuries to bilateral feet. As tissue begins to cool and freeze upon exposure to a cold environment, microvascular vasoconstriction develops which can cause local tissue ischemia and paresthesia. Prolonged exposure results in eventual capillary blood flow disruption, thrombosis, and irreversible tissue damage. Tissue injury also causes a release of inflammatory mediators, which peak during rewarming, leading to further hypoxia. Thrombolytics, rewarming techniques, anti-inflammatory medications, hyperbaric oxygen, and antibiotics are all used during the early phase of a frost bite injury, however, after a week out from injury the soft tissue damage is likely irreversible and these treatments are no longer helpful. Severe frost bite injuries can form a hard, black, leathery eschar over a week from injury. After three weeks a clear line of demarcation starts to form separating viable tissue with eschar. Dead tissue and eschar require amputation until viable tissue is reached. The line of demarcation may take up to 6-8 weeks to become clear so toe/foot amputation after frost bite injury is not performed urgently, but rather electively when the amount of tissue requiring amputation becomes clear. 1,2

    Answer D.

    References

    1. Millet JD, Brown RK, Levi B, Kraft CT, Jacobson JA, Gross MD, Wong KK. Frostbite: spectrum of imaging findings and guidelines for management. Radiographics. 2016 Nov;36(7):2154-69.
    2. Grieve AW, Davis P, Dhillon S, Richards P, Hillebrandt D, Imray C. A clinical review of the management of frostbite. BMJ Military Health. 2011 Mar 1;157(1):73-8.
    Incorrect

    This patient sustained frost bite injuries to bilateral feet. As tissue begins to cool and freeze upon exposure to a cold environment, microvascular vasoconstriction develops which can cause local tissue ischemia and paresthesia. Prolonged exposure results in eventual capillary blood flow disruption, thrombosis, and irreversible tissue damage. Tissue injury also causes a release of inflammatory mediators, which peak during rewarming, leading to further hypoxia. Thrombolytics, rewarming techniques, anti-inflammatory medications, hyperbaric oxygen, and antibiotics are all used during the early phase of a frost bite injury, however, after a week out from injury the soft tissue damage is likely irreversible and these treatments are no longer helpful. Severe frost bite injuries can form a hard, black, leathery eschar over a week from injury. After three weeks a clear line of demarcation starts to form separating viable tissue with eschar. Dead tissue and eschar require amputation until viable tissue is reached. The line of demarcation may take up to 6-8 weeks to become clear so toe/foot amputation after frost bite injury is not performed urgently, but rather electively when the amount of tissue requiring amputation becomes clear. 1,2

    Answer D.

    References

    1. Millet JD, Brown RK, Levi B, Kraft CT, Jacobson JA, Gross MD, Wong KK. Frostbite: spectrum of imaging findings and guidelines for management. Radiographics. 2016 Nov;36(7):2154-69.
    2. Grieve AW, Davis P, Dhillon S, Richards P, Hillebrandt D, Imray C. A clinical review of the management of frostbite. BMJ Military Health. 2011 Mar 1;157(1):73-8.
  28. Question 28 of 64
    28. Question


    A 61-year-old male presents to the ED with a 2-month history of a left foot infection. He is a poorly controlled insulin dependent diabetic with a history of right knee amputation two years ago. The left foot infection started as cellulitis over the last two months and progressed to full necrosis of the lateral aspect of this foot. He admits to black necrosis for the last 4 weeks. The patient has limited mental capacity and has not been taking his insulin routinely. He sought care “after seeing several bugs on the wound”. Figure 1 and 2 are images of his gangrenous left foot.  Blood cultures show a staphylococcus aureus bacteremia. Which intervention would best allow for a successful limited amputation compared to the need for a below the knee amputation?

    Correct

    This patient presents with an ischemic foot that has resulted in a gangrenous foot infection. The first priority for this patient is to remove the infected gangrenous tissue. Gangrene is an irreversible death of tissue due to loss of blood supply. Dry gangrene causes skin and tissues to dry, shrink, and turn black. Dry gangrene progresses slowly and can result it auto-amputation of the involved finger or toe. Wet gangrene can occur if bacteria starts to invade the avascular tissue causing swelling, drainage, and a foul smell. Treatment of wet gangrene is more emergent as the infection can spread quickly. It is crucial to assess the extremity for arterial occlusive disease with ultrasound or arteriogram studies. If ateriovascular reperfusion can be established, a more limited debridement has more success with healing. For example, a partial foot amputation may be viable if foot perfusion is re-established instead of a below the knee amputation for an ischemic foot. 1,2

    The patient had an arteriogram of the left lower extremity which showed an occlusion of the superficial femoral artery (FSA) and distal left anterior tibial artery. A percutaneous balloon angioplasty was performed to perfuse the anterior tibial artery and the FSA. The anterior tibial artery supply’s the dorsum of the foot where the posterior tibial artery supplies most of the plantar aspect of the foot. A left foot transmetatarsal amputation was performed to remove dead tissue due to his ischemic foot. 2

    Answer A.

    References

    1. Kota SK, Kota SK, Meher LK, Sahoo S, Mohapatra S, Modi KD. Surgical revascularization techniques for diabetic foot. J Cardiovasc Dis Res. 2013;4(2):79-83.
    2. Kinlay S. Management of Critical Limb Ischemia. Circ Cardiovasc Interv. 2016;9(2):e001946. doi:10.1161/CIRCINTERVENTIONS.115.001946

     

     

    Incorrect

    This patient presents with an ischemic foot that has resulted in a gangrenous foot infection. The first priority for this patient is to remove the infected gangrenous tissue. Gangrene is an irreversible death of tissue due to loss of blood supply. Dry gangrene causes skin and tissues to dry, shrink, and turn black. Dry gangrene progresses slowly and can result it auto-amputation of the involved finger or toe. Wet gangrene can occur if bacteria starts to invade the avascular tissue causing swelling, drainage, and a foul smell. Treatment of wet gangrene is more emergent as the infection can spread quickly. It is crucial to assess the extremity for arterial occlusive disease with ultrasound or arteriogram studies. If ateriovascular reperfusion can be established, a more limited debridement has more success with healing. For example, a partial foot amputation may be viable if foot perfusion is re-established instead of a below the knee amputation for an ischemic foot. 1,2

    The patient had an arteriogram of the left lower extremity which showed an occlusion of the superficial femoral artery (FSA) and distal left anterior tibial artery. A percutaneous balloon angioplasty was performed to perfuse the anterior tibial artery and the FSA. The anterior tibial artery supply’s the dorsum of the foot where the posterior tibial artery supplies most of the plantar aspect of the foot. A left foot transmetatarsal amputation was performed to remove dead tissue due to his ischemic foot. 2

    Answer A.

    References

    1. Kota SK, Kota SK, Meher LK, Sahoo S, Mohapatra S, Modi KD. Surgical revascularization techniques for diabetic foot. J Cardiovasc Dis Res. 2013;4(2):79-83.
    2. Kinlay S. Management of Critical Limb Ischemia. Circ Cardiovasc Interv. 2016;9(2):e001946. doi:10.1161/CIRCINTERVENTIONS.115.001946

     

     

  29. Question 29 of 64
    29. Question


    A 16-year old male high school basketball player presents to the office with a history of right foot pain. He initially injured his foot 2 months ago which resulted in a zone 3 proximal fifth diaphyseal metatarsal fracture that was treated nonoperatively. Figure 1 is an oblique x-ray at the time of injury. He was healing the fracture after being treated in a walking boot and crutches. An oblique x-ray at 6 weeks is shown in figure 2. He was transitioned into a sneaker for walking only at 8 weeks but the foot pain quickly returned. What is the next best step in treatment for this patient?

    Correct

    The patient was initially treated appropriately for a minimally displaced proximal 5th metatarsal fracture with a walking boot and crutches. The fracture is generally healed by 6-8 weeks and patients should be pain free by this time. Foot pain beyond 2 months raises suspicion of a developing non-union. CT scan is the best imaging modality to assess healing which would be indicated by bridging callus. For patients who fail to achieve fracture healing (osseous union) after 3 months of conservative treatment, surgery is indicated and includes open autogenous bone-grafting with percutaneous screw fixation. The use of a bone stimulator may help improve healing in patients showing signs of delayed union at 6-8 weeks. A bone stimulator is often used after surgery to improve healing rates as well. MRI is generally used to rule a stress fracture of the 5th metatarsal when radiographs are normal. 1,2

    Answer B.

    References

    1. Lareau CR, Anderson RB. Jones Fractures. Pathophysiology and Treatment. JBJS Reviews – Review Article: 28 July 2015 – Volume 3 – Issue 7 – p. e4
    2. 5th Metatarsal base fracture. http://www.orthobullets.com. Accessed on 7/26/21.
    Incorrect

    The patient was initially treated appropriately for a minimally displaced proximal 5th metatarsal fracture with a walking boot and crutches. The fracture is generally healed by 6-8 weeks and patients should be pain free by this time. Foot pain beyond 2 months raises suspicion of a developing non-union. CT scan is the best imaging modality to assess healing which would be indicated by bridging callus. For patients who fail to achieve fracture healing (osseous union) after 3 months of conservative treatment, surgery is indicated and includes open autogenous bone-grafting with percutaneous screw fixation. The use of a bone stimulator may help improve healing in patients showing signs of delayed union at 6-8 weeks. A bone stimulator is often used after surgery to improve healing rates as well. MRI is generally used to rule a stress fracture of the 5th metatarsal when radiographs are normal. 1,2

    Answer B.

    References

    1. Lareau CR, Anderson RB. Jones Fractures. Pathophysiology and Treatment. JBJS Reviews – Review Article: 28 July 2015 – Volume 3 – Issue 7 – p. e4
    2. 5th Metatarsal base fracture. http://www.orthobullets.com. Accessed on 7/26/21.
  30. Question 30 of 64
    30. Question

    A 69-year-old male presents to the emergency room (ER) with right hip pain after a fall on ice a few hours earlier in the day.  He was unable to get up and walk after the fall and was transported to the ER via ambulance. X-ray of the right hip shows a displaced femoral neck fracture.  He is a very healthy active patient who enjoys playing tennis on a near daily basis. What is the best treatment option?

    Correct

    Hemiarthroplasty, or replacement of the femoral side only, is the most common procedure performed in elderly (over 65 years of age) patients with displaced femoral neck fractures. However, over the last decade, total hip arthroplasty (THA) has become an increasingly popular treatment option for these patients. The most common indications for a THA after a displaced femoral neck fracture includes pre-existing hip arthritis and a fracture in an active elderly patient. Many studies show fewer complications associated with THA compared to hemiathroplasty for displaced femoral neck fractures, although most patients who have a hemiathroplasty are older and sicker patients. In general, THA has been shown to offer better hip function and quality of life compared to hemiarthroplasty. Low demand elderly patients are more appropriate for a hemiathroplasty which is generally associated with less operative time and blood loss. 1,2

    This patient is very active and has pre-existing arthritis (note the sclerosis on the acetabulum) making him a good candidate for a total hip arthroplasty. Percutaneous screw fixation is used for non-displaced or minimally displaced fractures in elderly patients and for displaced fractures in younger patients after closed reduction.

    Answer D.

    References

    1. Health Investigators. Total hip arthroplasty or hemiarthroplasty for hip fracture. New England Journal of Medicine. 2019 Dec 5;381(23):2199-208.
    2. Stronach BM, Bergin PF, Perez JL, Watson S, Jones LC, McGwin G, Ponce BA. The rising use of total hip arthroplasty for femoral neck fractures in the United States. HIP International. 2020 Jan;30(1):107-13.

     

     

     

    Incorrect

    Hemiarthroplasty, or replacement of the femoral side only, is the most common procedure performed in elderly (over 65 years of age) patients with displaced femoral neck fractures. However, over the last decade, total hip arthroplasty (THA) has become an increasingly popular treatment option for these patients. The most common indications for a THA after a displaced femoral neck fracture includes pre-existing hip arthritis and a fracture in an active elderly patient. Many studies show fewer complications associated with THA compared to hemiathroplasty for displaced femoral neck fractures, although most patients who have a hemiathroplasty are older and sicker patients. In general, THA has been shown to offer better hip function and quality of life compared to hemiarthroplasty. Low demand elderly patients are more appropriate for a hemiathroplasty which is generally associated with less operative time and blood loss. 1,2

    This patient is very active and has pre-existing arthritis (note the sclerosis on the acetabulum) making him a good candidate for a total hip arthroplasty. Percutaneous screw fixation is used for non-displaced or minimally displaced fractures in elderly patients and for displaced fractures in younger patients after closed reduction.

    Answer D.

    References

    1. Health Investigators. Total hip arthroplasty or hemiarthroplasty for hip fracture. New England Journal of Medicine. 2019 Dec 5;381(23):2199-208.
    2. Stronach BM, Bergin PF, Perez JL, Watson S, Jones LC, McGwin G, Ponce BA. The rising use of total hip arthroplasty for femoral neck fractures in the United States. HIP International. 2020 Jan;30(1):107-13.

     

     

     

  31. Question 31 of 64
    31. Question


    A 56-year-old male presents to the office 4 weeks out from a crush injury to his left index finger. He was working in the garage when a heavy object came down suddenly on the tip of his finger. On physical exam the nail is intact and he has tenderness to palpation over the tip of his finger. AP and lateral x-rays of the left hand show a comminuted fracture of the distal phalanx. He is a mechanic and is concerned about his finger sensitivity. Which statement is true regarding his fracture?

    Correct

    The distal phalanx is one of the most commonly fractured bone in the body. These fractures can cause significant disability both short term and long term with day to day activities. The tip of the finger has an abundance of nerves necessary for touch sensation. A crush injury to the nerves and bone can cause the fingertip to remain painful when picking up objects or pushing down with the finger for several months after the injury. Over 70% of distal phalanx fractures will have fingertip sensitivity for up to 6 months with the cold hypersensitivity being the most common complaint. Closed longitudinal fractures have the best prognosis whereas >80% of tuft (the most distal aspect of the phalanx) fractures can have symptoms beyond 6 months. Fibrous non-unions often develop in comminuted fractures with no functional consequences. Distal phalanx fractures rarely need surgical intervention as even the non-unions are asymptomatic. Treatment often involves 4-6 weeks of a clamshell type splint. 1,2

    Answer A.

    References

    1. DaCruz DJ, Slade RJ, Malone W. Fractures of the distal phalanges. The Journal of Hand Surgery: British & European Volume. 1988 Aug 1;13(3):350-2.
    2. Lee, Donald H. MD; Mignemi, Megan E. MD; Crosby, Samuel N. MD Fingertip Injuries: An Update on Management, Journal of the American Academy of Orthopaedic Surgeons: December 2013 – Volume 21 – Issue 12 – p 756-766

     

    Incorrect

    The distal phalanx is one of the most commonly fractured bone in the body. These fractures can cause significant disability both short term and long term with day to day activities. The tip of the finger has an abundance of nerves necessary for touch sensation. A crush injury to the nerves and bone can cause the fingertip to remain painful when picking up objects or pushing down with the finger for several months after the injury. Over 70% of distal phalanx fractures will have fingertip sensitivity for up to 6 months with the cold hypersensitivity being the most common complaint. Closed longitudinal fractures have the best prognosis whereas >80% of tuft (the most distal aspect of the phalanx) fractures can have symptoms beyond 6 months. Fibrous non-unions often develop in comminuted fractures with no functional consequences. Distal phalanx fractures rarely need surgical intervention as even the non-unions are asymptomatic. Treatment often involves 4-6 weeks of a clamshell type splint. 1,2

    Answer A.

    References

    1. DaCruz DJ, Slade RJ, Malone W. Fractures of the distal phalanges. The Journal of Hand Surgery: British & European Volume. 1988 Aug 1;13(3):350-2.
    2. Lee, Donald H. MD; Mignemi, Megan E. MD; Crosby, Samuel N. MD Fingertip Injuries: An Update on Management, Journal of the American Academy of Orthopaedic Surgeons: December 2013 – Volume 21 – Issue 12 – p 756-766

     

  32. Question 32 of 64
    32. Question


    A 52-year-old male presents to your office with right hip pain for 3 months. He works in construction and does a lot of bending and squatting but denies a known injury. The pain can be sharp and intense at times and is usually located in the groin. On physical exam the patient has pain with hip flexion, adduction, and internal rotation. AP x-ray of the right hip shows a prominence of the femoral head/neck junction indicated by a white arrow. What is this prominence called?

    Correct

    Femoroacetabular impingement (FAI) occurs when extra bone forms at the femoral head/neck junction and/or the acetabulum. The extra bone can cause abutment of the femoral head/neck junction and the acetabular rim during hip flexion and internal rotation. Repetitive contact between the bones can cause pain, decreased range or motion, labral abnormalities, and chondral wear. Extra bone at the femoral/head neck junction is called a CAM lesion. Extra bone on the acetabulum is called a pincer lesion.  Patients may present with just one lesion but up to 80% of patients have both lesions (combined CAM/ Pincer impingement). If a symptomatic CAM lesion is present and patients have failed at least 3 months of conservative treatment, an arthroscopic osteoplasty may be performed to “trim” down the extra bone. Hip arthroscopy is no indicated in patients with underlying hip arthritis. 1,2,3

    Answer C.

    References

    1. Horisberger M, Brunner A, Herzog RF. Arthroscopic treatment of femoroacetabular impingement of the hip: a new technique to access the joint. Clin Orthop Relat Res. 2010;468(1):182-190. doi:10.1007/s11999-009-1005-5
    2. Kweon C, Welton K, MD; Kelly BT, Larson C, Bedi A. Arthroscopic Treatment of Cam-Type Impingement of the Hip. JBJS Reviews – Review Article: 15 September 2015 – Volume 3 – Issue 9 – p. e3
    3. Femoroacetabular Impingement. http://www.orthobullets.com. Accessed on 4/21/21
    Incorrect

    Femoroacetabular impingement (FAI) occurs when extra bone forms at the femoral head/neck junction and/or the acetabulum. The extra bone can cause abutment of the femoral head/neck junction and the acetabular rim during hip flexion and internal rotation. Repetitive contact between the bones can cause pain, decreased range or motion, labral abnormalities, and chondral wear. Extra bone at the femoral/head neck junction is called a CAM lesion. Extra bone on the acetabulum is called a pincer lesion.  Patients may present with just one lesion but up to 80% of patients have both lesions (combined CAM/ Pincer impingement). If a symptomatic CAM lesion is present and patients have failed at least 3 months of conservative treatment, an arthroscopic osteoplasty may be performed to “trim” down the extra bone. Hip arthroscopy is no indicated in patients with underlying hip arthritis. 1,2,3

    Answer C.

    References

    1. Horisberger M, Brunner A, Herzog RF. Arthroscopic treatment of femoroacetabular impingement of the hip: a new technique to access the joint. Clin Orthop Relat Res. 2010;468(1):182-190. doi:10.1007/s11999-009-1005-5
    2. Kweon C, Welton K, MD; Kelly BT, Larson C, Bedi A. Arthroscopic Treatment of Cam-Type Impingement of the Hip. JBJS Reviews – Review Article: 15 September 2015 – Volume 3 – Issue 9 – p. e3
    3. Femoroacetabular Impingement. http://www.orthobullets.com. Accessed on 4/21/21
  33. Question 33 of 64
    33. Question


    A 52-year-old male presents to the emergency room with severe right hip pain after falling while getting out of his pool. He is one month out from a total hip replacement with anterior approach. On physical exam he has significant pain with any motion of the right hip. Figure 1 shows an AP pelvis just after surgery last month. Figures 2 and 3 show AP and lateral x-rays taken in the ED. What is the best treatment option?

    Correct

    The patient has a periprosthetic hip fracture with a subsided loose femoral component. The most common reasons for revision surgery after a primary total hip replacement include wound-related complications (49%), periprosthetic fracture (25%) and pain (18%). Treatment of perioprosthetic fractures is determined by the fracture location. Proximal fractures (greater trochanter fractures) non-displaced can be treated non-operatively with protected weight bearing and avoiding hip abduction. Displaced greater trochanter fractures are often treated with a trochanter plate with or without cerclage cables. Fractures at the midpoint of the prosthesis that extend into the diaphysis (like this patient) often cause the femoral prosthesis to subside and become unstable. These fractures require explant of the loose prosthesis with revision of the femoral component to a longer cement-less stem that bypasses the fracture line. Cerclage wires are often used proximally to prevent further displacement of the fracture fragments. Fractures that occur distal to a well fixed femoral prosthesis can be fixed with open reduction and internal fixation with a plate. 1,2

    Answer C.

    References

    1.Ledford, Cameron K. MD; Perry, Kevin I. MD; Hanssen, Arlen D. MD; Abdel, Matthew P. MD What Are the Contemporary Etiologies for Revision Surgery and Revision After Primary, Noncemented Total Hip Arthroplasty?, Journal of the American Academy of Orthopaedic Surgeons: December 15, 2019 – Volume 27 – Issue 24 – p 933-938

    1. Marsland D, Mears SC. A review of periprosthetic femoral fractures associated with total hip arthroplasty. Geriatric orthopaedic surgery & rehabilitation. 2012 Sep;3(3):107-20.

     

    Incorrect

    The patient has a periprosthetic hip fracture with a subsided loose femoral component. The most common reasons for revision surgery after a primary total hip replacement include wound-related complications (49%), periprosthetic fracture (25%) and pain (18%). Treatment of perioprosthetic fractures is determined by the fracture location. Proximal fractures (greater trochanter fractures) non-displaced can be treated non-operatively with protected weight bearing and avoiding hip abduction. Displaced greater trochanter fractures are often treated with a trochanter plate with or without cerclage cables. Fractures at the midpoint of the prosthesis that extend into the diaphysis (like this patient) often cause the femoral prosthesis to subside and become unstable. These fractures require explant of the loose prosthesis with revision of the femoral component to a longer cement-less stem that bypasses the fracture line. Cerclage wires are often used proximally to prevent further displacement of the fracture fragments. Fractures that occur distal to a well fixed femoral prosthesis can be fixed with open reduction and internal fixation with a plate. 1,2

    Answer C.

    References

    1.Ledford, Cameron K. MD; Perry, Kevin I. MD; Hanssen, Arlen D. MD; Abdel, Matthew P. MD What Are the Contemporary Etiologies for Revision Surgery and Revision After Primary, Noncemented Total Hip Arthroplasty?, Journal of the American Academy of Orthopaedic Surgeons: December 15, 2019 – Volume 27 – Issue 24 – p 933-938

    1. Marsland D, Mears SC. A review of periprosthetic femoral fractures associated with total hip arthroplasty. Geriatric orthopaedic surgery & rehabilitation. 2012 Sep;3(3):107-20.

     

  34. Question 34 of 64
    34. Question

    A 65-year-old female presents to your office with knee stiffness 10 weeks after a right total knee arthroplasty (TKA). Despite physical therapy twice a week since surgery, she is still unable to bend the knee past 70 degrees of flexion. She has full extension and there is no evidence of malposition of the implants on radiographs. She has no knee effusion or signs of infection. Which statement is true regarding this patients knee stiffness (arthrofibrosis) after TKA?

    Correct

    Most patients who have a total knee replacement (TKA) regain full extension and a maximal level of flexion that can be achieved. Knee stiffness after a TKA can be debilitating and is a common reason for a poor outcome. Physical therapy and patient motivation are critical components of regaining motion after surgery. The importance of regaining full knee flexion can be appreciated in many activities of daily living. The knee requires 83 degrees of flexion to climb stairs foot over foot, 93 degrees to sit in a chair without using hands, and 106 degrees for tying shoes while seated. For patients that develop stiffness within 2-3 months of TKA, manipulation under anesthesia (MUA) is the best treatment option to regain motion. Patient risk factors for knee stiffness after TKA requiring MUA includes younger patient age, history of smoking, and history of ipsilateral knee surgery. Criteria for MUA after TKA varies widely between surgeons. One common protocol includes MUA if patients fail to achieve 75 degrees of flexion 3 weeks postoperatively or 95 degrees at 3 months postoperatively. The average gain in knee flexion after MUA is 30 degrees. MUA performed within 12 weeks postoperatively is associated with increased gains in knee flexion when compared to MUA performed after 3 months post op. For this reason, MUA is generally performed before the 3-month post op timeframe if necessary. 1,2

    Answer B.

    References

    1. Issa K, Banerjee S, Kester MA, Khanuja HS, Delanois RE, Mont MA. The effect of timing of manipulation under anesthesia to improve range of motion and functional outcomes following total knee arthroplasty. JBJS. 2014 Aug 20;96(16):1349-57.
    2. Newman ET, Herschmiller TA, Attarian DE, Vail TP, Bolognesi MP, Wellman SS. Risk factors, outcomes, and timing of manipulation under anesthesia after total knee arthroplasty. The Journal of arthroplasty. 2018 Jan 1;33(1):245-9.
    Incorrect

    Most patients who have a total knee replacement (TKA) regain full extension and a maximal level of flexion that can be achieved. Knee stiffness after a TKA can be debilitating and is a common reason for a poor outcome. Physical therapy and patient motivation are critical components of regaining motion after surgery. The importance of regaining full knee flexion can be appreciated in many activities of daily living. The knee requires 83 degrees of flexion to climb stairs foot over foot, 93 degrees to sit in a chair without using hands, and 106 degrees for tying shoes while seated. For patients that develop stiffness within 2-3 months of TKA, manipulation under anesthesia (MUA) is the best treatment option to regain motion. Patient risk factors for knee stiffness after TKA requiring MUA includes younger patient age, history of smoking, and history of ipsilateral knee surgery. Criteria for MUA after TKA varies widely between surgeons. One common protocol includes MUA if patients fail to achieve 75 degrees of flexion 3 weeks postoperatively or 95 degrees at 3 months postoperatively. The average gain in knee flexion after MUA is 30 degrees. MUA performed within 12 weeks postoperatively is associated with increased gains in knee flexion when compared to MUA performed after 3 months post op. For this reason, MUA is generally performed before the 3-month post op timeframe if necessary. 1,2

    Answer B.

    References

    1. Issa K, Banerjee S, Kester MA, Khanuja HS, Delanois RE, Mont MA. The effect of timing of manipulation under anesthesia to improve range of motion and functional outcomes following total knee arthroplasty. JBJS. 2014 Aug 20;96(16):1349-57.
    2. Newman ET, Herschmiller TA, Attarian DE, Vail TP, Bolognesi MP, Wellman SS. Risk factors, outcomes, and timing of manipulation under anesthesia after total knee arthroplasty. The Journal of arthroplasty. 2018 Jan 1;33(1):245-9.
  35. Question 35 of 64
    35. Question


    A 22-year-old male presents to the office with lateral knee pain for 2 months. He is an avid runner and usually runs 4-5 miles every few days. The lateral knee pain has started to increase and he now is unable to run more than a mile. Radiographs of the right knee are unremarkable. Coronal MRI (figure 1) shows mild inflammation of the iliotibial band bursa at the lateral knee. Which physical exam test would be positive in this patient?

    Correct

    The iliotibial band originates from the level of the greater trochanter as a continuation of the tensor fasciae latae and gluteus muscles. The dense fibrous tissue then travels distally along the lateral thigh where it inserts at Gerdy’s tubercle at the anterolateral aspect of the proximal tibia. IT band syndrome (ITBS) is very common in runners as repetitive knee flexion can cause friction as the IT band slides over the lateral femoral condyle. Downhill running and running in the same direction continuously on a track can increase the likelihood of developing ITBS. The classic presentation includes lateral knee pain in a runner that increases proportionally with frequency and intensity of running. Pain to palpation is often over the IT band distally or around 3 cm proximal to the lateral joint line. The Ober test is the most common provocative test performed to determine ITB tightness. The test involves lying the patient on the unaffected side, then abducting and extending the affected leg, followed by slowly lowering the leg down to the table. The test is positive if the leg stays in the air and must be forced down to the table. The mainstay of treatment is nonoperative with a period of rest, NSAIDS, and physical therapy. A local corticosteroid injection can be beneficial for diagnostic and therapeutic purposes. Physical therapy can work on stretching the IT band and running mechanics. 1,2

    Answer D.

    References

    1. Strauss, Eric J. MD; Kim, Suezie MD; Calcei, Jacob G.; Park, Daniel PT, DPT Iliotibial Band Syndrome: Evaluation and Management, American Academy of Orthopaedic Surgeon: December 2011 – Volume 19 – Issue 12 – p 728-736
    2. Fredericson M, Wolf C. Iliotibial band syndrome in runners. Sports Medicine. 2005.May;35(5):451-9.

     

     

    Incorrect

    The iliotibial band originates from the level of the greater trochanter as a continuation of the tensor fasciae latae and gluteus muscles. The dense fibrous tissue then travels distally along the lateral thigh where it inserts at Gerdy’s tubercle at the anterolateral aspect of the proximal tibia. IT band syndrome (ITBS) is very common in runners as repetitive knee flexion can cause friction as the IT band slides over the lateral femoral condyle. Downhill running and running in the same direction continuously on a track can increase the likelihood of developing ITBS. The classic presentation includes lateral knee pain in a runner that increases proportionally with frequency and intensity of running. Pain to palpation is often over the IT band distally or around 3 cm proximal to the lateral joint line. The Ober test is the most common provocative test performed to determine ITB tightness. The test involves lying the patient on the unaffected side, then abducting and extending the affected leg, followed by slowly lowering the leg down to the table. The test is positive if the leg stays in the air and must be forced down to the table. The mainstay of treatment is nonoperative with a period of rest, NSAIDS, and physical therapy. A local corticosteroid injection can be beneficial for diagnostic and therapeutic purposes. Physical therapy can work on stretching the IT band and running mechanics. 1,2

    Answer D.

    References

    1. Strauss, Eric J. MD; Kim, Suezie MD; Calcei, Jacob G.; Park, Daniel PT, DPT Iliotibial Band Syndrome: Evaluation and Management, American Academy of Orthopaedic Surgeon: December 2011 – Volume 19 – Issue 12 – p 728-736
    2. Fredericson M, Wolf C. Iliotibial band syndrome in runners. Sports Medicine. 2005.May;35(5):451-9.

     

     

  36. Question 36 of 64
    36. Question


    A 32-year-old women presents to your office with right knee pain after a soccer injury two weeks ago. She tripped while playing and landed on her flexed right knee. She was unable to continue playing after the fall. She has tried running over the last week and the knee continues to feel unstable. On physical examination she has a small effusion with a grade 2 posterior drawer. Sagittal MRI (figure 1) shows an isolated posterior cruciate ligament (PCL) tear. What is the best treatment option?

    Correct

    Posterior cruciate ligament (PCL) tears are an uncommon injury as they occur much less often than anterior cruciate ligament (ACL) injuries. The PCL’s function is to limit posterior translation of the tibia in relation to the femur, which is opposite to the ACL. The mechanism injury causing a PCL injury is a posterior directed force to the proximal tibia with the knee in flexion. A common history is a motor vehicle accident when the proximal tibia hits the dashboard or the tibia is struck during athletic activities. Symptoms of a PCL injury can be vague with less swelling and pain compared to ACL injuries. Activities that require more knee bending and squatting tend to be more symptomatic. The posterior drawer test is the best test to diagnose a PCL tear and determine the degree of instability based on the amount of tibial translation. MRI is the diagnostic study of choice when a PCL injury is suspected. Grade 1 and 2 (<6mm and <10mm of translation, respectively) can be treated non-operatively. Physical therapy is initiated to regain motion and strength with an expected return to sports in 4-6 weeks. Grade 3 treatment (>10mm of translation) is more controversial. A higher degree of instability is more likely to result in recurrent symptoms. Non-operative treatment is a reasonable first step in treatment, combined with a PCL brace when returning to sports. If knee instability and symptoms persist past 3-6 months than a PCL reconstruction is the surgical treatment of choice. 1,2

    Answer C

    References

    1. Bedi, Asheesh MD; Musahl, Volker MD; Cowan, James B. MD Management of Posterior Cruciate Ligament Injuries, Journal of the American Academy of Orthopaedic Surgeons: May 2016 – Volume 24 – Issue 5 – p 277-289
    2. Cosgarea, Andrew J. MD; Jay, Peter R. MD Posterior Cruciate Ligament Injuries: Evaluation and Management, Journal of the American Academy of Orthopaedic Surgeons: September 2001 – Volume 9 – Issue 5 – p 297-307

     

     

    Incorrect

    Posterior cruciate ligament (PCL) tears are an uncommon injury as they occur much less often than anterior cruciate ligament (ACL) injuries. The PCL’s function is to limit posterior translation of the tibia in relation to the femur, which is opposite to the ACL. The mechanism injury causing a PCL injury is a posterior directed force to the proximal tibia with the knee in flexion. A common history is a motor vehicle accident when the proximal tibia hits the dashboard or the tibia is struck during athletic activities. Symptoms of a PCL injury can be vague with less swelling and pain compared to ACL injuries. Activities that require more knee bending and squatting tend to be more symptomatic. The posterior drawer test is the best test to diagnose a PCL tear and determine the degree of instability based on the amount of tibial translation. MRI is the diagnostic study of choice when a PCL injury is suspected. Grade 1 and 2 (<6mm and <10mm of translation, respectively) can be treated non-operatively. Physical therapy is initiated to regain motion and strength with an expected return to sports in 4-6 weeks. Grade 3 treatment (>10mm of translation) is more controversial. A higher degree of instability is more likely to result in recurrent symptoms. Non-operative treatment is a reasonable first step in treatment, combined with a PCL brace when returning to sports. If knee instability and symptoms persist past 3-6 months than a PCL reconstruction is the surgical treatment of choice. 1,2

    Answer C

    References

    1. Bedi, Asheesh MD; Musahl, Volker MD; Cowan, James B. MD Management of Posterior Cruciate Ligament Injuries, Journal of the American Academy of Orthopaedic Surgeons: May 2016 – Volume 24 – Issue 5 – p 277-289
    2. Cosgarea, Andrew J. MD; Jay, Peter R. MD Posterior Cruciate Ligament Injuries: Evaluation and Management, Journal of the American Academy of Orthopaedic Surgeons: September 2001 – Volume 9 – Issue 5 – p 297-307

     

     

  37. Question 37 of 64
    37. Question


    A 34-year-old female presents to the office with right knee pain after an injury two weeks ago. She was pushing a wheel barrel when she planted the right knee and twisted to turn. She had immediate sharp pain and had difficulty walking after the injury. On physical exam she has a mild effusion and sharp pain to palpation over the posterior medial joint line. MRI scan shows a posterior horn medial meniscal root tear, with mild extrusion. The articular cartilage remains intact. What is the best treatment option for this patient?

    Correct

    Meniscus tears are one of the most common injuries seen in orthopedic practice. The most common presentation of a meniscus tear includes sharp knee pain with twisting or squatting, knee swelling, and pain to palpation over the affected joint line. Meniscus tears can occur in many different orientations including radial, vertical, bucket handle, horizontal, complex, and root tears. Root tears can be the most difficult to diagnose and may not be as obvious on MRI as other types of tears. The medial and lateral meniscus roots anchor the anterior and posterior meniscal horns to the tibia. A tear of the meniscal root decreases the shock absorption capabilities of the meniscus leading to increased contact pressure on articular cartilage. Meniscal root tears can occur in active adults from a traumatic type injury or, more commonly, a degenerative type tear from chronic stresses to the knee. A posterior medial root tear is the most common location. The diagnostic study of choice for identifying meniscal root tears is an MRI, although identifying these tears can be difficult. One study found that only 33% of root tears were identified on MRI despite being read by fellowship-trained musculoskeletal radiologists. The diagnosis is made with a careful physical exam and patient history, in conjunction with MRI findings. Treatment involves surgical repair of the meniscal root to its native attachment site to prevent increases stresses on the articular cartilage and decrease the rate of progressive arthritis. Patients with existing arthritis at the time of injury are better candidates for meniscectomy. 1,2

    Answer C.

     

    References

    1. Krych, Aaron J. MD; Hevesi, Mario MD; Leland, Devin P. BS; Stuart, Michael J. MD Meniscal Root Injuries, Journal of the American Academy of Orthopaedic Surgeons: June 15, 2020 – Volume 28 – Issue 12 – p 491-499.
    2. Pache S, Aman ZS, Kennedy M, et al. Meniscal Root Tears: Current Concepts Review. Arch Bone Jt Surg. 2018;6(4):250-259.
    Incorrect

    Meniscus tears are one of the most common injuries seen in orthopedic practice. The most common presentation of a meniscus tear includes sharp knee pain with twisting or squatting, knee swelling, and pain to palpation over the affected joint line. Meniscus tears can occur in many different orientations including radial, vertical, bucket handle, horizontal, complex, and root tears. Root tears can be the most difficult to diagnose and may not be as obvious on MRI as other types of tears. The medial and lateral meniscus roots anchor the anterior and posterior meniscal horns to the tibia. A tear of the meniscal root decreases the shock absorption capabilities of the meniscus leading to increased contact pressure on articular cartilage. Meniscal root tears can occur in active adults from a traumatic type injury or, more commonly, a degenerative type tear from chronic stresses to the knee. A posterior medial root tear is the most common location. The diagnostic study of choice for identifying meniscal root tears is an MRI, although identifying these tears can be difficult. One study found that only 33% of root tears were identified on MRI despite being read by fellowship-trained musculoskeletal radiologists. The diagnosis is made with a careful physical exam and patient history, in conjunction with MRI findings. Treatment involves surgical repair of the meniscal root to its native attachment site to prevent increases stresses on the articular cartilage and decrease the rate of progressive arthritis. Patients with existing arthritis at the time of injury are better candidates for meniscectomy. 1,2

    Answer C.

     

    References

    1. Krych, Aaron J. MD; Hevesi, Mario MD; Leland, Devin P. BS; Stuart, Michael J. MD Meniscal Root Injuries, Journal of the American Academy of Orthopaedic Surgeons: June 15, 2020 – Volume 28 – Issue 12 – p 491-499.
    2. Pache S, Aman ZS, Kennedy M, et al. Meniscal Root Tears: Current Concepts Review. Arch Bone Jt Surg. 2018;6(4):250-259.
  38. Question 38 of 64
    38. Question


    A 57-year-old female presents to your office with 8 months of posterior right buttock pain. She denies a known injury or precipitating event. She has been seen at an orthopedic clinic at least 3-4 times for this pain where trochanteric bursa injections were performed with no relief. She also had an MRI of her lumbar spine which was unremarkable for any source of buttock and radicular pain. The pain feels deep into the posterior buttock and doesn’t radiate down the right leg. Axial MRI image shows an abnormal quadratus femoris muscle with edema (marked with white arrow) and narrowing of the ischiofemoral and quadratus femoris space. On physical exam the pain is worse with extension, adduction and external rotation of the right hip. What is the most likely diagnosis?

    Correct

    Posterior hip pain can be a difficult problem to diagnose and treat. Common sources of radiating pain to the posterior buttock, such as the lumbar spine and hip joint, must be ruled out before further work-up is performed. Often this means radiographs, MRI of the spine, and an intra-articular hip injection to exclude these areas as source. An understanding of the complex anatomy of the posterior hip and a careful physical exam also helps narrow the differential. Pain to palpation laterally over the greater trochanter is typically trochanteric bursitis, pain deep in the buttocks directly posterior to hip joint may include piriformis syndrome, ischiofemoral impingement, and obturator internus tears. Sacroiliac pain is usually broader over the entire posterior pelvis and buttock and may radiate down the leg much like sciatica. Radiographs and CT are useful for diagnosing hip and spine pathology however they are not useful for soft tissue pain. MRI is an essential modality to evaluate extra-articular soft tissue pain around the posterior hip. Guided injections offer the best diagnostic and therapeutic treatment option once a source of pain is suspected clinically and on MRI. For this patient, ischiofemoral impingement was suspected based on her MRI findings. Ischiofemoral impingement is a condition where the quadratus femoris is compressed directly between the lesser trochanter and the ischium. Symptoms may be reproduced by a combination of extension, adduction and external rotation of the hip. 1,2

    Answer A.

    References

    1. Gómez-Hoyos, Juan MD; Martin, Robroy L. PhD; Martin, Hal David DO Current Concepts Review: Evaluation and Management of Posterior Hip Pain, Journal of the American Academy of Orthopaedic Surgeons: September 1, 2018 – Volume 26 – Issue 17 – p 597-609
    2. Stafford GH, Villar RN. Ischiofemoral impingement. The Journal of bone and joint surgery. British volume. 2011 Oct;93(10):1300-2.
    Incorrect

    Posterior hip pain can be a difficult problem to diagnose and treat. Common sources of radiating pain to the posterior buttock, such as the lumbar spine and hip joint, must be ruled out before further work-up is performed. Often this means radiographs, MRI of the spine, and an intra-articular hip injection to exclude these areas as source. An understanding of the complex anatomy of the posterior hip and a careful physical exam also helps narrow the differential. Pain to palpation laterally over the greater trochanter is typically trochanteric bursitis, pain deep in the buttocks directly posterior to hip joint may include piriformis syndrome, ischiofemoral impingement, and obturator internus tears. Sacroiliac pain is usually broader over the entire posterior pelvis and buttock and may radiate down the leg much like sciatica. Radiographs and CT are useful for diagnosing hip and spine pathology however they are not useful for soft tissue pain. MRI is an essential modality to evaluate extra-articular soft tissue pain around the posterior hip. Guided injections offer the best diagnostic and therapeutic treatment option once a source of pain is suspected clinically and on MRI. For this patient, ischiofemoral impingement was suspected based on her MRI findings. Ischiofemoral impingement is a condition where the quadratus femoris is compressed directly between the lesser trochanter and the ischium. Symptoms may be reproduced by a combination of extension, adduction and external rotation of the hip. 1,2

    Answer A.

    References

    1. Gómez-Hoyos, Juan MD; Martin, Robroy L. PhD; Martin, Hal David DO Current Concepts Review: Evaluation and Management of Posterior Hip Pain, Journal of the American Academy of Orthopaedic Surgeons: September 1, 2018 – Volume 26 – Issue 17 – p 597-609
    2. Stafford GH, Villar RN. Ischiofemoral impingement. The Journal of bone and joint surgery. British volume. 2011 Oct;93(10):1300-2.
  39. Question 39 of 64
    39. Question


    A 75-year-old female presents to emergency room with severe right shoulder pain after a fall earlier in the day. AP x-ray of the right shoulder shows a comminuted proximal humerus fracture (figure 1). She is right hand dominant and continues to enjoy an active lifestyle. What is the best treatment option for this patient who wishes for the earliest functional recovery?

    Correct

    Proximal humerus fractures are a common type of fracture seen in the emergency room and most can be treated non-operatively. Complex comminuted fractures of the proximal humerus are much less common, although the incidence increases with advanced age, female gender, and osteoporotic bone. In elderly patients, proximal humerus fractures are the 3rd most common type of fracture, behind hip and distal radius fractures. Poor bone quality makes fixing displaced and comminuted fractures more difficult. Complex fractures are prone to non-union and osteonecrosis after operative fixation. A reverse shoulder arthroplasty is an increasingly used option for proximal humerus fractures that are deemed irreparable and have a high likelihood of osteonecrosis. The most common indication for a reverse total shoulder arthroplasty is a displaced 4-part proximal humerus fracture in a patient over 70 years of age. A reverse total shoulder allows for a rapid recovery without relying on fracture healing. Pain relief and shoulder function are sustained over time after recovering from a reverse shoulder arthroplasty. Reverse total shoulder arthroplasty has less complications and better shoulder function compared to shoulder hemiathroplasty for displaced proximal humerus fractures. 1,2

    Answer D.

    References

    1. Jobin, Charles M. MD; Galdi, Balazs MD; Anakwenze, Oke A. MD; Ahmad, Christopher S. MD; Levine, William N. MD Reverse Shoulder Arthroplasty for the Management of Proximal Humerus Fractures, Journal of the American Academy of Orthopaedic Surgeons: March 2015 – Volume 23 – Issue 3 – p 190-201
    2. Gallinet D, Cazeneuve JF, Boyer E, Menu G, Obert L, Ohl X, Bonnevialle N, Valenti P, Boileau P. Reverse shoulder arthroplasty for recent proximal humerus fractures: outcomes in 422 cases. Orthopaedics & Traumatology: Surgery & Research. 2019 Sep 1;105(5):805-11.
    Incorrect

    Proximal humerus fractures are a common type of fracture seen in the emergency room and most can be treated non-operatively. Complex comminuted fractures of the proximal humerus are much less common, although the incidence increases with advanced age, female gender, and osteoporotic bone. In elderly patients, proximal humerus fractures are the 3rd most common type of fracture, behind hip and distal radius fractures. Poor bone quality makes fixing displaced and comminuted fractures more difficult. Complex fractures are prone to non-union and osteonecrosis after operative fixation. A reverse shoulder arthroplasty is an increasingly used option for proximal humerus fractures that are deemed irreparable and have a high likelihood of osteonecrosis. The most common indication for a reverse total shoulder arthroplasty is a displaced 4-part proximal humerus fracture in a patient over 70 years of age. A reverse total shoulder allows for a rapid recovery without relying on fracture healing. Pain relief and shoulder function are sustained over time after recovering from a reverse shoulder arthroplasty. Reverse total shoulder arthroplasty has less complications and better shoulder function compared to shoulder hemiathroplasty for displaced proximal humerus fractures. 1,2

    Answer D.

    References

    1. Jobin, Charles M. MD; Galdi, Balazs MD; Anakwenze, Oke A. MD; Ahmad, Christopher S. MD; Levine, William N. MD Reverse Shoulder Arthroplasty for the Management of Proximal Humerus Fractures, Journal of the American Academy of Orthopaedic Surgeons: March 2015 – Volume 23 – Issue 3 – p 190-201
    2. Gallinet D, Cazeneuve JF, Boyer E, Menu G, Obert L, Ohl X, Bonnevialle N, Valenti P, Boileau P. Reverse shoulder arthroplasty for recent proximal humerus fractures: outcomes in 422 cases. Orthopaedics & Traumatology: Surgery & Research. 2019 Sep 1;105(5):805-11.
  40. Question 40 of 64
    40. Question


    A 29-year-old male presents to your office with complaints of right shoulder pain after an injury during a mixed martial arts competition. During the match his arm was pinned backward and his right shoulder dislocated anteriorly. He was brought to the emergency room where the shoulder was reduced. Post reduction AP x-ray is shown in figure 1. The dislocation resulted in a large deformity to the humeral head which is identified with a white arrow in figure 1. What is the name of this deformity?

    Correct

    The patient has a large Hill-Sachs lesion from his anterior shoulder dislocation. A Hill-Sachs lesion is a compression fracture of the posterior lateral humeral head as it strikes the anterior glenoid during an anterior shoulder dislocation. The incidence of a Hill-Sachs lesion after a shoulder dislocation varies widely in the literature or between 40% and 90%. Lesions can vary in size with larger lesions being more clinically significant. Lesions involving >40% of the humeral surface are almost always associated with recurrent shoulder instability. An “engaging” lesion is one that engages the rim of the glenoid when the shoulder is in abduction and external rotation. Symptoms of an engaging lesion include “catching” when the patient abducts and externally rotates the arm such as in a throwing motion. Engaging Hill-Sachs lesions have a high rate of instability and surgical failure when treated with a Bankart repair alone. The most common treatment for an engaging Hill-Sachs lesion is an arthroscopic Bankart repair with a remplissage procedure. A remplissage procedure involves filling the bony defect with capsule and infraspinatus tissue to prevent engagement of the glenoid. 1,2

    Answer D.

    References

    1. Buza JA, Lyengar JJ, Anakwenze OA, Ahmad CS, Levine W. Arthroscopic Hill-Sachs Remplissage. A Systematic Review. The Journal of Bone & Joint Surgery – Scientific Articles: 02 April 2014 – Volume 96 – Issue 7 – p. 549-555
    2. Provencher, Matthew T. MD; Frank, Rachel M. MD; LeClere, Lance E. MD; Metzger, Paul D. MD; Ryu, J. J.; Bernhardson, Andrew MD; Romeo, Anthony A. MD The Hill-Sachs Lesion: Diagnosis, Classification, and Management, Journal of the American Academy of Orthopaedic Surgeons: April 2012 – Volume 20 – Issue 4
    Incorrect

    The patient has a large Hill-Sachs lesion from his anterior shoulder dislocation. A Hill-Sachs lesion is a compression fracture of the posterior lateral humeral head as it strikes the anterior glenoid during an anterior shoulder dislocation. The incidence of a Hill-Sachs lesion after a shoulder dislocation varies widely in the literature or between 40% and 90%. Lesions can vary in size with larger lesions being more clinically significant. Lesions involving >40% of the humeral surface are almost always associated with recurrent shoulder instability. An “engaging” lesion is one that engages the rim of the glenoid when the shoulder is in abduction and external rotation. Symptoms of an engaging lesion include “catching” when the patient abducts and externally rotates the arm such as in a throwing motion. Engaging Hill-Sachs lesions have a high rate of instability and surgical failure when treated with a Bankart repair alone. The most common treatment for an engaging Hill-Sachs lesion is an arthroscopic Bankart repair with a remplissage procedure. A remplissage procedure involves filling the bony defect with capsule and infraspinatus tissue to prevent engagement of the glenoid. 1,2

    Answer D.

    References

    1. Buza JA, Lyengar JJ, Anakwenze OA, Ahmad CS, Levine W. Arthroscopic Hill-Sachs Remplissage. A Systematic Review. The Journal of Bone & Joint Surgery – Scientific Articles: 02 April 2014 – Volume 96 – Issue 7 – p. 549-555
    2. Provencher, Matthew T. MD; Frank, Rachel M. MD; LeClere, Lance E. MD; Metzger, Paul D. MD; Ryu, J. J.; Bernhardson, Andrew MD; Romeo, Anthony A. MD The Hill-Sachs Lesion: Diagnosis, Classification, and Management, Journal of the American Academy of Orthopaedic Surgeons: April 2012 – Volume 20 – Issue 4
  41. Question 41 of 64
    41. Question


    A 21-year-old male presents to the emergency room with severe right ankle pain and deformity. He was jumping off a bank and fell awkwardly on the right ankle. On physical exam he has an obviously deformity to the ankle but his skin and neurovascular status are intact. Figure 1 shows an AP and lateral of the ankle after closed reduction. The x-rays appear to show an osteochondral lesion of the talus with a distal fibular fracture. Which fracture pattern below is most likely to sustain an osteochondral lesion after an ankle fracture?

    Correct

    Ankle fractures are common injuries seen in orthopedic practice. Unstable ankle fractures are frequently treated with open reduction and internal fixation to restore anatomical alignment of the ankle. One aspect of an ankle fracture that is often missed at the initial presentation is an osteochondral injury. Osteochondral injuries that occur with ankle fractures are known to cause significantly poorer results comparted to ankle fractures alone. Damage to the articular cartilage of the ankle can cause early ankle arthritis. Osteochondral injuries typically occur at the talar dome and can occur in up to 80% of ankle fractures. Aktas et al found that osteochondral injuries occurred in 15% of bimalleolar fractures, 40% of trimalleolar fractures, and 70% of distal fibular fractures. The theory being more stable injuries transmit the forces through the cartilage and less on bone. Kwok et al found that lesions were present in no patients with Weber A fractures, 26% of the Weber B fractures, 24% of the Weber C fractures, and 20% of isolated medial malleolar fracture cases. This patient underwent an ankle arthroscopy at the time of fracture fixation which revealed an osteochondral impact injury to the talar done. The osteochondral lesion was debrided prior to ankle fixation. 1,2

    Answer A.

    References

    1. Ackermann J, Fraser EJ, Murawski CD, Desai P, Vig K, Kennedy JG. Trends of concurrent ankle arthroscopy at the time of operative treatment of ankle fracture: a national database review. Foot & ankle specialist. 2016 Apr;9(2):107-12.
    2. Thordarson DB, Bains R, Shepherd LE. The role of ankle arthroscopy on the surgical management of ankle fractures. Foot & ankle international. 2001 Feb;22(2):123-5.
    Incorrect

    Ankle fractures are common injuries seen in orthopedic practice. Unstable ankle fractures are frequently treated with open reduction and internal fixation to restore anatomical alignment of the ankle. One aspect of an ankle fracture that is often missed at the initial presentation is an osteochondral injury. Osteochondral injuries that occur with ankle fractures are known to cause significantly poorer results comparted to ankle fractures alone. Damage to the articular cartilage of the ankle can cause early ankle arthritis. Osteochondral injuries typically occur at the talar dome and can occur in up to 80% of ankle fractures. Aktas et al found that osteochondral injuries occurred in 15% of bimalleolar fractures, 40% of trimalleolar fractures, and 70% of distal fibular fractures. The theory being more stable injuries transmit the forces through the cartilage and less on bone. Kwok et al found that lesions were present in no patients with Weber A fractures, 26% of the Weber B fractures, 24% of the Weber C fractures, and 20% of isolated medial malleolar fracture cases. This patient underwent an ankle arthroscopy at the time of fracture fixation which revealed an osteochondral impact injury to the talar done. The osteochondral lesion was debrided prior to ankle fixation. 1,2

    Answer A.

    References

    1. Ackermann J, Fraser EJ, Murawski CD, Desai P, Vig K, Kennedy JG. Trends of concurrent ankle arthroscopy at the time of operative treatment of ankle fracture: a national database review. Foot & ankle specialist. 2016 Apr;9(2):107-12.
    2. Thordarson DB, Bains R, Shepherd LE. The role of ankle arthroscopy on the surgical management of ankle fractures. Foot & ankle international. 2001 Feb;22(2):123-5.
  42. Question 42 of 64
    42. Question


    A 36-year-old right-hand dominant male presents with pain and deformity of his left upper chest for one week. He was bench pressing at the gym when he went to put the weight up and felt a popping sensation in the left armpit area. He had immediate pain and had to have the weight lifted from him. He felt as though someone had dropped a weight on him. Pain and spasm subsequently ensued in the chest and upper arm area. He was seen at urgent care and x-rays did not show any acute abnormalities. On physical exam there is obvious asymmetry to the chest muscles with the left side being larger than the right. There is extensive ecchymosis in the left upper arm and tenderness to palpation of the left chest and axilla. Axial MRI image is shown in figure 1. What is the next best step in treatment?

    Correct

    The pectoralis major is a broad fan shaped muscle that originates at clavicular and sternal heads. The pec major converges to a 5 cm flat tendon that inserts on the humerus at the lateral and distal aspect of the bicipital groove. The main function of the pec major is adduction and internal rotation of the humerus (bench press maneuver). A pec major tear is a rare injury that occurs almost exclusively in weight lifting males between the ages of 20 and 40 years old. Pec major tears are often associated with a popping sensation in the upper arm, localized ecchymosis, asymmetry of chest muscles and muscle weakness. The muscle deformity can be enhanced with resisted adduction as the torn muscle belly retracts medially. MRI is the study of choice to diagnose tears if there is any clinical suspicion. Most patients with a pec major tendon tear will elect for surgical repair instead of accepting a residual deformity to the chest wall and chronic chest weakness. Surgical repair is a very successful procedure that allows patients to return to a pre-injury level of strength and motion. Athletes can expect a return to full participation 3 months postoperatively. 1,2

    Answer C.

    References

    1. Nute DW, Kusnezov N, Dunn JC, Waterman BR. Return to Function, Complication, and Reoperation Rates Following Primary Pectoralis Major Tendon Repair in Military Service Members. The Journal of Bone and Joint Surgery – Scientific Articles: 04 January 2017 – Volume 99 – Issue 1 – p. 25-32
    2. Petilon J, Carr DR, Sekiya JK, Unger DV.Pectoralis Major Muscle Injuries: Evaluation and Management. J Am Acad Orthop Surg 2005;13:59-68
    Incorrect

    The pectoralis major is a broad fan shaped muscle that originates at clavicular and sternal heads. The pec major converges to a 5 cm flat tendon that inserts on the humerus at the lateral and distal aspect of the bicipital groove. The main function of the pec major is adduction and internal rotation of the humerus (bench press maneuver). A pec major tear is a rare injury that occurs almost exclusively in weight lifting males between the ages of 20 and 40 years old. Pec major tears are often associated with a popping sensation in the upper arm, localized ecchymosis, asymmetry of chest muscles and muscle weakness. The muscle deformity can be enhanced with resisted adduction as the torn muscle belly retracts medially. MRI is the study of choice to diagnose tears if there is any clinical suspicion. Most patients with a pec major tendon tear will elect for surgical repair instead of accepting a residual deformity to the chest wall and chronic chest weakness. Surgical repair is a very successful procedure that allows patients to return to a pre-injury level of strength and motion. Athletes can expect a return to full participation 3 months postoperatively. 1,2

    Answer C.

    References

    1. Nute DW, Kusnezov N, Dunn JC, Waterman BR. Return to Function, Complication, and Reoperation Rates Following Primary Pectoralis Major Tendon Repair in Military Service Members. The Journal of Bone and Joint Surgery – Scientific Articles: 04 January 2017 – Volume 99 – Issue 1 – p. 25-32
    2. Petilon J, Carr DR, Sekiya JK, Unger DV.Pectoralis Major Muscle Injuries: Evaluation and Management. J Am Acad Orthop Surg 2005;13:59-68
  43. Question 43 of 64
    43. Question


    A 43-year old male presents to the office with left shoulder pain after doing a bench press 2 days ago. He had immediate pain, felt a “pop” in the shoulder, and was unable to lift the bar any higher. On physical exam he has pain to palpation over the anterior shoulder with ecchymosis into the axilla. He has weakness with adduction of the humerus on the left comparted to the right side. He has no asymmetry of the chest wall bilaterally. MRI of the left chest wall demonstrates a full-thickness rupture of the pectoralis major tendon distally with 4 cm of retraction (figure 1). Which statement is true regarding surgical fixation?

    Correct

    The pectoralis major (PM) muscle is located on the anterior chest wall. The muscle originates from the sternum and adjacent first through six ribs and inserts on the proximal humerus just lateral and distal to the bicipital groove. The primary function of the PM is forward flexion, internal rotation, and adduction of the humerus. The most common mechanism of injury is a bench press maneuver when the shoulder is extended and externally rotated. During a PM injury, patients will often describe a popping sensation followed by pain and weakness of the arm. Initially the physical exam may reveal ecchymosis in the axilla without any obvious deformity to the pectoralis contour. Positioning the patients arm in abduction and external rotation can accentuate the pectoralis deformity to become visible. MRI is the gold standard study to confirm the diagnosis of a PM tear. MRI of the chest wall (not MRI of the shoulder), performed in a prone position to reduce respiratory motion interference, is recommended. Operative treatment is recommended in active patients (non-elderly) to preserve shoulder adduction strength. The PM is generally repaired back down to bone using suture anchors within 2-4 weeks of injury. Surgical repair after 8 weeks leads to worse outcomes as the tendon retracts and becomes harder to repair. 1,2

    Answer C.

    References

    1. Kowalczuk M, Elmaraghy A. Pectoralis Major Rupture: Evaluation and Management, Journal of the American Academy of Orthopaedic Surgeons: January 12, 2022 – Volume – Issue – 10.5435/JAAOS-D-21-00541
    2. Brown SM, Cole WW, Provencher MT, Mary MK. Pectoralis Major Injuries Presentation, Diagnosis, and Management JBJS Reviews – Review Articles: 06 May 2021 – Volume 9 – Issue 5 – p. e20.00097

     

    Incorrect

    The pectoralis major (PM) muscle is located on the anterior chest wall. The muscle originates from the sternum and adjacent first through six ribs and inserts on the proximal humerus just lateral and distal to the bicipital groove. The primary function of the PM is forward flexion, internal rotation, and adduction of the humerus. The most common mechanism of injury is a bench press maneuver when the shoulder is extended and externally rotated. During a PM injury, patients will often describe a popping sensation followed by pain and weakness of the arm. Initially the physical exam may reveal ecchymosis in the axilla without any obvious deformity to the pectoralis contour. Positioning the patients arm in abduction and external rotation can accentuate the pectoralis deformity to become visible. MRI is the gold standard study to confirm the diagnosis of a PM tear. MRI of the chest wall (not MRI of the shoulder), performed in a prone position to reduce respiratory motion interference, is recommended. Operative treatment is recommended in active patients (non-elderly) to preserve shoulder adduction strength. The PM is generally repaired back down to bone using suture anchors within 2-4 weeks of injury. Surgical repair after 8 weeks leads to worse outcomes as the tendon retracts and becomes harder to repair. 1,2

    Answer C.

    References

    1. Kowalczuk M, Elmaraghy A. Pectoralis Major Rupture: Evaluation and Management, Journal of the American Academy of Orthopaedic Surgeons: January 12, 2022 – Volume – Issue – 10.5435/JAAOS-D-21-00541
    2. Brown SM, Cole WW, Provencher MT, Mary MK. Pectoralis Major Injuries Presentation, Diagnosis, and Management JBJS Reviews – Review Articles: 06 May 2021 – Volume 9 – Issue 5 – p. e20.00097

     

  44. Question 44 of 64
    44. Question


    A 62-year-old female presents to your office with left shoulder pain after a fall the night before. She tripped and fell with her arm extended. She had immediate pain and was unable to lift her arm after. Figure 1 shows an AP radiograph of the left shoulder. Which statement is true regarding this patients fracture?

    Correct

    The great tuberosity is a fused apophysis of the posterior lateral proximal humerus that acts as the attachment site for the rotator cuff. An isolated greater tuberosity is a relatively uncommon injury that can occur during a fall with an impact to the shoulder. An avulsion fracture usually occurs when the rotator cuff is suddenly contracted pulling a piece of bone off the greater tuberosity. A larger vertical type fracture is thought to occur during a shoulder instability episode. If the fracture displaces more than 5mm, the fragment may cause mechanical impingement at the subacromial space with shoulder abduction and forward elevation. Patients with less than 5mm of fracture displacement can be treated conservatively with no arm abduction or forward elevation for 4-6 weeks. If the fracture displacement is unclear on initial radiographs, CT may be warranted to better determine displacement. Rotator cuff injuries generally don’t occur with greater tuberosity fractures (16% incidence of concomitant rotator cuff tears), but an MRI may be warranted if weakness and shoulder pain continue after the greater tuberosity fracture has healed. 1,2

    Answer A

    References

    1. White EA, Skalski MR, Patel DB, Gross JS, Tomasian A, Heckmann N, Matcuk GR. Isolated greater tuberosity fractures of the proximal humerus: anatomy, injury patterns, multimodality imaging, and approach to management. Emergency Radiology. 2018 Jun;25(3):235-46.
    2. Rouleau DM, Mutch J, Laflamme GY. Surgical treatment of displaced greater tuberosity fractures of the humerus. JAAOS-Journal of the American Academy of Orthopaedic Surgeons. 2016 Jan 1;24(1):46-56.
    Incorrect

    The great tuberosity is a fused apophysis of the posterior lateral proximal humerus that acts as the attachment site for the rotator cuff. An isolated greater tuberosity is a relatively uncommon injury that can occur during a fall with an impact to the shoulder. An avulsion fracture usually occurs when the rotator cuff is suddenly contracted pulling a piece of bone off the greater tuberosity. A larger vertical type fracture is thought to occur during a shoulder instability episode. If the fracture displaces more than 5mm, the fragment may cause mechanical impingement at the subacromial space with shoulder abduction and forward elevation. Patients with less than 5mm of fracture displacement can be treated conservatively with no arm abduction or forward elevation for 4-6 weeks. If the fracture displacement is unclear on initial radiographs, CT may be warranted to better determine displacement. Rotator cuff injuries generally don’t occur with greater tuberosity fractures (16% incidence of concomitant rotator cuff tears), but an MRI may be warranted if weakness and shoulder pain continue after the greater tuberosity fracture has healed. 1,2

    Answer A

    References

    1. White EA, Skalski MR, Patel DB, Gross JS, Tomasian A, Heckmann N, Matcuk GR. Isolated greater tuberosity fractures of the proximal humerus: anatomy, injury patterns, multimodality imaging, and approach to management. Emergency Radiology. 2018 Jun;25(3):235-46.
    2. Rouleau DM, Mutch J, Laflamme GY. Surgical treatment of displaced greater tuberosity fractures of the humerus. JAAOS-Journal of the American Academy of Orthopaedic Surgeons. 2016 Jan 1;24(1):46-56.
  45. Question 45 of 64
    45. Question


    A 23-year-old male presents to the office with left thumb pain after falling down a ladder two days ago. The thumb jammed on the side of the ladder and he had immediate pain and swelling at the interphalangeal joint of the left thumb. AP and lateral x-rays of the left thumb (figures 1 and 2) show a displaced intra-articular fracture of the distal aspect of the proximal phalanx. The goal of surgical fixation for this fracture would primarily focus on what?

    Correct

    Phalangeal fractures are one of the most common types of fractures seen in the upper extremity. The most common mechanisms of injury include a direct blow or accidental fall. Failure to recognize and appropriately treat proximal phalanx fractures can lead to significant hand disability. Displaced intra-articular fractures can lead to problems with flexion and extension deformities, joint instability, joint stiffness, and eventual post-traumatic arthritis. Prolonged non-operative treatment with immobilization often leads to a very stiff and painful finger, which can often times be permanent. Surgical fixation is also challenging as the fracture fragments are often small making it difficult to achieve an anatomical reduction and fixation. The most common methods of surgical fixation include K-wire or screw and plate fixation. Plate fixation is generally used for larger fracture patterns as adequate bone stock is required for stable fixation with a screw. The primarily goal of fixing a displaced intra-articular fracture is to restore a congruent joint space to mitigate post-traumatic arthritis. The joint must be immobilized for 4 weeks post-op after fracture fixation to allow for fracture healing. 1,2

    Answer B

    References

    1. Boyer JS, London DA, Stepan JG, Goldfarb CA. Pediatric proximal phalanx fractures: outcomes and complications after the surgical treatment of displaced fractures. Journal of pediatric orthopedics. 2015 Apr;35(3):219.
    2. Hornbach EE, Cohen MS: Closed reduction and percutaneous pinning of fractures of the proximal phalanx. J Hand Surg Br 2001;26:45-49..
    Incorrect

    Phalangeal fractures are one of the most common types of fractures seen in the upper extremity. The most common mechanisms of injury include a direct blow or accidental fall. Failure to recognize and appropriately treat proximal phalanx fractures can lead to significant hand disability. Displaced intra-articular fractures can lead to problems with flexion and extension deformities, joint instability, joint stiffness, and eventual post-traumatic arthritis. Prolonged non-operative treatment with immobilization often leads to a very stiff and painful finger, which can often times be permanent. Surgical fixation is also challenging as the fracture fragments are often small making it difficult to achieve an anatomical reduction and fixation. The most common methods of surgical fixation include K-wire or screw and plate fixation. Plate fixation is generally used for larger fracture patterns as adequate bone stock is required for stable fixation with a screw. The primarily goal of fixing a displaced intra-articular fracture is to restore a congruent joint space to mitigate post-traumatic arthritis. The joint must be immobilized for 4 weeks post-op after fracture fixation to allow for fracture healing. 1,2

    Answer B

    References

    1. Boyer JS, London DA, Stepan JG, Goldfarb CA. Pediatric proximal phalanx fractures: outcomes and complications after the surgical treatment of displaced fractures. Journal of pediatric orthopedics. 2015 Apr;35(3):219.
    2. Hornbach EE, Cohen MS: Closed reduction and percutaneous pinning of fractures of the proximal phalanx. J Hand Surg Br 2001;26:45-49..
  46. Question 46 of 64
    46. Question


    You are rounding on a 66-year-old female who presented to the emergency room with a 5-day history of spontaneous wounds of bilateral feet and ankles. The wounds are increasing in pain and seem to be ulcerating fairly rapidly. The ulceration on the left foot is the largest and most painful. She was admitted to the hospital and has developed fever and skin necrosis on the left foot despite broad spectrum IV antibiotics. Figure 1 shows the left foot at initial presentation in the ED and figure 2 shows the foot 2 days later while in the hospital. The medical team suspects Pyoderma Gangrenosum. Which choice below is most helpful in confirming the diagnosis?

    Correct

    Pyoderma Gangrenosum (PG) is a rare painful inflammatory skin disease characterized by an initial area of erythema or pustule followed by progressive would ulceration. The ulceration can progress rapidly with a raised inflammatory boarder, a necrotic base, and purulent discharge. The most common location to occur is the lower extremity, but any part of the body can be affected. The ulceration may become malodorous due to a secondary infection. The cause of PG is thought to be autoimmune as there is a higher incidence in patients with inflammatory bowel disease, although up to 50% of cases are idiopathic. The diagnosis of PG is primarily made on clinic features as histopathological findings are non-specific. Neutrophilic proliferation is often seen in tissue samples. Skin biopsy should be performed to rule out other causes of ulceration including infection, vasculitis, and malignancy. The clinical course of PG varies greatly and is unpredictable. Lesions can heal and not re-occur, while other lesions can last for years or heal and return months to years later. Severe cases can progress to where surgical debridement, skin grafts, and even limb amputation may be necessary. Systemic corticosteroids are the initial treatment of choice to halt progression of the ulceration. Other immunosuppressive agents may be used along with antibiotics to treat secondary infection. Wound management is a critical part of treatment with wet to dry dressings and frequent wound checks. 1,2

    Answer A.

    References

    1. Callen JP. Pyoderma gangrenosum. The Lancet. 1998 Feb 21;351(9102):581-5.
    2. Wollina U. Pyoderma gangrenosum–a review. Orphanet Journal of Rare Diseases. 2007 Dec;2(1):1-8.

     

    Incorrect

    Pyoderma Gangrenosum (PG) is a rare painful inflammatory skin disease characterized by an initial area of erythema or pustule followed by progressive would ulceration. The ulceration can progress rapidly with a raised inflammatory boarder, a necrotic base, and purulent discharge. The most common location to occur is the lower extremity, but any part of the body can be affected. The ulceration may become malodorous due to a secondary infection. The cause of PG is thought to be autoimmune as there is a higher incidence in patients with inflammatory bowel disease, although up to 50% of cases are idiopathic. The diagnosis of PG is primarily made on clinic features as histopathological findings are non-specific. Neutrophilic proliferation is often seen in tissue samples. Skin biopsy should be performed to rule out other causes of ulceration including infection, vasculitis, and malignancy. The clinical course of PG varies greatly and is unpredictable. Lesions can heal and not re-occur, while other lesions can last for years or heal and return months to years later. Severe cases can progress to where surgical debridement, skin grafts, and even limb amputation may be necessary. Systemic corticosteroids are the initial treatment of choice to halt progression of the ulceration. Other immunosuppressive agents may be used along with antibiotics to treat secondary infection. Wound management is a critical part of treatment with wet to dry dressings and frequent wound checks. 1,2

    Answer A.

    References

    1. Callen JP. Pyoderma gangrenosum. The Lancet. 1998 Feb 21;351(9102):581-5.
    2. Wollina U. Pyoderma gangrenosum–a review. Orphanet Journal of Rare Diseases. 2007 Dec;2(1):1-8.

     

  47. Question 47 of 64
    47. Question


    An 11-year-old girl presents to your office with left knee pain for 2 months. The pain seems to be located at the front of the knee just past the patella on the lower leg. The pain is made worse with running and jumping. She has tried ice and anti-inflammatories which offered mild relief. On physical exam the patient has deformity or knee effusion. She has tenderness to palpation over the tibial tubercle on the left knee. AP and lateral x-rays are shown in figures 1 and 2. What is the next best step in treatment?

    Correct

    Osgood-Schlatter disease (OSD) is a common knee condition seen in active adolescence which results from traction apophysitis at the tibial tubercle. Symptoms start during a period of rapid skeletal growth while patients participate in athletic activities. The diagnosis is straightforward with pain to palpation over the tibial tubercle and pain that is made worse with running and jumping activities. Radiographs are usually normal but may show irregularity and fragmentation of the tibial tubercle. Advanced imaging with MRI and/or CT is not indicated to confirm the diagnosis. Treatment involves NSAIDS, ice, activity modification and quadriceps stretching. Patients may have to pause athletic activities for a short period if pain is significant, however most patients can play through mild discomfort. OSD can be a nagging type of condition that lasts up to 12-18 months but generally resolves when patients reach skeletal maturity. Maintaining quadriceps flexibility through sport activities is critical to managing symptoms. 1,2

    Answer A.

     

    1. Bloom OJ, Mackler L. What is the best treatment for Osgood-Schlatter disease?. Clinical Inquiries, 2004 (MU). 2004.
    2. Circi E, Atalay Y, Beyzadeoglu T. Treatment of Osgood–Schlatter disease: review of the literature. Musculoskeletal surgery. 2017 Dec;101(3):195-200.
    Incorrect

    Osgood-Schlatter disease (OSD) is a common knee condition seen in active adolescence which results from traction apophysitis at the tibial tubercle. Symptoms start during a period of rapid skeletal growth while patients participate in athletic activities. The diagnosis is straightforward with pain to palpation over the tibial tubercle and pain that is made worse with running and jumping activities. Radiographs are usually normal but may show irregularity and fragmentation of the tibial tubercle. Advanced imaging with MRI and/or CT is not indicated to confirm the diagnosis. Treatment involves NSAIDS, ice, activity modification and quadriceps stretching. Patients may have to pause athletic activities for a short period if pain is significant, however most patients can play through mild discomfort. OSD can be a nagging type of condition that lasts up to 12-18 months but generally resolves when patients reach skeletal maturity. Maintaining quadriceps flexibility through sport activities is critical to managing symptoms. 1,2

    Answer A.

     

    1. Bloom OJ, Mackler L. What is the best treatment for Osgood-Schlatter disease?. Clinical Inquiries, 2004 (MU). 2004.
    2. Circi E, Atalay Y, Beyzadeoglu T. Treatment of Osgood–Schlatter disease: review of the literature. Musculoskeletal surgery. 2017 Dec;101(3):195-200.
  48. Question 48 of 64
    48. Question


    A 14-year-old boy presents to the office accompanied by his parents with complaints of right lateral elbow pain for 6 months. He denies a known injury and the pain is getting worse over the last month since baseball season started. He is involved in gymnastics and baseball most months throughout the year. On physical exam the patient has tenderness to palpation over the radiocapitellar joint. Range of motion of the elbow is full and equal to the contralateral side. AP x-ray of the right elbow (figure 1) shows a focal radiolucent lesion of the capitellum with flattening of the articular surface. MRI of the elbow in figure 2 shows a cyst in the capitellum with a joint effusion and loose bodies. What is the best treatment option

    Correct

    Osteochondritis dissecans (OCD) most commonly occurs at the capitellum in adolescent athletes. Gymnastics and baseball have the highest incidence of OCD of the capitellum which is likely attributed to compression of the radiocapitellar joint during hand stands and overhead throwing. This patient is presenting with advanced OCD which is indicated by radiographic changes and the presence of loose bodies in the elbow joint. Unstable OCD lesions with loose bodies generally require surgery. An arthroscopic or open excision of the loose bodies must be performed to prevent mechanical symptoms and recurrent effusions. Fixation of unstable lesions or osteochondral fragment removal can be performed depending on the size and stability of the lesion. In general fragments consisting of 50% or more of the capitellum require fixation. Osteochondral autograft plugs are used for large defects to restore the articular surface. 1,2

    Answer C.

    References

    1. Baker III, Champ L., Anthony A. Romeo, and Champ L. Baker Jr. “Osteochondritis dissecans of the capitellum.” The American journal of sports medicine 38.9 (2010): 1917-1928.
    2. Ruchelsman DE, Hall MP, Youm T. Osteochondritis Dissecans of the Capitellum: Current Concepts. J Am Acad Orthop Surg 2010;18:

     

    Incorrect

    Osteochondritis dissecans (OCD) most commonly occurs at the capitellum in adolescent athletes. Gymnastics and baseball have the highest incidence of OCD of the capitellum which is likely attributed to compression of the radiocapitellar joint during hand stands and overhead throwing. This patient is presenting with advanced OCD which is indicated by radiographic changes and the presence of loose bodies in the elbow joint. Unstable OCD lesions with loose bodies generally require surgery. An arthroscopic or open excision of the loose bodies must be performed to prevent mechanical symptoms and recurrent effusions. Fixation of unstable lesions or osteochondral fragment removal can be performed depending on the size and stability of the lesion. In general fragments consisting of 50% or more of the capitellum require fixation. Osteochondral autograft plugs are used for large defects to restore the articular surface. 1,2

    Answer C.

    References

    1. Baker III, Champ L., Anthony A. Romeo, and Champ L. Baker Jr. “Osteochondritis dissecans of the capitellum.” The American journal of sports medicine 38.9 (2010): 1917-1928.
    2. Ruchelsman DE, Hall MP, Youm T. Osteochondritis Dissecans of the Capitellum: Current Concepts. J Am Acad Orthop Surg 2010;18:

     

  49. Question 49 of 64
    49. Question


    A 5-year-old girl presents to an urgent care with left elbow pain after falling off the monkey bars a few hours earlier. She landed on the left hand with the arm in extension. She has been unable to move the elbow since the injury. AP and lateral x-rays of the left elbow are shown in figures 1 and 2, respectively. What is the best treatment option for this patient?

    Correct

    A Monteggia fracture is characterized by a proximal third ulnar fracture with a dislocation of the radial head. These fractures are rare in children and can be missed at the initial presentation in up to 50% of pediatric patients.  A missed diagnosis can cause disabling consequence as a persistent radial head dislocation can start to cause permanent loss of elbow motion and stability after just two weeks from injury. When establishing a diagnosis, the patients range of motion should be carefully scrutinized for full elbow flexion and extension and full forearm supination and pronation. It is crucial for providers to be able to recognize an intact radiocapitellar joint on radiographs. This can often be difficult with the numerous ossification centers of the elbow. If there is any confusion on radiographic interpretation an x-ray of the contralateral elbow should be obtained for a side to side comparison. The treatment of choice is an acute closed reduction and long arm cast with the forearm placed in supination. The radial head will often reduce spontaneously when ulna length is restored. Closed reduction should be performed as soon as possible as the reduction becomes more difficult as time from injury passes. Chronic Monteggia fractures, defined as a dislocation beyond 4 weeks, usually require an open reduction. Open reduction and internal fixation is only performed in patients with closed growth plates. 1, 2

    Answer B.

    References

    1. Hubbard James, MD; Chauhan Aakash, MD, MBA; Fitzgerald Ryan, MD; Abrams Reid, MD; Mubarak Scott, MD; Sangimino Mark, MD;. Missed Pediatric Monteggia Fractures. JBJS Reviews – Review Article: 05 June 2018 – Volume 6 – Issue 6 – p. e2-e2
    2. Monteggia fracture. http://www.orthobullets.com. Accessed 9/23/2020
    Incorrect

    A Monteggia fracture is characterized by a proximal third ulnar fracture with a dislocation of the radial head. These fractures are rare in children and can be missed at the initial presentation in up to 50% of pediatric patients.  A missed diagnosis can cause disabling consequence as a persistent radial head dislocation can start to cause permanent loss of elbow motion and stability after just two weeks from injury. When establishing a diagnosis, the patients range of motion should be carefully scrutinized for full elbow flexion and extension and full forearm supination and pronation. It is crucial for providers to be able to recognize an intact radiocapitellar joint on radiographs. This can often be difficult with the numerous ossification centers of the elbow. If there is any confusion on radiographic interpretation an x-ray of the contralateral elbow should be obtained for a side to side comparison. The treatment of choice is an acute closed reduction and long arm cast with the forearm placed in supination. The radial head will often reduce spontaneously when ulna length is restored. Closed reduction should be performed as soon as possible as the reduction becomes more difficult as time from injury passes. Chronic Monteggia fractures, defined as a dislocation beyond 4 weeks, usually require an open reduction. Open reduction and internal fixation is only performed in patients with closed growth plates. 1, 2

    Answer B.

    References

    1. Hubbard James, MD; Chauhan Aakash, MD, MBA; Fitzgerald Ryan, MD; Abrams Reid, MD; Mubarak Scott, MD; Sangimino Mark, MD;. Missed Pediatric Monteggia Fractures. JBJS Reviews – Review Article: 05 June 2018 – Volume 6 – Issue 6 – p. e2-e2
    2. Monteggia fracture. http://www.orthobullets.com. Accessed 9/23/2020
  50. Question 50 of 64
    50. Question


    A 18-year-old male presents to your office with a growth on his right knee for a few years. The mass has progressively enlarged over the last 2-3 years and is now causing catching and locking as he bends the knee. He denies night pain and unexplained weight loss. On physical exam of the right knee there is a palpable hard mass to the anterolateral aspect of the knee adjacent to the superior portion of the patella and the patella is sitting slightly more medial than the left knee. The mass is non-tender to palpation and non-mobile. Figures 1 and 2 show a broad based tumor emanating from the distal metaphysis of the right femur. Axial MRI (figure 3) shows a dome shaped osseous lesion emanating from the lateral aspect of the distal femur metaphysis. There is a slender cartilaginous cap measuring 3 mm thick. What is the best treatment option for this patient?

    Correct

    The patient is presenting with a benign osteochondroma. This osteochondroma is uncommon in that it is located on the anterolateral femur causing mechanical impingement on the patella during knee motion. An osteochondroma is the most common benign tumor of childhood and young adults. Solitary osteochondromas may, although very rare (<1%), transform to a malignant chondrosarcoma over time. It is essential to identify tumor characteristics on radiographs and MRI to establish the correct diagnosis. Radiographic features of a benign lesion include tumor location at the metaphysis, the cortex of the lesion continuous with the cortex of the surrounding femur, and no metastases. Patterns of bone destruction signal a more malignant nature of a tumor. These patterns may include moth-eaten bone, a fast growing tumor, and an absent sclerotic boarder. MRI is the study of choice to assess the characteristics of the lesion to help determine if the lesion is benign or malignant. A cartilaginous cap greater than 1.5 cm is thickness is suspicious for malignant transformation. Surgical treatment of a solitary osteochondroma involves a marginal resection that includes the base of the stalk, the cartilage cap, and overlying periosteum. A wide resection is the treatment of choice for a chondrosarcoma. 1,2,3

    Answer C.

    References

    1. August Image Quiz: Osteochondroma. Journal of Orthopaedics for Physician Assistants – Image Quiz: 01 August 2015 – Volume 3 – Issue 3 – p. 16
    2. Aboulafia, Albert J. MD; Kennon, Robert E. MD; Jelinek, James S. MD Benign Bone Tumors of Childhood, Journal of the American Academy of Orthopaedic Surgeons: November 1999 – Volume 7 – Issue 6 – p 377-388.
    3. Osteochondroma. http://www.radiopaedia.org. Accessed on 3/28/21.
    Incorrect

    The patient is presenting with a benign osteochondroma. This osteochondroma is uncommon in that it is located on the anterolateral femur causing mechanical impingement on the patella during knee motion. An osteochondroma is the most common benign tumor of childhood and young adults. Solitary osteochondromas may, although very rare (<1%), transform to a malignant chondrosarcoma over time. It is essential to identify tumor characteristics on radiographs and MRI to establish the correct diagnosis. Radiographic features of a benign lesion include tumor location at the metaphysis, the cortex of the lesion continuous with the cortex of the surrounding femur, and no metastases. Patterns of bone destruction signal a more malignant nature of a tumor. These patterns may include moth-eaten bone, a fast growing tumor, and an absent sclerotic boarder. MRI is the study of choice to assess the characteristics of the lesion to help determine if the lesion is benign or malignant. A cartilaginous cap greater than 1.5 cm is thickness is suspicious for malignant transformation. Surgical treatment of a solitary osteochondroma involves a marginal resection that includes the base of the stalk, the cartilage cap, and overlying periosteum. A wide resection is the treatment of choice for a chondrosarcoma. 1,2,3

    Answer C.

    References

    1. August Image Quiz: Osteochondroma. Journal of Orthopaedics for Physician Assistants – Image Quiz: 01 August 2015 – Volume 3 – Issue 3 – p. 16
    2. Aboulafia, Albert J. MD; Kennon, Robert E. MD; Jelinek, James S. MD Benign Bone Tumors of Childhood, Journal of the American Academy of Orthopaedic Surgeons: November 1999 – Volume 7 – Issue 6 – p 377-388.
    3. Osteochondroma. http://www.radiopaedia.org. Accessed on 3/28/21.
  51. Question 51 of 64
    51. Question


    A 57 year-old male presents to the emergency room with right knee pain after slipping on the ice earlier in the day. During the fall the knee went into flexion and he felt a pop as he fell. He has not been able to put weight on the leg or extend the leg since the fall. He has not injured this knee before. AP and lateral radiographs (figures 1 and 2) of the right knee demonstrate a low riding patella with a superior pole avulsion. On physical exam of the right knee he has mild swelling to the knee without obvious deformity. He is unable to straight leg raise. What is the next best step in treatment for this patient?

    Correct

    The patient has clearly disrupted his extensor mechanism as he is unable to straight leg raise. Radiographs indicate he has ruptured his quadriceps tendon as a low riding patella (patella baja) is present. Quadriceps ruptures occur almost exclusively in men over 40 years of age. Quadriceps tendon ruptures are usually pretty obvious clinically with a palpable defect in the tendon near the patella insertion (suprapatellar gap), inability to straight leg raise, and patella baja on radiographs. MRI is only necessary if the diagnosis remains unclear. Acute ruptures are treated surgically with repair of the tendon back down to the patella. Patients should be treated surgically within 7 days as tendon retraction can occur beyond 72 hours. Retraction over time can make the repair more difficult and a graft may be necessary to fill the gap in cases of chronic tears. The most appropriate initial treatment in the emergency room setting is to place the injured extremity in a knee immobilizer with instructions to weight bear as tolerated with crutches. Instructions to follow-up with an orthopedic surgeon within 1-2 day is critical to ensure a timely repair. 1,2

    Answer B.

    References

    1. Ilan, Doron I. MD; Tejwani, Nirmal MD; Keschner, Mitchell MD; Leibman, Matthew MD Quadriceps Tendon Rupture, Journal of the American Academy of Orthopaedic Surgeons: May 2003 – Volume 11 – Issue 3 – p 192-200
    2. Quadriceps rupture. http://www.orthobullets.com. Accessed on 2/28/21.
    Incorrect

    The patient has clearly disrupted his extensor mechanism as he is unable to straight leg raise. Radiographs indicate he has ruptured his quadriceps tendon as a low riding patella (patella baja) is present. Quadriceps ruptures occur almost exclusively in men over 40 years of age. Quadriceps tendon ruptures are usually pretty obvious clinically with a palpable defect in the tendon near the patella insertion (suprapatellar gap), inability to straight leg raise, and patella baja on radiographs. MRI is only necessary if the diagnosis remains unclear. Acute ruptures are treated surgically with repair of the tendon back down to the patella. Patients should be treated surgically within 7 days as tendon retraction can occur beyond 72 hours. Retraction over time can make the repair more difficult and a graft may be necessary to fill the gap in cases of chronic tears. The most appropriate initial treatment in the emergency room setting is to place the injured extremity in a knee immobilizer with instructions to weight bear as tolerated with crutches. Instructions to follow-up with an orthopedic surgeon within 1-2 day is critical to ensure a timely repair. 1,2

    Answer B.

    References

    1. Ilan, Doron I. MD; Tejwani, Nirmal MD; Keschner, Mitchell MD; Leibman, Matthew MD Quadriceps Tendon Rupture, Journal of the American Academy of Orthopaedic Surgeons: May 2003 – Volume 11 – Issue 3 – p 192-200
    2. Quadriceps rupture. http://www.orthobullets.com. Accessed on 2/28/21.
  52. Question 52 of 64
    52. Question


    An 11-year-old male presents to your clinic with left ankle pain after twisting his ankle while playing basketball 2 days ago. He has had pain and swelling in the ankle since and is having trouble bearing weight. On physical exam there is mild bruising and swelling to the lateral ankle. He has tenderness to palpation over the distal fibula and anterior talofibular ligament (ATFL). AP and lateral radiographs are shown in figures 1 and 2. Which statement is true regarding the possibility of a lateral ankle sprain vs. a Salter-Harris I distal fibular fracture in this patient?

     

    Correct

    Lateral ankle injuries are commonly seen in the pediatric population. In fact, ankle trauma is the most common pediatric injury. In skeletally immature patients it can be hard to conclude whether the injury is a lateral ankle sprain or a Salter-Harris I (non-displaced) fracture. A careful physical exam is critical in making the diagnosis, however kids often have difficulty pinpointing the exact location of the pain. Historically most lateral ankle injuries have been presumed to be growth plate injuries. The theory is that in growing kids, the physis is weaker than the surrounding ligaments and therefore injury to the growth plate is more likely. This theory has since been proven untrue. Boutis et al. performed an MRI on 135 pediatric patients who sustained a lateral ankle injury and found that almost all patients injured the lateral ligaments and not the physis. Only 4 of 135 (3%) patients had an MRI confirmed Salter-Harris I fracture. Salter-Harris I fractures and lateral ankle sprains have a similar treatment course which includes short-term immobilization or bracing (usually 3 weeks depending on symptoms) with a gradual return to activities as tolerated. Physical therapy is an important treatment for lateral ankle sprains to regain ankle motion, reduce swelling, and strengthening. Misdiagnosing lateral ankle sprains for physeal injuries often prevents physical therapy referral and leads to unnecessary immobilization and follow-up radiographs.

    Answer D.

    References

    1. Boutis, Kathy, et al. “Radiograph-negative lateral ankle injuries in children: occult growth plate fracture or sprain?.” JAMA pediatrics 170.1 (2016): e154114-e154114.
    2. Rougereau G, Noailles T, El Khoury G, Bauer T, Langlais T, Hardy A. Is lateral ankle sprain of the child and adolescent a myth or a reality? A systematic review of the literature. Foot and Ankle Surgery. 2021 May 1.
    Incorrect

    Lateral ankle injuries are commonly seen in the pediatric population. In fact, ankle trauma is the most common pediatric injury. In skeletally immature patients it can be hard to conclude whether the injury is a lateral ankle sprain or a Salter-Harris I (non-displaced) fracture. A careful physical exam is critical in making the diagnosis, however kids often have difficulty pinpointing the exact location of the pain. Historically most lateral ankle injuries have been presumed to be growth plate injuries. The theory is that in growing kids, the physis is weaker than the surrounding ligaments and therefore injury to the growth plate is more likely. This theory has since been proven untrue. Boutis et al. performed an MRI on 135 pediatric patients who sustained a lateral ankle injury and found that almost all patients injured the lateral ligaments and not the physis. Only 4 of 135 (3%) patients had an MRI confirmed Salter-Harris I fracture. Salter-Harris I fractures and lateral ankle sprains have a similar treatment course which includes short-term immobilization or bracing (usually 3 weeks depending on symptoms) with a gradual return to activities as tolerated. Physical therapy is an important treatment for lateral ankle sprains to regain ankle motion, reduce swelling, and strengthening. Misdiagnosing lateral ankle sprains for physeal injuries often prevents physical therapy referral and leads to unnecessary immobilization and follow-up radiographs.

    Answer D.

    References

    1. Boutis, Kathy, et al. “Radiograph-negative lateral ankle injuries in children: occult growth plate fracture or sprain?.” JAMA pediatrics 170.1 (2016): e154114-e154114.
    2. Rougereau G, Noailles T, El Khoury G, Bauer T, Langlais T, Hardy A. Is lateral ankle sprain of the child and adolescent a myth or a reality? A systematic review of the literature. Foot and Ankle Surgery. 2021 May 1.
  53. Question 53 of 64
    53. Question


    A 2 year old boy is brought to your office by his mother with a 3 day history of limping and left lower extremity pain. The child was at daycare when one staff member noticed the child limping on the playground. On physical exam you notice mild swelling to the distal leg but otherwise no deformity or bruising. The child refuses to ambulate on the left leg or let you examine him. AP and lateral x-rays are shown in figures 1 and 2. You suspect a toddler’s fracture. Which statement is true regarding a toddler’s fracture?

    Correct

    A toddler’s fracture is a low impact fracture of the distal one-third tibia with an intact fibula that usually occurs in children under the age of 2.5 years. A toddler’s fracture can occur as the tibia is rotated and the foot is fixed to the ground which causes the classic spiral fracture pattern. The injury can occur while running or falling from a height. The injury frequently goes unnoticed by parents or caregivers which may delay the presentation to clinic. Common physical exam findings include swelling and refusal to bear weight on the affected extremity. The fracture is generally non-displaced and may not be evident on initial radiographs. A toddler that presents to the office with a limp, pain along the affected tibia, and normal x-rays should be treated as a fracture. Treatment involves placing the child in a long leg cast with the knee flexed at 30 degrees to avoid weight bearing. Healing callus may be evident along the tibial periosteum as early as 2-3 weeks after the injury and may be the only evidence that a fracture occurred. The differential diagnosis in a limping toddler may include other subtle fractures that are caused by minor trauma including metatarsal buckle fractures, fractures of the midfoot, and fractures of the proximal tibia and distal tibia and fibula. 1,2

    Answer C

    References

    1. John, Susan D., Chetan S. Moorthy, and Leonard E. Swischuk. “Expanding the concept of the toddler’s fracture.” Radiographics 17.2 (1997): 367-376.
    2. Halsey, Matthew F., et al. “Toddler’s fracture: presumptive diagnosis and treatment.” Journal of Pediatric Orthopaedics 21.2 (2001): 152-156.
    Incorrect

    A toddler’s fracture is a low impact fracture of the distal one-third tibia with an intact fibula that usually occurs in children under the age of 2.5 years. A toddler’s fracture can occur as the tibia is rotated and the foot is fixed to the ground which causes the classic spiral fracture pattern. The injury can occur while running or falling from a height. The injury frequently goes unnoticed by parents or caregivers which may delay the presentation to clinic. Common physical exam findings include swelling and refusal to bear weight on the affected extremity. The fracture is generally non-displaced and may not be evident on initial radiographs. A toddler that presents to the office with a limp, pain along the affected tibia, and normal x-rays should be treated as a fracture. Treatment involves placing the child in a long leg cast with the knee flexed at 30 degrees to avoid weight bearing. Healing callus may be evident along the tibial periosteum as early as 2-3 weeks after the injury and may be the only evidence that a fracture occurred. The differential diagnosis in a limping toddler may include other subtle fractures that are caused by minor trauma including metatarsal buckle fractures, fractures of the midfoot, and fractures of the proximal tibia and distal tibia and fibula. 1,2

    Answer C

    References

    1. John, Susan D., Chetan S. Moorthy, and Leonard E. Swischuk. “Expanding the concept of the toddler’s fracture.” Radiographics 17.2 (1997): 367-376.
    2. Halsey, Matthew F., et al. “Toddler’s fracture: presumptive diagnosis and treatment.” Journal of Pediatric Orthopaedics 21.2 (2001): 152-156.
  54. Question 54 of 64
    54. Question


    A 26-year-old male presents to your office with a one-week history of left knee pain and instability. The injury occurred during a soccer game when an opponent collided with the outside of his knee. He had immediate pain and swelling. He went to an urgent care where x-rays of the left knee were unremarkable for an acute injury. An MRI was then ordered showing a grade 3 medial collateral ligament (MCL) sprain with pes anserine entrapment causing a Stener-like lesion (figure 1). What is the best treatment option?

    Correct

    The medial collateral ligament (MCL) is the most commonly injured ligament in the knee. The MCL is typically injured during athletic activities where the knee is struck from the side, or is externally rotated, causing a valgus load to the knee. Physical exam findings include increased laxity to the MCL with valgus stress at 0 and 30 degrees. Comparing knee laxity to the contralateral knee is required to get a true measure of an acute MCL injury. Grade 1 MCL injury is <5mm of medal joint line opening with a firm endpoint, grad 2 is 5 to 10mm with a firm endpoint, and grade 3 is >10mm with no endpoint to valgus stress. Grade 1 and 2 injuries should be treated with a hinged knee brace, weight bearing as tolerated, and physical therapy. Treatment of grade 3 injuries remains controversial. Midsubstance tears have an improved healing rate compared to MCL injuries that avulse off the bone. An indication for surgery includes a complete tibial-sided MCL tear with pes anserine entrapment as seen with this patient. In this patient the pes anserine is displacing the distal MCL and preventing contact of the tendon from its native bony insertion, which prevents healing. Surgical repair can be achieved by retracting the pes tendons off the MCL insertion site so the MCL can be tied down with suture anchors. If the tendon is no longer viable for a tendon to bone repair, a reconstruction with allograft may be required. 1,2,3

     

    Answer C.

    References

    1. Miyamoto RG, Bosco JA, Sherman OH. Treatment of medial collateral ligament injuries. JAAOS-Journal of the American Academy of Orthopaedic Surgeons. 2009 Mar 1;17(3):152-61.
    2. Wijdicks CA, Griffith CJ, Johansen S, Engebretsen L, LaPrade RF. Injuries to the Medial Collateral Ligament and Associated Medial Structures of the Knee. J Bone Joint Surg Am, May 2010; 92(5); 1266-1280
    3. Corten K, Hoser C, Fink C, Bellemans J. Case reports: a Stener-like lesion of the medial collateral ligament of the knee. Clin Orthop Relat Res. 2010;468(1):289-293.
    Incorrect

    The medial collateral ligament (MCL) is the most commonly injured ligament in the knee. The MCL is typically injured during athletic activities where the knee is struck from the side, or is externally rotated, causing a valgus load to the knee. Physical exam findings include increased laxity to the MCL with valgus stress at 0 and 30 degrees. Comparing knee laxity to the contralateral knee is required to get a true measure of an acute MCL injury. Grade 1 MCL injury is <5mm of medal joint line opening with a firm endpoint, grad 2 is 5 to 10mm with a firm endpoint, and grade 3 is >10mm with no endpoint to valgus stress. Grade 1 and 2 injuries should be treated with a hinged knee brace, weight bearing as tolerated, and physical therapy. Treatment of grade 3 injuries remains controversial. Midsubstance tears have an improved healing rate compared to MCL injuries that avulse off the bone. An indication for surgery includes a complete tibial-sided MCL tear with pes anserine entrapment as seen with this patient. In this patient the pes anserine is displacing the distal MCL and preventing contact of the tendon from its native bony insertion, which prevents healing. Surgical repair can be achieved by retracting the pes tendons off the MCL insertion site so the MCL can be tied down with suture anchors. If the tendon is no longer viable for a tendon to bone repair, a reconstruction with allograft may be required. 1,2,3

     

    Answer C.

    References

    1. Miyamoto RG, Bosco JA, Sherman OH. Treatment of medial collateral ligament injuries. JAAOS-Journal of the American Academy of Orthopaedic Surgeons. 2009 Mar 1;17(3):152-61.
    2. Wijdicks CA, Griffith CJ, Johansen S, Engebretsen L, LaPrade RF. Injuries to the Medial Collateral Ligament and Associated Medial Structures of the Knee. J Bone Joint Surg Am, May 2010; 92(5); 1266-1280
    3. Corten K, Hoser C, Fink C, Bellemans J. Case reports: a Stener-like lesion of the medial collateral ligament of the knee. Clin Orthop Relat Res. 2010;468(1):289-293.
  55. Question 55 of 64
    55. Question


    A 42-year-old male presents to your office with severe left buttock pain after lifting a heavy object in his garage 3 days ago. He has developed bruising in the left buttock area over the last day. On physical exam he is very tender over the ischial tuberosity on the left and has very weak hamstring strength. There is no evidence of a bony avulsion fracture on x-ray. Sagittal STIR MRI image of the pelvis shows an avulsion tear of the conjoined tendon of the biceps femoris, semitendinosus, and semimembranosus with 5 cm of distal retraction. What is the best treatment option?

    Correct

    The hamstrings are made of 3 muscles including the semimembranosus, semitendinosus, and the biceps femoris which all attach to the pelvis at the ischial tuberosity. The biceps femoris has a long head, which attaches to the ischial tuberosity and a short head, which attaches to the mid-femur. The hamstring muscles can tear with a sudden extension of the knee in a flexed position, or a sudden contraction of the hamstrings. Most tears are muscular (better described as a strain) and occur near the midpoint of the hamstrings. Proximal avulsion injuries, where the hamstring tendons avulse off the ischial tuberosity, represent a more severe injury. Patients with proximal injuries have trouble sitting, ecchymosis in the buttocks a few days after the injury, and severe hamstring weakness. Radiographs should be taken in all patients with a suspected proximal rupture to rule out a bony avulsion fracture, which is common in the pediatric population. MRI is the study of choice to determine the extent of proximal avulsion injuries. Non-operative treatment is indicated with hamstring strains, one tendon proximal tears, and two tendon proximal tears with less than 2mm of retraction. Surgical repair is indicated when all three tendons are torn off the ischial tuberosity or if there is >2mm retraction from two tendons. Acute repair should be performed within 4 weeks of the injury or the tendon begins to retract and becomes difficult to repair. 1,2

    Answer D.

    References

    1. Alzahrani M, Aldebeyan S, Abduljabbar F. Hamstring Injuries in Athletes: Diagnosis and Treatment. JBJS Reviews – Review Article: 30 June 2015 – Volume 3 – Issue 6 – p. e5
    2. Cohen S, Bradley J. Acute Proximal Hamstring Rupture, Journal of the American Academy of Orthopaedic Surgeons: June 2007 – Volume 15 – Issue 6 – p 350-355

     

    Incorrect

    The hamstrings are made of 3 muscles including the semimembranosus, semitendinosus, and the biceps femoris which all attach to the pelvis at the ischial tuberosity. The biceps femoris has a long head, which attaches to the ischial tuberosity and a short head, which attaches to the mid-femur. The hamstring muscles can tear with a sudden extension of the knee in a flexed position, or a sudden contraction of the hamstrings. Most tears are muscular (better described as a strain) and occur near the midpoint of the hamstrings. Proximal avulsion injuries, where the hamstring tendons avulse off the ischial tuberosity, represent a more severe injury. Patients with proximal injuries have trouble sitting, ecchymosis in the buttocks a few days after the injury, and severe hamstring weakness. Radiographs should be taken in all patients with a suspected proximal rupture to rule out a bony avulsion fracture, which is common in the pediatric population. MRI is the study of choice to determine the extent of proximal avulsion injuries. Non-operative treatment is indicated with hamstring strains, one tendon proximal tears, and two tendon proximal tears with less than 2mm of retraction. Surgical repair is indicated when all three tendons are torn off the ischial tuberosity or if there is >2mm retraction from two tendons. Acute repair should be performed within 4 weeks of the injury or the tendon begins to retract and becomes difficult to repair. 1,2

    Answer D.

    References

    1. Alzahrani M, Aldebeyan S, Abduljabbar F. Hamstring Injuries in Athletes: Diagnosis and Treatment. JBJS Reviews – Review Article: 30 June 2015 – Volume 3 – Issue 6 – p. e5
    2. Cohen S, Bradley J. Acute Proximal Hamstring Rupture, Journal of the American Academy of Orthopaedic Surgeons: June 2007 – Volume 15 – Issue 6 – p 350-355

     

  56. Question 56 of 64
    56. Question


    A 29-year-old male presents to your office with left arm pain after lifting a lawn tractor at work. He felt a sharp paining in his elbow when he lifted the tractor and now the arm feels weak. On physical exam of the elbow he has full range of motion. He has no bruising or deformity to the arm. His distal biceps can be felt with a hook test and he has tenderness to palpation over the radial tuberosity at the attachment of the distal biceps tendon. X-rays are unremarkable for fractures or abnormal findings. Sagittal and coronal STIR MRI images show a partial tear (<50%) of the distal biceps tendon (figures 1 and 2). What is the best treatment option?

    Correct

    Distal biceps injuries can be a challenge to diagnose and treat.  The distal biceps tendon attaches to the radial tuberosity at the antecubital fossa. The function of the distal biceps is to flex the elbow and supinate the wrist. Partial or complete tears generally occur at the attachment site of the radial tuberosity after a sudden flexion of the elbow while lifting with the arm. Partial tears may have mild symptoms with full range of motion of the elbow and mild pain. Elbow flexion and supination of the forearm commonly shows varying degrees of weakness. Complete tears present with more swelling, ecchymosis, and a positive hook test. The hook test involves the examiner using his/her index finger to hook the intact distal biceps at the antecrubal fossa. MRI is the diagnostic study of choice to identify partial vs complete tears of the distal biceps tendon. Partial tears through less than 50% of the tendon are usually treated non-operatively where greater than 50% may do better with surgery. Non-operative treatment varies widely in the literature but in general a period of 2-3 weeks of elbow immobilization, followed by 2-3 weeks of full motion without lifting, then another month or two of lifting restrictions based on the patient’s symptoms. If patients fail 3-6 months of non-operative treatment, surgical detachment and re-attachment of the tendon is a successful treatment option. 1,2

    Answer A

    References

    1. Bain, Gregory I., Luke J. Johnson, and Perry C. Turner. “Treatment of partial distal biceps tendon tears.” Sports Medicine and Arthroscopy Review 16.3 (2008): 154-161.
    2. Dürr, Hans Roland, et al. “Partial rupture of the distal biceps tendon.” Clinical Orthopaedics and Related Research (1976- ) 374 (2000): 195-200.
    Incorrect

    Distal biceps injuries can be a challenge to diagnose and treat.  The distal biceps tendon attaches to the radial tuberosity at the antecubital fossa. The function of the distal biceps is to flex the elbow and supinate the wrist. Partial or complete tears generally occur at the attachment site of the radial tuberosity after a sudden flexion of the elbow while lifting with the arm. Partial tears may have mild symptoms with full range of motion of the elbow and mild pain. Elbow flexion and supination of the forearm commonly shows varying degrees of weakness. Complete tears present with more swelling, ecchymosis, and a positive hook test. The hook test involves the examiner using his/her index finger to hook the intact distal biceps at the antecrubal fossa. MRI is the diagnostic study of choice to identify partial vs complete tears of the distal biceps tendon. Partial tears through less than 50% of the tendon are usually treated non-operatively where greater than 50% may do better with surgery. Non-operative treatment varies widely in the literature but in general a period of 2-3 weeks of elbow immobilization, followed by 2-3 weeks of full motion without lifting, then another month or two of lifting restrictions based on the patient’s symptoms. If patients fail 3-6 months of non-operative treatment, surgical detachment and re-attachment of the tendon is a successful treatment option. 1,2

    Answer A

    References

    1. Bain, Gregory I., Luke J. Johnson, and Perry C. Turner. “Treatment of partial distal biceps tendon tears.” Sports Medicine and Arthroscopy Review 16.3 (2008): 154-161.
    2. Dürr, Hans Roland, et al. “Partial rupture of the distal biceps tendon.” Clinical Orthopaedics and Related Research (1976- ) 374 (2000): 195-200.
  57. Question 57 of 64
    57. Question


    A 20 year-old female presents to the office with 6 months of right hip pain. She is an avid runner and has been training for a marathon for the past year.  Over the last 6 months she is having sharp hip pain with prolonged walking, running, and squatting. She tried 2 months of rest without running and anti-inflammatory medications, followed by a 3 month course of physical therapy for the hip without significant relief. She also tried an intra-articular cortisone injection 3 months ago which provided her 100% relief for 2-3 weeks. Sagittal MRI arthrogram image above shows a small anterior superior labral tear. On physical exam the patient has sharp pain with flexion, abduction, and internal rotation of the hip (FADIR impingement). What is the best treatment option for this patient?

    Correct

    The hip labrum is a soft tissue bumper around the acetabulum that contributes to hip stability. The labrum can become damaged with repetitive sports and in patients with hip dysplasia and femoroacetabular impingement. Labral tears can also be found in a high percentage of asymptomatic patients so a careful physical exam and history must be included with image findings to determine the etiology of hip pain. One study found that 69% of asymptomatic patients between the ages of 15 and 66 had a labral tear. In patients with MRI evidence of a hip labral tear who have failed conservative treatment (rest, NSAIDS, PT), a reasonable first step approach is to perform an intra-articular injection into the hip joint. In young active patients the current literature supports arthroscopic labral repair over debridement. A short period of relief from the injection helps make the diagnosis of an intra-articular source of hip pain such as a labral tear. Patients with a labral tear who have failed 3 months of conservative treatment are candidates for arthroscopic labral repair vs. debridement. Labral debridement alone has been found to be associated with a significantly higher risk of progression of hip arthritis compared with labral repair. If the labrum has viable tissue and a good vascular supply, then labral repair is favored in young active patients without arthritic changes. A typical rehab protocol for a labral repair includes 2-4 weeks of partial weight bearing with crutches, avoidance of deep hip flexion beyond 90 degrees for 6 weeks, and 2-3 months of physical therapy. 1,2

    Answer B.

    References

    1. Register B, Pennock AT, Ho CP, Strickland CD, Lawand A, Philippon MJ. Prevalence of abnormal hip findings in asymptomatic participants: a prospective, blinded study. Am J Sports Med. , December 2012 ; 40(12); 2720-272
    2. Domb BG, Hartigan DE, Perets I. Decision Making for Labral Treatment in the Hip: Repair Versus Débridement Versus Reconstruction, Journal of the American Academy of Orthopaedic Surgeons: March 2017 – Volume 25 – Issue 3 – p e53-e62

     

    Incorrect

    The hip labrum is a soft tissue bumper around the acetabulum that contributes to hip stability. The labrum can become damaged with repetitive sports and in patients with hip dysplasia and femoroacetabular impingement. Labral tears can also be found in a high percentage of asymptomatic patients so a careful physical exam and history must be included with image findings to determine the etiology of hip pain. One study found that 69% of asymptomatic patients between the ages of 15 and 66 had a labral tear. In patients with MRI evidence of a hip labral tear who have failed conservative treatment (rest, NSAIDS, PT), a reasonable first step approach is to perform an intra-articular injection into the hip joint. In young active patients the current literature supports arthroscopic labral repair over debridement. A short period of relief from the injection helps make the diagnosis of an intra-articular source of hip pain such as a labral tear. Patients with a labral tear who have failed 3 months of conservative treatment are candidates for arthroscopic labral repair vs. debridement. Labral debridement alone has been found to be associated with a significantly higher risk of progression of hip arthritis compared with labral repair. If the labrum has viable tissue and a good vascular supply, then labral repair is favored in young active patients without arthritic changes. A typical rehab protocol for a labral repair includes 2-4 weeks of partial weight bearing with crutches, avoidance of deep hip flexion beyond 90 degrees for 6 weeks, and 2-3 months of physical therapy. 1,2

    Answer B.

    References

    1. Register B, Pennock AT, Ho CP, Strickland CD, Lawand A, Philippon MJ. Prevalence of abnormal hip findings in asymptomatic participants: a prospective, blinded study. Am J Sports Med. , December 2012 ; 40(12); 2720-272
    2. Domb BG, Hartigan DE, Perets I. Decision Making for Labral Treatment in the Hip: Repair Versus Débridement Versus Reconstruction, Journal of the American Academy of Orthopaedic Surgeons: March 2017 – Volume 25 – Issue 3 – p e53-e62

     

  58. Question 58 of 64
    58. Question


    A 52-year-old women presents to your office with a firm nodule developing in the palm of her right hand. The nodule is non-painful and she is able to use the hand without limitations. On physical exam you palpate a firm nodule in the palm of the hand involving the 3rd and 4th digits. She is able to fully extend all fingers and there is no flexion contracture of the metacarpophalangeal (MCP) joints. A picture of the right hand is shown in figure 1. What is the best treatment option for this patient?

    Correct

    Dupuytren’s disease, or Dupuytren’s contracture, is an abnormal proliferation of the palmar fascia of the hand which leads to thickening of the palmar fascia. As the “cord” or thickening of the palmar fascia progresses, a flexion deformity of the MCP joint can start to develop. This patient is presenting early in the disease with a non-painful enlarging nodule and cords in the palm of her hand. Dupuytren’s disease is difficult to treat as there is no cure and the disease often progresses and reoccurs after surgical treatment. Observation is the best treatment option for patients who present early in the disease without pain and functional limitations. Physical therapy and bracing have not been shown to prevent or reverse Dupuytren’s contractures. Intralesional steroid injections may soften the nodule and potentially slow the progression of the disease. Injection of Clostridium histolyticum collagenase (Xiaflex) into the disease nodule can help break up and rupture cords to release flexion contractures. Surgical resection of the cords is indicated when there is decreased hand function caused by a flexion contracture at the MCP joint >30 degrees and the PIP joint >15 degrees. 1,2

    Answer A.

    References

    1. Rayan G. Dupuytren Disease: Anatomy, Pathology, Presentation, and Treatment. The Journal of Bone & Joint Surgery – Selected Instructional Course Lecture: January 2007 – Volume 89 – Issue 1 – p. 189-199
    2. Riester S, Van Wijnen A, Rizzo M, Kakar S. Pathogenesis and Treatment of Dupuytren Disease. JBJS Reviews – Review Article: 08 April 2014 – Volume 2 – Issue 4 – p. e2
    Incorrect

    Dupuytren’s disease, or Dupuytren’s contracture, is an abnormal proliferation of the palmar fascia of the hand which leads to thickening of the palmar fascia. As the “cord” or thickening of the palmar fascia progresses, a flexion deformity of the MCP joint can start to develop. This patient is presenting early in the disease with a non-painful enlarging nodule and cords in the palm of her hand. Dupuytren’s disease is difficult to treat as there is no cure and the disease often progresses and reoccurs after surgical treatment. Observation is the best treatment option for patients who present early in the disease without pain and functional limitations. Physical therapy and bracing have not been shown to prevent or reverse Dupuytren’s contractures. Intralesional steroid injections may soften the nodule and potentially slow the progression of the disease. Injection of Clostridium histolyticum collagenase (Xiaflex) into the disease nodule can help break up and rupture cords to release flexion contractures. Surgical resection of the cords is indicated when there is decreased hand function caused by a flexion contracture at the MCP joint >30 degrees and the PIP joint >15 degrees. 1,2

    Answer A.

    References

    1. Rayan G. Dupuytren Disease: Anatomy, Pathology, Presentation, and Treatment. The Journal of Bone & Joint Surgery – Selected Instructional Course Lecture: January 2007 – Volume 89 – Issue 1 – p. 189-199
    2. Riester S, Van Wijnen A, Rizzo M, Kakar S. Pathogenesis and Treatment of Dupuytren Disease. JBJS Reviews – Review Article: 08 April 2014 – Volume 2 – Issue 4 – p. e2
  59. Question 59 of 64
    59. Question


    A 39-year-old female presents to the emergency room with severe right ankle pain after a motorcycle accident. She has an obvious valgus deformity to the ankle but the skin is intact. AP radiograph of the talus fracture with subtalar dislocation is shown in figure 1. The ankle was closed reduced and splinted in the emergency room. AP and lateral radiographs post-reduction are shown in figures 2 and 3, respectively. What is the most likely incidence of osteonecrosis of the talus after this fracture?

    Correct

    The patient sustained a talar neck fracture with subtalar dislocation. The fracture was closed reduced in the emergency room to relieve pressure off the lateral skin and the patient was brought to the operating room for open reduction and internal fixation of the talus fracture the next day. The most common type of talus fracture is a talar neck fracture. Neck fractures with >2mm of displacement require surgical fixation. Fractures with significant displacement should be closed reduced urgently to protect the blood supply and promote revascularization. Depending on fracture displacement, osteonecrosis can be a common complication that arises from displaced fractures. This patients Hawkins type II fractures has a 21%-42% chance of developing avascular necrosis (AVN) even with closed reduction and surgical fixation. Comparably a minimally displaced Hawkins type I fracture has an 8%-10% chance of AVN and a 45% to 90% chance for type III fractures. The appearance of osteonecrosis on x-ray can start on average at 7 months after surgical fixation, with a range from 3 to 9 months. 1,2

    Answer B.

    References

    1. Lee, Christopher MD; Brodke, Dane MD; Perdue, Paul W. Jr MD; Patel, Tejas MD Talus Fractures: Evaluation and Treatment, Journal of the American Academy of Orthopaedic Surgeons: October 15, 2020 – Volume 28 – Issue 20 – p e878-e887.
    2. Vallier HA, Nork SE, Barei DP, Benirschke SK, Sangeozan BJ. Talar Neck Fractures: Results and Outcomes. The Journal of Bone & Joint Surgery – Scientific Articles: August 2004 – Volume 86 – Issue 8 – p. 1616-1624

     

    Incorrect

    The patient sustained a talar neck fracture with subtalar dislocation. The fracture was closed reduced in the emergency room to relieve pressure off the lateral skin and the patient was brought to the operating room for open reduction and internal fixation of the talus fracture the next day. The most common type of talus fracture is a talar neck fracture. Neck fractures with >2mm of displacement require surgical fixation. Fractures with significant displacement should be closed reduced urgently to protect the blood supply and promote revascularization. Depending on fracture displacement, osteonecrosis can be a common complication that arises from displaced fractures. This patients Hawkins type II fractures has a 21%-42% chance of developing avascular necrosis (AVN) even with closed reduction and surgical fixation. Comparably a minimally displaced Hawkins type I fracture has an 8%-10% chance of AVN and a 45% to 90% chance for type III fractures. The appearance of osteonecrosis on x-ray can start on average at 7 months after surgical fixation, with a range from 3 to 9 months. 1,2

    Answer B.

    References

    1. Lee, Christopher MD; Brodke, Dane MD; Perdue, Paul W. Jr MD; Patel, Tejas MD Talus Fractures: Evaluation and Treatment, Journal of the American Academy of Orthopaedic Surgeons: October 15, 2020 – Volume 28 – Issue 20 – p e878-e887.
    2. Vallier HA, Nork SE, Barei DP, Benirschke SK, Sangeozan BJ. Talar Neck Fractures: Results and Outcomes. The Journal of Bone & Joint Surgery – Scientific Articles: August 2004 – Volume 86 – Issue 8 – p. 1616-1624

     

  60. Question 60 of 64
    60. Question

    A 32 year old female presents to the office with a 3 month history of right medial ankle pain. She denies a known injury or precipitating event. The pain seems to be worse during exercise and when standing all day at her job as a nurse. On physical exam she has a pes planus deformity and mild hindfoot valgus. She has poorly localized pain over the posterior medial ankle and heel. Deep palpation to the medial heel causes radiating pain to the plantar aspect of the foot. She also complains of burning pain in the plantar aspect of the foot with the foot in dorsiflexion and eversion. Her radiographs show no evidence of arthritis or deformity. What is the next best step in treatment?

    Correct

    The patient is presenting with symptoms of tarsal tunnel syndrome (TTS). Her radiographs show no evidence of arthritis or deformity. TTS is a relatively uncommon diagnosis that is thought to be over diagnosed. TTS is a compression neuropathy of the tibial nerve at the medial ankle. The tarsal tunnel is formed by the flexor retinaculum medially and the posterior process of the talus and calcaneus laterally. Structures that pass through the tunnel include the tibial nerve, posterior tibial artery, flexor hallucis longus, flexor digitorum longus, and the tibialis posterior tendons. Foot and ankle deformities such as pes planus, hindfoot valgus, and forefoot adduction place tension on the tibial nerve and contribute to the neuropathy. Placing the foot in dorsiflexion and eversion maximally stretches the tibial nerve and may reproduce symptoms. Compression of the tibial nerve at the tarsal tunnel often sends radiating tingling, burning, or numbness to the plantar aspect of the foot where the nerve endings terminate. A positive tinel’s causing radiating symptoms is a valuable provocative test. MRI is the first study to order when tarsal tunnel is suspected. MRI closely evaluates the structures of the medial ankle and can identify a space occupying lesion such as a ganglion cyst, tumor, or ostephyte that may be compressing the tibial nerve. Nerve conduction velocity studies may be helpful after the MRI but can be limited by false negative rates. TTS is first managed with immobilization, neuroleptic drugs, and NSAIDs if no space occupying lesion is found on MRI. 1,2

    Answer C.

    References

    1. Wukich DK, Tuaxon DA. Diagnosis and Treatment of Chronic Ankle Pain. The Journal of Bone & Joint Surgery – Selected Instructional Course Lecture: 18 August 2010 – Volume 92 – Issue 10 – p. 2002-2016
    2. Lareau, Craig R. MD; Sawyer, Gregory A. MD; Wang, Joanne H. BA; DiGiovanni, Christopher W. MD Plantar and Medial Heel Pain, Journal of the American Academy of Orthopaedic Surgeons: June 2014 – Volume 22 – Issue 6 – p 372-380 doi: 10.5435/JAAOS-22-06-37.
    Incorrect

    The patient is presenting with symptoms of tarsal tunnel syndrome (TTS). Her radiographs show no evidence of arthritis or deformity. TTS is a relatively uncommon diagnosis that is thought to be over diagnosed. TTS is a compression neuropathy of the tibial nerve at the medial ankle. The tarsal tunnel is formed by the flexor retinaculum medially and the posterior process of the talus and calcaneus laterally. Structures that pass through the tunnel include the tibial nerve, posterior tibial artery, flexor hallucis longus, flexor digitorum longus, and the tibialis posterior tendons. Foot and ankle deformities such as pes planus, hindfoot valgus, and forefoot adduction place tension on the tibial nerve and contribute to the neuropathy. Placing the foot in dorsiflexion and eversion maximally stretches the tibial nerve and may reproduce symptoms. Compression of the tibial nerve at the tarsal tunnel often sends radiating tingling, burning, or numbness to the plantar aspect of the foot where the nerve endings terminate. A positive tinel’s causing radiating symptoms is a valuable provocative test. MRI is the first study to order when tarsal tunnel is suspected. MRI closely evaluates the structures of the medial ankle and can identify a space occupying lesion such as a ganglion cyst, tumor, or ostephyte that may be compressing the tibial nerve. Nerve conduction velocity studies may be helpful after the MRI but can be limited by false negative rates. TTS is first managed with immobilization, neuroleptic drugs, and NSAIDs if no space occupying lesion is found on MRI. 1,2

    Answer C.

    References

    1. Wukich DK, Tuaxon DA. Diagnosis and Treatment of Chronic Ankle Pain. The Journal of Bone & Joint Surgery – Selected Instructional Course Lecture: 18 August 2010 – Volume 92 – Issue 10 – p. 2002-2016
    2. Lareau, Craig R. MD; Sawyer, Gregory A. MD; Wang, Joanne H. BA; DiGiovanni, Christopher W. MD Plantar and Medial Heel Pain, Journal of the American Academy of Orthopaedic Surgeons: June 2014 – Volume 22 – Issue 6 – p 372-380 doi: 10.5435/JAAOS-22-06-37.
  61. Question 61 of 64
    61. Question


    A 65-year-old male presents to your office with right knee pain and fullness behind the knee for 3 months. He has a known history of osteoarthritis in the knee and has had at least a few intra-articular steroid injections, with the most recent one two months ago. The knee has progressively become stiff and over the last few weeks he is having trouble bending the knee. He also has pain with full extension and is starting to get lower extremity edema in the right leg only over the last week.  Sagittal and axial MRI images in figures 1 and 2 show a large popliteal cyst behind the knee. What is the next best step in treatment?

    Correct

    A popliteal synovial cyst, or a Baker’s cyst, is a fluid filled cyst that forms behind the knee capsule as a result of an intra-articular disorder. The most common cause of a Baker’s cyst is knee osteoarthritis, followed by a meniscus tear. Intra-articular conditions such as osteoarthritis cause inflammation and fluid to build up in the knee and this fluid can escape out of the knee and encapsulate behind the knee. A one-way valve is created from the knee to the cyst as knee fluid can make the cyst bigger but the fluid usually doesn’t return to the knee. Baker’s cyst are often asymptomatic but they can grow large enough to create fullness behind the knee and knee stiffness. The cysts are usually unilocular but can be multilocular as well. In rare cases large Baker’s cyst can cause thrombophlebitis, compartment syndrome, compressive neuropathies, and lower extremity edema due to venous obstruction. Treatment starts with identifying the underlying condition. An intra-articular steroid injection is the most common first step which has been shown to decrease the size of the cyst. If the cyst becomes large enough an ultrasound guided aspiration may be performed but patients should be aware this may be a short term fix as the re-occurrence rate is high. The amount of fluid volume drained is proportional to the amount of pain relief achieved after aspiration. Bottom line, large symptomatic Baker’s cyst are one of the few types that are reasonable to drain. 1,2

    Answer B.

    References

    1. Frush TJ, Noyes FR. Baker’s Cyst: Diagnostic and Surgical Considerations. Sports Health. 2015;7(4):359-365. doi:10.1177/1941738113520130
    2. Köroğlu M, Çallıoğlu M, Eriş HN, Kayan M, Çetin M, Yener M, Gürses C, Erol B, Türkbey B, Parlak AE, Akhan O. Ultrasound guided percutaneous treatment and follow-up of Baker’s cyst in knee osteoarthritis. European journal of radiology. 2012 Nov 1;81(11):3466-71.
    Incorrect

    A popliteal synovial cyst, or a Baker’s cyst, is a fluid filled cyst that forms behind the knee capsule as a result of an intra-articular disorder. The most common cause of a Baker’s cyst is knee osteoarthritis, followed by a meniscus tear. Intra-articular conditions such as osteoarthritis cause inflammation and fluid to build up in the knee and this fluid can escape out of the knee and encapsulate behind the knee. A one-way valve is created from the knee to the cyst as knee fluid can make the cyst bigger but the fluid usually doesn’t return to the knee. Baker’s cyst are often asymptomatic but they can grow large enough to create fullness behind the knee and knee stiffness. The cysts are usually unilocular but can be multilocular as well. In rare cases large Baker’s cyst can cause thrombophlebitis, compartment syndrome, compressive neuropathies, and lower extremity edema due to venous obstruction. Treatment starts with identifying the underlying condition. An intra-articular steroid injection is the most common first step which has been shown to decrease the size of the cyst. If the cyst becomes large enough an ultrasound guided aspiration may be performed but patients should be aware this may be a short term fix as the re-occurrence rate is high. The amount of fluid volume drained is proportional to the amount of pain relief achieved after aspiration. Bottom line, large symptomatic Baker’s cyst are one of the few types that are reasonable to drain. 1,2

    Answer B.

    References

    1. Frush TJ, Noyes FR. Baker’s Cyst: Diagnostic and Surgical Considerations. Sports Health. 2015;7(4):359-365. doi:10.1177/1941738113520130
    2. Köroğlu M, Çallıoğlu M, Eriş HN, Kayan M, Çetin M, Yener M, Gürses C, Erol B, Türkbey B, Parlak AE, Akhan O. Ultrasound guided percutaneous treatment and follow-up of Baker’s cyst in knee osteoarthritis. European journal of radiology. 2012 Nov 1;81(11):3466-71.
  62. Question 62 of 64
    62. Question


    A 65-year-old male presents to the emergency department with worsening lower back pain 4 months out from a right sided L3-L4 discectomy. The pain from surgery never really subsided and seems to be getting worse over the last 3-4 weeks. He denies having fevers but has noticed intermittent chills and sweats the last few weeks as well. He is hemodynamically stable and is afebrile in the ED at 98.1 F. On physical exam he has no neurological deficits in the lower extremity and his prior lumbar incision is well healed without signs of infection. Blood cultures in the emergency department are positive for Staph aureus. MRI of the lumbar spine (figure 1) shows bone marrow edema at the L3 and L4 vertebral bodies and fluid signal in the intervertebral space compatible with acute discitis-osteomyelitis with surrounding prevertebral phlegmon and edema. There is no epidural abscess or neural compression. What is the next best step in treatment?

    Correct

    The patient is presenting with vertebral osteomyelitis/ discitis which was caused by a postoperative infection. Other causes of discitis other than surgical inoculation include hematogenous dissemination (most common) which may be from intravenous drug use or a recent systemic infection. The most common pathogen is Staph aureus followed by Staph epidermidis and then gram-negative organisms associated with respiratory or genital urinary infections. The most common symptoms of discitis include diffuse lower back pain that is worse at night. The most common location for discitis includes the lumbar spine followed by thoracic, then cervical and lastly, the sacrum. Fever is present in one-third of patients and neurological symptoms occur in less than 20% of patients. Treatment involves identifying the causative organism before antibiotics are administered, either by blood culture or CT guided bone biopsy. If the blood cultures are positive than a CT biopsy can be avoided. Surgical management is recommended when there is evidence of neurological deficits, large abscess formation, or spinal instability. 1,2

    Answer A.

    References
    1. Ahsan K, Hasan S, Khan SI, Zaman N, Almasri SS, Ahmed N, Chaurasia B. Conservative versus operative management of postoperative lumbar discitis. J Craniovertebr Junction Spine. 2020 Jul-Sep;11(3):198-209. doi: 10.4103/jcvjs.JCVJS_111_20. Epub 2020 Aug 14. PMID: 33100770; PMCID: PMC7546051.
    2. Adult pyogenic vertebral osteomyelitis. http://www.orthobullets.com. Accessed on 9/10/22.

    Incorrect

    The patient is presenting with vertebral osteomyelitis/ discitis which was caused by a postoperative infection. Other causes of discitis other than surgical inoculation include hematogenous dissemination (most common) which may be from intravenous drug use or a recent systemic infection. The most common pathogen is Staph aureus followed by Staph epidermidis and then gram-negative organisms associated with respiratory or genital urinary infections. The most common symptoms of discitis include diffuse lower back pain that is worse at night. The most common location for discitis includes the lumbar spine followed by thoracic, then cervical and lastly, the sacrum. Fever is present in one-third of patients and neurological symptoms occur in less than 20% of patients. Treatment involves identifying the causative organism before antibiotics are administered, either by blood culture or CT guided bone biopsy. If the blood cultures are positive than a CT biopsy can be avoided. Surgical management is recommended when there is evidence of neurological deficits, large abscess formation, or spinal instability. 1,2

    Answer A.

    References
    1. Ahsan K, Hasan S, Khan SI, Zaman N, Almasri SS, Ahmed N, Chaurasia B. Conservative versus operative management of postoperative lumbar discitis. J Craniovertebr Junction Spine. 2020 Jul-Sep;11(3):198-209. doi: 10.4103/jcvjs.JCVJS_111_20. Epub 2020 Aug 14. PMID: 33100770; PMCID: PMC7546051.
    2. Adult pyogenic vertebral osteomyelitis. http://www.orthobullets.com. Accessed on 9/10/22.

  63. Question 63 of 64
    63. Question


    A 28-year-old male presents to your office with left knee pain after a fall five days prior. He was stepping off his truck and his knee hit a steel railing. He had a fair amount of pain initially, but he has able to walk fine after and overall the pain is improving. On physical exam he has no pain over the patella and is able to straight leg raise without discomfort. He went to an urgent care yesterday to have the knee checked out and was diagnosed with a patella fracture. AP, lateral, and merchant view x-rays are shown in figures 1, 2 and 3, respectively. He was placed in a knee immobilizer with crutches. What is the next best step in treatment?

    Correct

    The patient has a classic bipartite patella on x-ray which was misdiagnosed as a patella fracture at the urgent care he went to. A bipartite patella is a congenital abnormality caused by failure of the patella growth plate to fuse. The abnormality is usually asymptomatic and found incidentally on x-ray. The x-ray of a bipartite patella classically shows a bony island at the superolateral pole of the patella. Most (75%) are located at the superolateral portion of the patella, 20% are lateral margin and 5% are inferior pole. A bipartite patella has smooth edges in between the bone island and main patella which helps differentiate from a fracture. A painful bipartite patella is rare and may be caused by a traumatic injury to the knee that disrupts the fibrocartilaginous connection between the main patella and accessory fragment. The presence of edema on MRI along the fibrocartilaginous connection can confirm an acute injury. Surgical excision of the accessary bone is only considered in patients with persistent pain beyond 6 months. Initially, patients should be treated with local ice, oral anti-inflammatories, and activities to tolerance. 1,2
    Answer C.
    References
    1. Iossifidis A, Brueton RN. Painful bipartite patella following injury. Injury. 1995 Apr 1;26(3):175-6.
    2. Bipartite patella. http://www.orthobullets.com. Accessed on 9/9/22.

    Incorrect

    The patient has a classic bipartite patella on x-ray which was misdiagnosed as a patella fracture at the urgent care he went to. A bipartite patella is a congenital abnormality caused by failure of the patella growth plate to fuse. The abnormality is usually asymptomatic and found incidentally on x-ray. The x-ray of a bipartite patella classically shows a bony island at the superolateral pole of the patella. Most (75%) are located at the superolateral portion of the patella, 20% are lateral margin and 5% are inferior pole. A bipartite patella has smooth edges in between the bone island and main patella which helps differentiate from a fracture. A painful bipartite patella is rare and may be caused by a traumatic injury to the knee that disrupts the fibrocartilaginous connection between the main patella and accessory fragment. The presence of edema on MRI along the fibrocartilaginous connection can confirm an acute injury. Surgical excision of the accessary bone is only considered in patients with persistent pain beyond 6 months. Initially, patients should be treated with local ice, oral anti-inflammatories, and activities to tolerance. 1,2
    Answer C.
    References
    1. Iossifidis A, Brueton RN. Painful bipartite patella following injury. Injury. 1995 Apr 1;26(3):175-6.
    2. Bipartite patella. http://www.orthobullets.com. Accessed on 9/9/22.

  64. Question 64 of 64
    64. Question


    A 15 year old male presents to your office with left ankle pain after twisting his ankle on a hike in the woods a day earlier. He has had difficulty with weightbearing on the ankle since. On physical exam he has mild swelling over the medial ankle and pain to palpation of the medial and lateral malleolus. AP and lateral radiographs are shown in figures 1 and 2. What is the next best step in treatment for this patient?

    Correct

    The patient has a Salter Harris IV fracture of the medial malleolus as the fracture extends from the epiphysis through the physis and includes a small metaphyseal fragment. The fracture was found to be displaced 5 mm on CT which is well beyond the acceptable 2mm of displacement allowed for conservative treatment. Articular step off greater than 2mm is associated with an increased risk of post-traumatic arthritis. CT imaging is recommended for all patients suspected of having an articular step off. CT can often change the course of treatment as determining fracture pattern and displacement can be very difficult on plain x-ray. This patient is approaching skeletal majority as the distal tibial physis usually closes by age 16 in males. As such, screw fixation from the medial malleolus through the physis can be performed without fear of causing growth arrest. This patients surgery required an open reduction as his thick periosteum blocked attempts at closed reduction. 1,2

     

    Answer C.

    1. Su AW, Larson AN. Pediatric Ankle Fractures: Concepts and Treatment Principles. Foot Ankle Clin. 2015 Dec;20(4):705-19. doi: 10.1016/j.fcl.2015.07.004. Epub 2015 Oct 16. PMID: 26589088; PMCID: PMC4912125.
    2. Pediatric ankle fractures. http://www.orthobullets.com. Accessed on 8/2/22.
    Incorrect

    The patient has a Salter Harris IV fracture of the medial malleolus as the fracture extends from the epiphysis through the physis and includes a small metaphyseal fragment. The fracture was found to be displaced 5 mm on CT which is well beyond the acceptable 2mm of displacement allowed for conservative treatment. Articular step off greater than 2mm is associated with an increased risk of post-traumatic arthritis. CT imaging is recommended for all patients suspected of having an articular step off. CT can often change the course of treatment as determining fracture pattern and displacement can be very difficult on plain x-ray. This patient is approaching skeletal majority as the distal tibial physis usually closes by age 16 in males. As such, screw fixation from the medial malleolus through the physis can be performed without fear of causing growth arrest. This patients surgery required an open reduction as his thick periosteum blocked attempts at closed reduction. 1,2

     

    Answer C.

    1. Su AW, Larson AN. Pediatric Ankle Fractures: Concepts and Treatment Principles. Foot Ankle Clin. 2015 Dec;20(4):705-19. doi: 10.1016/j.fcl.2015.07.004. Epub 2015 Oct 16. PMID: 26589088; PMCID: PMC4912125.
    2. Pediatric ankle fractures. http://www.orthobullets.com. Accessed on 8/2/22.
Proudly powered by WordPress | Theme: Motif by WordPress.com.